Exam 2 Review Questions version 2 Flashcards

1
Q

The nurse is caring for a patient who was admitted to the telemetry unit with a diagnosis of rule/out acute MI. The patient’s chest pain began 3 hours ago. Which of the following laboratory tests would be most helpful in confirming the diagnosis of a current MI?

a) Creatinine kinase-myoglobin (CK-MB) level
b) CK-MM
c) Troponin C level
d) Myoglobin level

A

a) Creatinine kinase-myoglobin (CK-MB) level
Explanation:
Elevated CK-MB assessment by mass assay is an indicator of acute MI; the levels begin to increase within a few hours and peak within 24 hours of an MI. If the area is reperfused (due to thrombotic therapy or PCI), it peaks earlier. CK-MM (skeletal muscle) is not an indicator of cardiac muscle damage. There are three isomers of troponin: C, I, and T. Troponin I and T are specific for cardiac muscle, and these biomarkers are currently recognized as reliable and critical markers of myocardial injury. An increase in myoglobin is not very specific in indicating an acute cardiac event; however, negative results are an excellent parameter for ruling out an acute MI.

How well did you know this?
1
Not at all
2
3
4
5
Perfectly
2
Q

Which of the following nursing interventions should a nurse perform to reduce cardiac workload in a patient diagnosed with myocarditis?

a) Maintain the patient on bed rest.
b) Elevate the patient’s head.
c) Administer supplemental oxygen.
d) Administer a prescribed antipyretic.

A

a) Maintain the patient on bed rest.
Explanation:
The nurse should maintain the patient on bed rest to reduce cardiac workload and promote healing. Bed rest also helps decrease myocardial damage and the complications of myocarditis. The nurse should administer supplemental oxygen to relieve tachycardia that may develop from hypoxemia. If the patient has a fever, the nurse should administer a prescribed antipyretic along with independent nursing measures such as minimizing layers of bed linen, promoting air circulation and evaporation of perspiration, and offering oral fluids. The nurse should elevate the patient’s head to promote maximal breathing potential.

How well did you know this?
1
Not at all
2
3
4
5
Perfectly
2
Q

A nurse is caring for a patient who experienced an MI. The patient is ordered metoprolol (Lopressor). The nurse understands that the therapeutic effect of this medication is which of the following?

a) Decreases cholesterol level
b) Decreases platelet aggregation
c) Increases cardiac output
d) Decreases resting heart rate

A

d) Decreases resting heart rate
Explanation:
The therapeutic effects of beta-adrenergic blocking agents such as metoprolol are to reduce the myocardial oxygen consumption by blocking beta-adrenergic sympathetic stimulation to the heart. The result is reduced heart rate, slowed conduction of impulses through the conduction system, decreased blood pressure, and reduced myocardial contractility to balance the myocardial oxygen needs and amount of oxygen available. This helps to control chest pain and delays the onset of ischemia during work or exercise. This classification of medication also reduces the incidence of recurrent angina, infarction, and cardiac mortality. Generally the dosage of medication is titrated to achieve a resting heart rate of 50–60 bpm. Metoprolol is not administered to decrease cholesterol levels, increase cardiac output, or decrease platelet aggregation.

How well did you know this?
1
Not at all
2
3
4
5
Perfectly
3
Q

A thoracentesis is performed to obtain a sample of pleural fluid or a biopsy specimen from the pleural wall for diagnostic purposes. What does bloody fluid indicate?

a) Emphysema
b) Trauma
c) Malignancy
d) Infection

A

c) Malignancy

Explanation:
A thoracentesis may be performed to obtain a sample of pleural fluid or to biopsy a specimen from the pleural wall for diagnostic purposes. The fluid, which may be clear, serous, bloody, or purulent, provides clues to the pathology. Bloody fluid may indicate malignancy, whereas purulent fluid usually indicates an infection. Pneumothorax, tension pneumothorax, subcutaneous emphysema, and pyogenic infection are complications of a thoracentesis. Pulmonary edema or cardiac distress can occur after a sudden shift in mediastinal contents when large amounts of fluid are aspirated.

How well did you know this?
1
Not at all
2
3
4
5
Perfectly
3
Q

Which type of oxygen therapy includes the administration of oxygen at pressure greater than 1 atmosphere?

a) Transtracheal
b) Low-flow systems
c) High-flow systems
d) Hyperbaric

A

d) Hyperbaric

Explanation:
Hyperbaric oxygen therapy is the administration of oxygen at pressures greater than 1 atmosphere. As a result, the amount of oxygen dissolved in plasma is increased, which increases oxygen levels in the tissues. Low-flow systems contribute partially to the inspired gas the patient breathes, which means that the patient breathes some room air along with the oxygen. High-flow systems are indicated for patients who require a constant and precise amount of oxygen. During transtracheal oxygenation, patients achieve adequate oxygenation at lower rates, making this method less expensive and more efficient.

How well did you know this?
1
Not at all
2
3
4
5
Perfectly
3
Q

In general, chest drainage tubes are not indicated for a patient undergoing which of the following procedures?

a) Wedge resection
b) Segmentectomy
c) Lobectomy
d) Pneumonectomy

A

d) Pneumonectomy

Explanation:
Usually, no drains are used for the patient having a pneumonectomy because the accumulation of fluid in the empty hemothorax prevents mediastinal shift. With lobectomy, two chest tubes are usually inserted for drainage, the upper tube for air and the lower tube for fluid. With wedge resection, the pleural cavity usually is drained because of the possibility of an air or blood leak. With segmentectomy, drains are usually used because of the possibility of an air or blood leak.

How well did you know this?
1
Not at all
2
3
4
5
Perfectly
3
Q

A patient is scheduled to have a cholecystectomy. Which of the nurse’s finding is least likely to contribute to surgical complications?

a) Pregnancy
b) Osteoporosis
c) Urinary tract infection
d) Diabetes

A

b) Osteoporosis
Explanation:
Osteoporosis is most likely not going to contribute to complications related to a cholecystectomy. Pregnancy decreases maternal reserves. Diabetes increases wound-healing problems and risks for infection. Urinary tract infection decreases the immune system, increasing the chance for infections.

How well did you know this?
1
Not at all
2
3
4
5
Perfectly
3
Q

The nurse is educating a patient scheduled for elective surgery. The patient currently takes aspirin daily. What education should the nurse provide in regard to the medication?

a) Continue to take the aspirin as ordered.
b) Stop taking the aspirin 7 days before the surgery, unless otherwise directed by your physician.
c) Take half doses of the aspirin until 1 week after surgery.
d) Aspirin should be increased until 3 days before surgery, and then it should be discontinued until 3 days after surgery.

A

b) Stop taking the aspirin 7 days before the surgery, unless otherwise directed by your physician.
Explanation:
Aspirin should be stopped at least 7 to 10 days before surgery. The other directions provided are incorrect

How well did you know this?
1
Not at all
2
3
4
5
Perfectly
3
Q

Which statement about an institutional ethics
committee is correct?
1. The ethics committee would be the first option
in addressing an ethical dilemma.
2. The ethics committee replaces decision making
by the patient and health care providers.
3. The ethics committee relieves health care
professionals from dealing with ethical issues.
4. The ethics committee provides education, policy
recommendations, and case consultation

A
  1. The ethics committee provides education, policy recommendations, and case consultation

The ethics committee is an additional resource
for patients and health care professionals.

How well did you know this?
1
Not at all
2
3
4
5
Perfectly
3
Q

The client at greatest risk for postoperative wound infection is:

  1. A 3-month-old infant postoperative from pyloric stenosis repair
  2. A 78-year-old postoperative from inguinal hernia repair
  3. An 18-year-old drug user postoperative from removal of a bullet in the leg
  4. A 32-year-old diabetic postoperative from an appendectomy
A
  1. An 18-year-old drug user postoperative from removal of a bullet in the leg; All are at risk for infection. Answer 3 is at greatest risk, because the bullet is unclean, and a drug user is at great risk for immune deficiency.
How well did you know this?
1
Not at all
2
3
4
5
Perfectly
4
Q

The nurse is providing discharge instructions to a patient following nasal surgery who has nasal packing. Which of the following discharge instructions would be most appropriate for the patient?

a) Administer normal saline nasal drops as ordered.
b) Decrease the amount of daily fluids.
c) Decrease the amount of daily fluids.
d) Avoid sports activities for 6 weeks.

A

d) Avoid sports activities for 6 weeks.

Explanation:
The nurse instructs the patient to avoid sports activities for 6 weeks. There is no indication for the patient to refrain from taking oral fluids. Mouth rinses help to moisten the mucous membranes and to reduce the odor and taste of dried blood in the oropharynx and nasopharynx. The patient should take analgesic agents, such as acetaminophen or NSAIDs, (i.e., ibuprofen or naproxen) to decrease nasal discomfort, not aspirin. The patient does not need to use nasal drops when nasal packing is in place.

How well did you know this?
1
Not at all
2
3
4
5
Perfectly
4
Q

The nurse is caring for a patient in the ICU diagnosed with coronary artery disease (CAD). Which of the following assessment data indicates the patient is experiencing a decrease in cardiac output?

a) Disorientation, 20 mL of urine over the last 2 hours
b) BP 108/60 mm Hg, ascites, and crackles
c) Elevated jugular venous distention (JVD) and postural changes in BP
d) Reduced pulse pressure and heart murmur

A

a) Disorientation, 20 mL of urine over the last 2 hours
Explanation:
Assessment findings associated with reduced cardiac output include reduced pulse pressure, hypotension, tachycardia, reduced urine output, lethargy, or disorientation.

How well did you know this?
1
Not at all
2
3
4
5
Perfectly
4
Q

The nurse is caring for a patient in the postanesthesia care unit (PACU) with the following vital signs, pulse 115, respiration 20, temperature 97.2°F oral, blood pressure 84/50. What should the nurse do first?

a) Review the patient’s preoperative vital signs.
b) Increase rate of IV fluids.
c) Assess for bleeding.
d) Notify the physician.

A

c) Assess for bleeding.
Explanation:
The patient is tachycardic with a low blood pressure; thus assessing for hemorrhage is the priority action. While the physician may need to be notified, the nurse needs to be able to communicate a complete picture of the patient, which would include bleeding, when calling the physician. The rate of IV fluid administration should be adjusted according to a physician order. The nurse should review prior vital signs but only after the immediate threat of hemorrhage is assessed.

How well did you know this?
1
Not at all
2
3
4
5
Perfectly
5
Q

A patient comes to the clinic with complaints of fever, chills, and sore throat and is diagnosed with streptococcal pharyngitis. A nurse knows that early diagnosis and effective treatment is essential to avoid which of the following preventable diseases?

a) Pericarditis
b) Mitral stenosis
c) Rheumatic fever
d) Cardiomyopathy

A

c) Rheumatic fever
Explanation:
Rheumatic fever is a preventable disease. Diagnosing and effectively treating streptococcal pharyngitis can prevent rheumatic fever and, therefore, rheumatic heart disease.

How well did you know this?
1
Not at all
2
3
4
5
Perfectly
6
Q

Which type of cell is believed to play a significant role in cutaneous immune system reactions?

a) Phagocytes
b) Langerhans cells
c) Melanocytes
d) Merkel cells

A

b) Langerhans cells

Explanation:
Langerhans cells, which are common to the epidermis, are accessory cells of the afferent immune system process. Merkel cells are the receptor cells in the epidermis that transmit stimuli to the axon via a chemical response. Melanocytes are special cells of the epidermis that are primarily involved in producing melanin, which colors the hair and skin. Phagocytes are white blood cells that engulf and destroy foreign materials.

How well did you know this?
1
Not at all
2
3
4
5
Perfectly
6
Q

The nurse is caring for a patient with diabetes who is scheduled for a cardiac catheterization. Prior to the procedure, it is most important for the nurse to ask which of the following questions?

a) “When was the last time you ate or drank?”
b) “Are you having chest pain?”
c) “What was your morning blood sugar reading?”
d) “Are you allergic to shellfish?”

A

d) “Are you allergic to shellfish?”
Explanation:
Radiopaque contrast agents are used to visualize the coronary arteries. Some contrast agents contain iodine, and the patient is assessed before the procedure for previous reactions to contrast agents or allergies to iodine-containing substances (e.g., seafood). If the patient has a suspected or known allergy to the substance, antihistamines or methylprednisolone (Solu-Medrol) may be administered before the procedure. Although the other questions are important to ask the patient, it is most important to ascertain if the patient has an allergy to shellfish.

How well did you know this?
1
Not at all
2
3
4
5
Perfectly
6
Q

Which of the following statements are true when the nurse is measuring blood pressure (BP)? Select all that apply.

a) Ask the patient to sit quietly while the BP is being measured.
b) The patient’s arm should be positioned at the level of the heart.
c) Using a BP cuff that is too small will give a higher BP measurement.
d) The patient’s BP should be taken 1 hour after the consumption of alcohol.
e) Using a BP cuff that is too large will give a higher BP measurement.

A

a) Ask the patient to sit quietly while the BP is being measured., b) The patient’s arm should be positioned at the level of the heart., c) Using a BP cuff that is too small will give a higher BP measurement.
Explanation:
These statements are all true when measuring a BP. When using a BP cuff that is too large the reading will be lower than the actual BP. The patient should avoid smoking cigarettes or drinking caffeine for 30 minutes before BP is measured.

How well did you know this?
1
Not at all
2
3
4
5
Perfectly
6
Q

A patient is admitted to the hospital with possible acute pericarditis and pericardial effusion. The nurse knows to prepare the patient for which diagnostic test used to confirm the patient’s diagnosis?

a) CT scan
b) Chest x-ray
c) Cardiac cauterization
d) Echocardiogram

A

d) Echocardiogram
Explanation:
Echocardiograms are useful in detecting the presence of the pericardial effusions associated with pericarditis. An echocardiogram may detect inflammation, pericardial effusion, tamponade, and heart failure. It may help confirm the diagnosis.

How well did you know this?
1
Not at all
2
3
4
5
Perfectly
6
Q

The nurse is changing the dressing of a chronic wound. There is no sign of infection or heavy drainage. How long will the nurse leave the wound covered for?

  1. 6-12 hr
  2. 12-24 hr
  3. 24-36 hr
  4. 48-72 hr48
A

48-72 hr

How well did you know this?
1
Not at all
2
3
4
5
Perfectly
6
Q

An appropriate nursing diagnosis for a client with large areas of skin excoriation resulting from scratching an allergic rash is:

  1. Risk for Impaired Skin Integrity
  2. Impaired Skin Integrity
  3. Impaired Tissue Integrity
  4. Risk for Infection
A
  1. Impaired Skin Integrity; The client has an actual impairment of the skin due to the rash and the scratching so is no longer “at risk”.

Because the damage is at the skin level, it is not impaired tissue integrity (option 3) since that would involve deeper tissues. Surface excoriation is also not prone to becoming infected.

How well did you know this?
1
Not at all
2
3
4
5
Perfectly
7
Q

Which of the following is a factor that causes wrinkles among older adults?

a) Decrease in sebum
b) Loss of the subcutaneous tissue
c) Decrease in the production of estrogen
d) Decrease in melanin

A

b) Loss of the subcutaneous tissue

Explanation:
The loss of the subcutaneous tissue causes wrinkles in older adults. The decrease in melanin results in a change of hair color to gray. The decrease in the production of estrogen and sebum do not cause wrinkles in older adults.

How well did you know this?
1
Not at all
2
3
4
5
Perfectly
8
Q

A patient with a history of alcoholism and scheduled for an urgent surgery asks the nurse, “Why is everyone so concerned about how much I drink?” What is the best response by the nurse?

a) “The amount of alcohol you drink will determine the amount of pain medication you will need postoperatively.”
b) “We can have counselors available after surgery; if it is determined you need help for your drinking.”
c) “It is a required screening question for all patients having surgery.”
d) “It is important for us to know how much and how often you drink to help prevent surgical complications.”

A

d) “It is important for us to know how much and how often you drink to help prevent surgical complications.”
Explanation:
Alcohol use and alcoholism can contribute to serious postoperative complications. If the medical and nursing staff is aware of the use or abuse, measures can be implemented proactively to prevent complications. Although alcohol may interfere with a medication’s effectiveness, it does not determine the amount of pain medications that are prescribed following surgery. Even though this is a required screening question and counselors can be made available for those who want help, those are not the best responses to answer the patient’s question.

How well did you know this?
1
Not at all
2
3
4
5
Perfectly
9
Q

The nurse is having difficulty seeing a patient’s rash. To facilitate the assessment, the nurse should do which of the following? Select all that apply.

a) Stretch the skin gently.
b) Apply an emollient.
c) Pull the skin in a downward position.
d) To facilitate assessment of the rash, the nurse should stretch the skin gently and/or point a penlight laterally across the skin. The skin should never be pulled; applying an emollient will increase the nurse’s difficulty in assessing the rash.
e) Point a penlight laterally across the affected part.

A

a) Stretch the skin gently., e) Point a penlight laterally across the affected part.

Explanation:
To facilitate assessment of the rash, the nurse should stretch the skin gently and/or point a penlight laterally across the skin. The skin should never be pulled; applying an emollient will increase the nurse’s difficulty in assessing the rash.

How well did you know this?
1
Not at all
2
3
4
5
Perfectly
10
Q

For both outpatients and inpatients scheduled for diagnostic procedures of the cardiovascular system, the nurse performs a thorough initial assessment to establish accurate baseline data. Which of the following data is necessary to collect if the patient is experiencing chest pain?

a) Blood pressure in the left arm
b) Description of the pain
c) Sound of the apical pulses
d) Pulse rate in upper extremities

A

b) Description of the pain

Explanation:
If the patient is experiencing chest pain, a history of its location, frequency, and duration is necessary, as is a description of the pain, if it radiates to a particular area, what precipitates its onset, and what brings relief. The nurse weighs the patient and measures vital signs. The nurse may measure BP in both arms and compare findings. The nurse assesses apical and radial pulses, noting rate, quality, and rhythm. The nurse also checks peripheral pulses in the lower extremities.

How well did you know this?
1
Not at all
2
3
4
5
Perfectly
10
Q

A patient complaining of heart palpitations is diagnosed with atrial fibrillation caused by mitral valve prolapse. In order to relieve the symptoms, the nurse should teach the patient which of the following dietary interventions?

a) Eliminate caffeine and alcohol
b) Decrease the amount of sodium and saturated fat.
c) Decrease the amount of acidic beverages and fruits.
d) A patient complaining of heart palpitations is diagnosed with atrial fibrillation caused by mitral valve prolapse. In order to relieve the symptoms, the nurse should teach the patient which of the following dietary interventions?

A

a) Eliminate caffeine and alcohol
Explanation:
To minimize symptoms of mitral valve prolapse, the nurse should instruct the patient to avoid caffeine and alcohol. The nurse encourages the patient to read product labels, particularly on over-the-counter products such as cough medicine, because these products may contain alcohol, caffeine, ephedrine, and epinephrine, which may produce dysrhythmias and other symptoms. The nurse also explores possible diet, activity, sleep, and other lifestyle factors that may correlate with symptoms.

How well did you know this?
1
Not at all
2
3
4
5
Perfectly
11
Q

The nurse is caring for a patient with clubbing of the fingers and toes. The nurse should complete which of the following actions given these findings?

a) Assess the patient’s capillary refill.
b) Obtain an oxygen saturation level.
c) Assess the patient for pitting edema.
d) Obtain a 12-lead ECG tracing

A

b) Obtain an oxygen saturation level.
Explanation:
Clubbing of the fingers and toes indicates chronic hemoglobin desaturation (decreased oxygen supply) and is associated with congenital heart disease. The nurse should assess the patient’s O2 saturation level and intervene as directed. The other assessments are not indicated.

How well did you know this?
1
Not at all
2
3
4
5
Perfectly
11
Q

The nurse is educating the pt in the use of a mini-nebulizer. What should the nurse encourage the student to do? (Select all that apply)

  1. Hold the breath at the end of inspiration for a few seconds
  2. Cough frequently
  3. Take rapid, deep breaths
  4. Frequently evaluate progress
  5. Prolong the expiratory phase after using nebulizer
A

Hold the breath at the end of the inspiration for a few seconds,

Cough frequently

Frequently evaluate progress

How well did you know this?
1
Not at all
2
3
4
5
Perfectly
12
Q

The nurse is instructing unlicensed personnel on gerontologic considerations of the skin. The nurse finds that the participants understand the instructions when they know that the elderly are at a higher risk for shear injuries due to which of the following?

a) Loss of subcutaneous tissue
b) Sun damage over time
c) Decreased capillary loops
d) Loss of rete ridges

A

d) Loss of rete ridges

Explanation:
Elderly patients are at a higher risk for shear injuries due to loss of rete ridges from thinning at the junction of the dermis and epidermis. The loss of rete ridges (anchoring sites between the two skin layers) enables even minor injury/stress to the epidermis to cause it shear away from the dermis. The other answers do not apply.

How well did you know this?
1
Not at all
2
3
4
5
Perfectly
12
Q

A patient has been administered ketamine (Ketalar) for moderate sedation. What is the priority nursing intervention?

a) Frequent monitoring of vital signs
b) Administering oxygen
c) Providing a quiet dark room
d) Assessing for hallucinations

A

a) Frequent monitoring of vital signs
Explanation:
Vital signs must be monitored frequently to assess for respiratory depression and intervene quickly. Oxygen may need to be administered if respiratory depression occurs; therefore, monitoring vital signs is a higher priority nursing intervention. Providing a dark quiet room is appropriate after the procedure is completed and the patient is recovering. Hallucinations may be experienced as a side effect of the medication.

How well did you know this?
1
Not at all
2
3
4
5
Perfectly
12
Q

Which of the following is a physiological effect of
prolonged bed rest?
1. An increase in cardiac output
2. A decrease in lean body mass
3. A decrease in lung expansion
4. A decrease in urinary excretion of nitrogen

A

Immobility causing decreased lung elastic
recoiling and secretions accumulating in portions of
the lungs

How well did you know this?
1
Not at all
2
3
4
5
Perfectly
13
Q

A patient is admitted with aortic regurgitation. Which of the following medication classifications are contraindicated since they can cause bradycardia and decrease ventricular contractility?

a) Beta blockers
b) Nitrates
c) Ace inhibitors
d) Calcium channel blockers

A

d) Calcium channel blockers
Explanation:
The calcium channel blockers diltiazem (Cardizem) and verapamil (Calan, Isoptin) are contraindicated for patients with aortic regurgitation as they decrease ventricular contractility and may cause bradycardia.

How well did you know this?
1
Not at all
2
3
4
5
Perfectly
13
Q

The nurse working in the ED is evaluating a patient for signs and symptoms of appendicitis. Which of the patient’s signs/symptoms should the nurse include in the report to the physician on the patient’s signs/symptoms of appendicitis?

a) Pain when pressure is applied to the right lower quadrant of the abdomen
b) High fever
c) Nausea
d) Left lower quadrant pain

A

c) Nausea
Explanation:
Nausea is typically associated with appendicitis with or without vomiting. Pain is generally felt in the right lower quadrant. Rebound tenderness, or pain felt with release of pressure applied to the abdomen, may be present with appendicitis. Low-grade fever is associated with appendicitis.

How well did you know this?
1
Not at all
2
3
4
5
Perfectly
14
Q

A 76-year-old man presents to the ED complaining of “laryngitis.” The triage nurse should ask if the patient has a past medical history that includes which of the following?

a) Gastroesophageal reflux disease (GERD)
b) Respiratory failure (RF)
c) Chronic obstructive pulmonary disease (COPD)
d) Congestive heart failure (CHF)

A

a) Gastroesophageal reflux disease (GERD)

Explanation:
The nurse should ask if the patient has a past medical history of GERD. Laryngitis in the older adults is common and may be secondary to GERD. Older adults are more likely to have impaired esophageal peristalsis and a weaker esophageal sphincter. COPD, CHF, and RF are not associated with laryngitis in the older adult.

How well did you know this?
1
Not at all
2
3
4
5
Perfectly
15
Q

An 18-year-old woman is in the emergency department with fever and cough. The nurse obtains her vital signs, auscultates her lung sounds, listens to her heart sounds, determines her level of comfort, and collects blood and sputum samples for analysis. Which standard of practice is performed?

A. Diagnosis

B. Evaluation

C. Assessment

D. Implementation

A

C. Assessment

Assessment is the collection of comprehensive data pertinent to the patient’s health and/or the situation.

How well did you know this?
1
Not at all
2
3
4
5
Perfectly
15
Q

The factor that best advanced the practice of
nursing in the twenty-first century was:
1. Growth of cities
2. Teachings of Christianity
3. Better education of nurses
4. Improved conditions for women

A
  1. Better education of nurses

Nursing is a combination of knowledge from the
physical sciences, humanities, and social sciences
along with clinical competencies

How well did you know this?
1
Not at all
2
3
4
5
Perfectly
16
Q

A nurse is preparing to assess a patient for postural BP changes. Which of the following indicates the need for further education?

a) Obtaining the supine measurements prior to the sitting and standing measurements
b) Positioning the patient supine for 10 minutes prior to taking the initial BP and HR
c) Letting 30 seconds elapse after each position change before measuring BP and heart rate (HR)
d) Taking the patient’s BP with the patient sitting on the edge of the bed with feet dangling

A

c) Letting 30 seconds elapse after each position change before measuring BP and heart rate (HR)

Explanation:
The following steps are recommended when assessing patients for postural hypotension: Position the patient supine for 10 minutes before taking the initial BP and HR measurements; reposition the patient to a sitting position with legs in the dependent position, wait 2 minutes then reassess both BP and HR measurements; if the patient is symptom free or has no significant decreases in systolic or diastolic BP, assist the patient into a standing position, obtain measurements immediately and recheck in 2 minutes; continue measurements every 2 minutes for a total of 10 minutes to rule out postural hypotension. Return the patient to supine position if postural hypotension is detected or if the patient becomes symptomatic. Document HR and BP measured in each position (e.g., supine, sitting, and standing) and any signs or symptoms that accompany the postural changes.

How well did you know this?
1
Not at all
2
3
4
5
Perfectly
16
Q

When measuring the blood pressure in each of the patient’s arms, the nurse recognizes that in the healthy adult, which of the following is true?

a) Pressures may vary 10 mm Hg or more between arms.
b) Pressures may vary, with the higher pressure found in the left arm.
c) Pressures must be equal in both arms.
d) Pressures should not differ more than 5 mm Hg between arms.

A

d) Pressures should not differ more than 5 mm Hg between arms.
Explanation:
Normally, in the absence of disease of the vasculature, there is a difference of no more than 5 mm Hg between arm pressures. The pressures in each arm do not have to be equal in order to be considered normal. Pressures that vary more than 10 mm Hg between arms indicate an abnormal finding. The left arm pressure is not anticipated to be higher than the right as a normal anatomic variant.

How well did you know this?
1
Not at all
2
3
4
5
Perfectly
16
Q

Following a percutaneous coronary intervention (PCI), a patient is returned to the nursing unit with large peripheral vascular access sheaths in place. The nurse understands that which of the following methods to induce hemostasis after sheath is contraindicated?

a) Application of a sandbag to the area
b) Direct manual pressure
c) Application of a vascular closure device
d) Application of a mechanical compression device

A

a) Application of a sandbag to the area
Explanation:
Applying a sandbag to the sheath insertion site is ineffective in reducing the incidence of bleeding and is not an acceptable standard of care. Application of a vascular closure device (Angioseal, VasoSeal), direct manual pressure to the sheath introduction site, and application of a mechanical compression device (C-shaped clamp) are all appropriate methods used to induce hemostasis following peripheral sheath removal.

How well did you know this?
1
Not at all
2
3
4
5
Perfectly
16
Q

A pt is visiing the physician to determine what type of allergy is causing a rash. What type of testing does the nurse anticipate the physician will schedule?

  1. Skin biopsy
  2. Skin scrapings
  3. Tzanck smear
  4. Patch test
A

Patch test

How well did you know this?
1
Not at all
2
3
4
5
Perfectly
16
Q

A patient with an abdominal surgical wound sneezes and states, “Something doesn’t feel right with my wound.” The nurse asses the upper half of the surgical wounds edges are no longer approximated and the lower half remains well approximated. What documentation by the nurse is most appropriate?

a) Following a sneeze, the wound pustulated.
b) Following a sneeze, the wound hemorrhaged.
c) Following a sneeze, the wound dehisced.
d) Following a sneeze, the wound eviscerated.

A

c) Following a sneeze, the wound dehisced.

Explanation:
Dehiscence is the partial or complete separation of wound edges. Evisceration is the protrusion of organs through the surgical incision. Pustulated refers to the formation of pustules Hemorrhage is excessive bleeding.

How well did you know this?
1
Not at all
2
3
4
5
Perfectly
17
Q

The nurse assesses a patient who is bleeding profusely from the nose. The nurse documents this finding as which of the following conditions?

a) Rhinorrhea
b) Dysphagia
c) Epistaxis
d) Xerostomia

A

c) Epistaxis

Explanation:
Epistaxis is due to rupture of tiny, distended vessels in the mucous membrane of any area of the nose. Xerostomia refers to dryness of the mouth. Rhinorrhea refers to drainage of a large amount of fluid from the nose. Dysphagia refers to difficulties in swallowing.

How well did you know this?
1
Not at all
2
3
4
5
Perfectly
17
Q

When the nurse observes that the postoperative patient demonstrates a constant low level of oxygen saturation via the O2 saturation monitor, although the patient’s breathing appears normal what action should the nurse take first?

a) Notify the physician.
b) Assess the patient’s heart rhythm and nail beds.
c) Apply oxygen.
d) Document the findings.

A

b) Assess the patient’s heart rhythm and nail beds.

Explanation:
A patient may demonstrate low oxygenation readings due to wearing certain colors of nail polish or irregular heart rate such as atrial fibrillation. These items should be assessed to ensure the accuracy of the oxygen reading. Once the reading is confirmed as accurate, then the nurse may need to apply oxygen, notify the physician, and document the findings.

How well did you know this?
1
Not at all
2
3
4
5
Perfectly
18
Q

A patient is administered succinylcholine and propofol (Diprivan) for induction of anesthesia. One hour after administration, the patient is demonstrating muscle rigidity with a heart rate of 180. What should the nurse do first?

a) Administer dantrolene sodium (Dantrium).
b) Notify the surgical team.
c) Obtain cooling blankets.
d) Document the assessment findings.

A

b) Notify the surgical team.
Explanation:
Tachycardia and muscle rigidity is often the earliest sign of malignant hyperthermia. Early recognition of malignant hyperthermia increases survival. The nurse would document the findings, administer dantrolene sodium (Dantrium), obtain cooling blankets as part of the treatment for malignant hyperthermia, but the nurse would need to ensure the surgical team is aware of the findings first.

How well did you know this?
1
Not at all
2
3
4
5
Perfectly
19
Q

When caring for a postsurgical patient, the nurse observes that the client has hemorrhaged and is in hypovolemic shock. Which nursing intervention will manage and minimize hemorrhage and shock?

a) Reinforcing dressing or applying pressure if bleeding is frank
b) Monitoring vital signs every 15 minutes
c) Elevating the head of the bed
d) Encouraging the patient to breathe deeply

A

a) Reinforcing dressing or applying pressure if bleeding is frank
Explanation:
The nurse should reinforce the dressing or apply pressure if bleeding is frank. The nurse should keep the head of the bed flat unless it is contraindicated. Encouraging the patient to breathe deeply will not help manage and minimize hemorrhage and shock. Monitoring the vital signs every 15 minutes is an appropriate nursing intervention but will not minimize hemorrhage and shock; it will just help to determine the extent and progression of the problem.

How well did you know this?
1
Not at all
2
3
4
5
Perfectly
20
Q

The nurse is caring for a patient following a coronary artery bypass graft (CABG). The nurse notes persistent oozing of bloody drainage from various puncture sites. The nurse anticipates that the physician will order which of the following medications to neutralize the unfractionated heparin the patient received?

a) Aspirin
b) Clopidogrel (Plavix)
c) Protamine sulfate
d) Alteplase (t-PA)

A

c) Protamine sulfate
Explanation:
Protamine sulfate is known as the antagonist for unfractionated heparin (it neutralizes heparin). Alteplase is a thrombolytic agent. Clopidogrel (Plavix) is an antiplatelet medication that is given to reduce the risk of thrombus formation post coronary stent placement. The antiplatelet effect of aspirin does not reverse the effects of heparin.

How well did you know this?
1
Not at all
2
3
4
5
Perfectly
20
Q

The pt is advised to apply a suspension-type lotion to a dermatosis site. the nurse should advise the pt to apply the lotion how often to be effective?

  1. Every hour
  2. Every 3 hours
  3. Every 12 hours
  4. Every day at the same time
A

Every 3 hours

How well did you know this?
1
Not at all
2
3
4
5
Perfectly
21
Q

The nurse is reading the physician’s report of an elderly patient’s physical examination. The patient demonstrates xanthelasma, which refers to which of the following symptoms?

a) Dark discoloration of the skin
b) Bright red moles
c) Liver spots
d) Yellowish waxy deposits on upper eyelids

A

d) Yellowish waxy deposits on upper eyelids

Explanation:
Xanthelasma is a common, benign manifestation of aging skin or it can sometimes signal hyperlipidemia. The symptoms are yellowish waxy deposits on the upper eyelids. Solar lentigo is the term that refers to liver spots. Melasma is the term that refers to dark discoloration of the skin. Cherry angioma is the term that is used to describe a bright red mole.

How well did you know this?
1
Not at all
2
3
4
5
Perfectly
21
Q

A patient continuously states, “I know all will go well.” What cognitive coping strategy should the nurse document?

a) Music therapy
b) Optimistic self-recitation
c) Distraction
d) Imagery

A

b) Optimistic self-recitation
Explanation:
When that patient verbalizes this statement, it is an optimistic response. Imagery occurs when the patient concentrates on a pleasant experience or restful scene. Distraction occurs when the patient thinks of an enjoyable story or recites a favorite poem or song. Music therapy would be an incorrect answer.

How well did you know this?
1
Not at all
2
3
4
5
Perfectly
21
Q

A patient refuses to remove her wedding band when preparing for surgery. What is the best action for the nurse to take?

a) Allow the ring to stay on the patient and cover it with tape.
b) Remove the ring once the patient is sedated.
c) Notify the surgeon to cancel surgery.
d) Discuss the risk for infection caused by wearing the ring.

A

a) Allow the ring to stay on the patient and cover it with tape.
Explanation:
Most facilities will allow a wedding band to remain on the patient during the surgical procedure. The nurse must secure the ring with tape. Although it is appropriate to discuss the risk for infection, the patient has already refused removal of the ring. The surgery should not be canceled and the ring should not be removed without permission.

How well did you know this?
1
Not at all
2
3
4
5
Perfectly
21
Q

A patient is undergoing a lumbar puncture. The nurse educates the patient about surgical positioning. Which of the following statements by the nurse is appropriate?

a) “You will be on your back with the head of the bed at 30 degrees.”
b) “You will be placed flat on the table, face down.”
c) “You will be lying on your side with your knees to your chest.”
d) “You will be flat on your back with the table slanted so your head is below your feet.”

A

c) “You will be lying on your side with your knees to your chest.”
Explanation:
For the lumbar puncture procedure, the patient usually lies on the side in a knee-chest position. Flat on the table, face down does not open the vertebral spaces to allow access for the lumbar puncture. Having the patient lie on their back does not allow for access to the surgical site.

How well did you know this?
1
Not at all
2
3
4
5
Perfectly
22
Q

A nurse is caring for an older-adult couple in a community-based assisted living facility. During the family assessment he notes that the couple has many expired medications and multiple medications for their respective chronic illnesses. They note that they go to two different health care providers. The nurse begins to work with the couple to determine what they know about their medications and helps them decide on one care provider rather than two. This is an example of which Quality and Safety in the Education of Nurses (QSEN) competency?

A. Patient-centered care

B. Safety

C. Teamwork and collaboration

D. Informatics

A

B. Safety

Helping the patients understand the consequences and complications of multiple medications helps to build the competency in safety.

How well did you know this?
1
Not at all
2
3
4
5
Perfectly
22
Q

The nurse documenting an acute open wound should include which of the following? Select all that apply.

a) Periwound skin
b) Wound size
c) Pattern of eruption
d) Wound bed

A

a) Periwound skin, b) Wound size, d) Wound bed

Explanation:
When documenting an acute open wound, the nurse should consider the wound’s size, the condition of the periwound skin (skin surrounding the wound), and a description of the wound bed. The pattern of eruption relates to the patterns of lesions on a patient’s skin and does not apply to an acute open wound.

How well did you know this?
1
Not at all
2
3
4
5
Perfectly
22
Q

Which of the following is an autoimmune disease involving immunoglobulin G?

a) Bullous pemphigoid
b) Stevens-Johnson syndrome (SJS)
c) Pemphigus
d) Toxic epidural necrolysis (TEN)

A

c. Pemphigus

Explanation:
Pemphigus is an autoimmune disease involving immunoglobulin G. TEN, SJS, and bullous pemphigoid do not involve immunoglobulin G

How well did you know this?
1
Not at all
2
3
4
5
Perfectly
23
Q

In caring for a client on contact precautions for a draining infected foot ulcer, which action should the nurse perform?

  1. Wear a mask during dressing changes.
  2. Provide disposable meal trays and silverware.
  3. Follow standard precautions in all interactions with the client.
  4. Use surgical aseptic technique for all direct contact with the client.
A
  1. Follow standard precautions in all interactions with the client.

Rationale: Standard precautions include all aspects of contact precautions with the exception of placing the client in a private room. A mask is indicated when working over a sterile wound rather than an infected one (option 1). Disposable food trays are not necessary for clients with infected wounds unlikely to contaminate the client’s hands (option 2). Sterile technique (surgical asepsis) is not indicated for all contact with the client (option 4). The nurse would utilize clean technique when dressing the wound to prevent introduction of additional microbes.

How well did you know this?
1
Not at all
2
3
4
5
Perfectly
24
Q

The nurse is calculating the patient’s smoking history in pack-years. The patient has recently been diagnosed with malignant lung cancer. The patient states he has been smoking two packs of cigarettes a day for the past 11 years. The nurse correctly documents the patient’s pack-years as which of the following?

a) 11
b) 5
c) 22
d) 10

A

c) 22

Explanation:
Smoking history is usually expressed in pack-years, which is the number of packs of cigarettes smoked per day times the number of years the patient smoked. It is important to find out if the patient is still smoking or when the patient quit smoking. In this situation, the patient’s pack years is 22 (2 × 11).

How well did you know this?
1
Not at all
2
3
4
5
Perfectly
24
Q

The nurse is educating patients requiring surgery for various ailments on the perioperative experience. What education provided by the nurse is most appropriate?

a) Three phases of surgery and safety measures for each phase
b) Intraoperative techniques used to perform the surgery
c) Risks and benefits of the surgical procedures
d) Expected pain levels and narcotic pain medication used to treat the pain

A

a) Three phases of surgery and safety measures for each phase
Explanation:
The perioperative period includes the preoperative, intraoperative, and postoperative phases. Specific safety guidelines are followed for all surgical patients. The information provided should be general enough to be informative about surgery and should not focus on individual surgeries, as all the patients are having different surgeries. Intraoperative techniques, expected pain levels, and pain medication are specific to the patient and type of surgery. The risks and benefits of the surgical procedure should be discussed by the physician.

How well did you know this?
1
Not at all
2
3
4
5
Perfectly
24
Q

What is the highest priority nursing intervention for a patient in the immediate postoperative phase?

a) Maintaining a patent airway
b) Assessing for hemorrhage
c) Monitoring vital signs at least every 15 minutes
d) Assessing urinary output every hour

A

a) Maintaining a patent airway
Explanation:
All interventions listed are correct. The highest priority intervention is maintaining a patent airway. Without a patent airway, the other interventions of monitoring vital signs and urinary output, along with assessing for hemorrhage, become secondary to the possibility of a lack of oxygen.

How well did you know this?
1
Not at all
2
3
4
5
Perfectly
26
Q

The nurse is caring for a patient who has undergone peripheral arteriography. How should the nurse assess the adequacy of peripheral circulation?

a) By observing the patient for bleeding
b) By hemodynamic monitoring
c) By checking peripheral pulses
d) By checking for cardiac dysrhythmias

A

c) By checking peripheral pulses
Explanation:
Peripheral arteriography is used to diagnose occlusive arterial disease in smaller arteries. The nurse observes the patient for bleeding and cardiac dysrhythmias and assesses the adequacy of peripheral circulation by frequently checking the peripheral pulses. Hemodynamic monitoring is used to assess the volume and pressure of blood in the heart and vascular system.

How well did you know this?
1
Not at all
2
3
4
5
Perfectly
26
Q

A 35-year-old female patient has been diagnosed with hypertension. The patient is a stock broker, smokes daily, and is also a diabetic. During a follow-up appointment, the patient states that she finds it cumbersome and time consuming to visit the doctor regularly just to check her blood pressure (BP). As the nurse, which of the following aspects of patient teaching would you recommend?

a) Advising a smoking cessation
b) Purchasing a self-monitoring BP cuff
c) Discussing methods for stress reduction
d) Administering glycemic control

A

b) Purchasing a self-monitoring BP cuff
Explanation:
Because this patient finds it time consuming to visit the doctor just for a blood pressure reading, as the nurse, you can suggest the use of an automatic cuff at a local pharmacy, or purchasing a self-monitoring cuff. Discussing methods for stress reduction, advising a smoking cessation, and administering glycemic control would constitute patient education in managing hypertension.

How well did you know this?
1
Not at all
2
3
4
5
Perfectly
26
Q

The nurse observes an African-American pt with a large hypertrophied area of scar tissue on the left ear lobe. What does the nurse document this a finding as?

  1. Atrophy
  2. Scar
  3. Lichenification
  4. Keloid
A

Keloid

How well did you know this?
1
Not at all
2
3
4
5
Perfectly
26
Q

A surgical patient has been transferred to the holding area. What nursing intervention(s) promote safe and effective nursing care? Select all that apply.

a) Maintain an aseptic environment.
b) Identify the patient using two identifiers.
c) Verify the surgical site and mark it appropriately.
d) Provide oral fluids to the patient.
e) Review the medical records.
f) Apply grounding devices to the patient.

A

b) Identify the patient using two identifiers., c) Verify the surgical site and mark it appropriately., e) Review the medical records.
Explanation:
Identifying the patient, verifying and marking the surgical site, and reviewing the medical records all promote safe and effective care while the patient is in the holding area. Maintaining an aseptic environment and applying grounding devices are part of the intraoperative phase. Oral fluids should not be provided while the patient is in the holding area.

How well did you know this?
1
Not at all
2
3
4
5
Perfectly
27
Q

The nurse is caring for a patient admitted to the ED with an uncomplicated nasal fracture. Nasal packing has been completed. Which of the following interventions should the nurse include in the patient’s care?

a) Apply pressure to the convex of the nose
b) Apply an ice pack
c) Restrict fluid intake
d) Position the patient in the side-lying position.

A

b) Apply an ice pack

Explanation:
Following a nasal fracture, the nurse applies ice and encourages the patient to keep the head elevated. The nurse instructs the patient to apply ice packs to the nose to decrease swelling. The packing inserted to stop the bleeding may be uncomfortable and unpleasant, and obstruction of the nasal passages by the packing forces the patient to breathe through the mouth. This, in turn, causes the oral mucous membranes to become dry. Mouth rinses help to moisten the mucous membranes and to reduce the odor and taste of dried blood in the oropharynx and nasopharynx. Applying direct pressure is not indicated in this situation.

How well did you know this?
1
Not at all
2
3
4
5
Perfectly
27
Q

The nurse is screening a patient prior to a magnetic resonance angiogram (MRA) of the heart. Which of the following actions should the nurse complete prior to the patient undergoing the procedure? Select all that apply.

a) Offer the patient a headset to listen to music during the procedure.
b) Remove the patient’s jewelry.
c) Sedate the patient prior to the procedure.
d) Position the patient on his/her stomach for the procedure.
e) Remove the patient’s Transderm Nitro patch.

A

a) Offer the patient a headset to listen to music during the procedure., b) Remove the patient’s jewelry., e) Remove the patient’s Transderm Nitro patch.

Explanation:
Transdermal patches that contain a heat-conducting aluminized layer (e.g., NicoDerm, Androderm, Transderm Nitro, Transderm Scop, Catapres-TTS) must be removed before MRA to prevent burning of the skin. A patient who is claustrophobic may need to receive a mild sedative before undergoing an MRA. During an MRA, the patient is positioned supine on a table that is placed into an enclosed imager or tube containing the magnetic field. Patients are instructed to remove any jewelry, watches, or other metal items (e.g., ECG leads). An intermittent clanking or thumping that can be annoying is generated by the magnetic coils, so the patient may be offered a headset to listen to music.

How well did you know this?
1
Not at all
2
3
4
5
Perfectly
28
Q

The nurse is administering medications on a medical surgical unit. A patient is ordered to receive 40 mg of oral Corgard (nadolol) for the treatment of hypertension. Prior to administering the medication, the nurse should complete which of the following?

a) Weighing the patient
b) Checking the patient’s serum K+ level
c) Checking the patient’s heart rate
d) Checking the patient’s urine output

A

c) Checking the patient’s heart rate
Explanation:
Corgard is a beta-blocker. A desired effect of this medication is to reduce the pulse rate in patients with tachycardia and an elevated blood pressure (BP). The nurse should check the patient’s heart rate (HR) prior to administering Corgard to ensure that the patient’s pulse rate is not below 60 (beats per minute (bpm). The other interventions are not indicated prior to administering a beta-blocker medication.

How well did you know this?
1
Not at all
2
3
4
5
Perfectly
28
Q

When a patient who has been diagnosed with angina pectoris complains that he is experiencing chest pain more frequently even at rest, the period of pain is longer, and it takes less stress for the pain to occur, the nurse recognizes that the patient is describing which type of angina?

a) Intractable
b) Variant
c) Unstable
d) Refractory

A

c) Unstable
Explanation:
Unstable angina is also called crescendo or preinfarction angina and indicates the need for a change in treatment. Intractable or refractory angina produces severe, incapacitating chest pain that does not respond to conventional treatment. Variant angina is described as pain at rest with reversible ST-segment elevation and is thought to be caused by coronary artery vasospasm. Intractable or refractory angina produces severe, incapacitating chest pain that does not respond to conventional treatment.

How well did you know this?
1
Not at all
2
3
4
5
Perfectly
28
Q

The nurse is irrigating a patient’s colostomy when the patient begins to complain of cramping. What is the appropriate action by the nurse?

a) Increase the rate of administration.
b) Discontinue the irrigation immediately.
c) Change irrigation fluid to normal saline.
d) Clamp the tubing and allow patient to rest.

A

d) Clamp the tubing and allow patient to rest.
Explanation:
The nurse should clamp the tubing and allow the patient to rest when the patient begins to complain of cramping during colostomy irrigation. Once the cramping has stopped, the nurse can resume the irrigation.

How well did you know this?
1
Not at all
2
3
4
5
Perfectly
29
Q

A patient is postoperative hour 8 following an appendectomy and is anxious stating, “Something is not right. My pain is worse than ever and my stomach is swollen.” Blood pressure is 88/50, pulse is 115, and respirations are 24 and labored. Abdomen is soft and distended. No obvious bleeding noted. What action by the nurse is most appropriate?

a) Inform the patient this is the normal progression following abdominal surgery.
b) Administer morphine per orders.
c) Ambulate the patient to reduce abdominal distention.
d) Notify the physician.

A

d) Notify the physician.
Explanation:
The physician should be notified of the findings. The patient may be hemorrhaging internally and may need to return to surgery. The patient may be in need of pain medication but morphine will lower the blood pressure further and may cause further complications. Ambulating the patient increases the risk of injury because the patient may experience orthostatic hypotension. What the patient is experiencing is not the normal progression following abdominal surgery.

How well did you know this?
1
Not at all
2
3
4
5
Perfectly
30
Q

A recently extubated postoperative patient starts to gag and make vomiting sounds. What action should the nurse do first?

a) Administer antiemetic.
b) Obtain suction equipment.
c) Turn patient on her side.
d) Provide emesis basin.

A

c) Turn patient on her side.
Explanation:
The nurse should turn the patient on her side to avoid aspiration. The nurse may need to obtain suction equipment, provide an emesis basin, or administer and antiemetic but the first priority is protecting the patient’s airway by preventing aspiration

How well did you know this?
1
Not at all
2
3
4
5
Perfectly
31
Q

The nurse is caring for a client newly diagnosed with secondary hypertension. Which of the following conditions contributes to the development of secondary hypertension?

a) Calcium deficit
b) Hepatic function
c) Renal disease
d) Acid-based imbalance

A

c) Renal disease
Explanation:
Secondary hypertension occurs when a cause for the high blood pressure can be identified. These causes include renal parenchymal disease, narrowing of the renal arteries, hyperaldosteronism (mineralocorticoid hypertension), pheochromocytoma, certain medications (e.g., prednisone, epoietin alfa [Epogen]), and coarctation of the aorta. High blood pressure can also occur with pregnancy; women who experience high blood pressure during pregnancy are at increased risk of ischemic heart disease, heart attacks, strokes, kidney disease, diabetes, and death from heart attack. Calcium deficiency or acid-based imbalance does not contribute to hypertension.

How well did you know this?
1
Not at all
2
3
4
5
Perfectly
31
Q

The nurse is caring for an adult pt with a normal BP. What should the nurse know would be the approiamate insensible water loss per day in this pt?

  1. 250 ml/da
  2. 600 ml/day
  3. 800ml/day
  4. 1000 ml/day
A

600 ml/day

How well did you know this?
1
Not at all
2
3
4
5
Perfectly
32
Q

The nurse is preparing to perform tracheostomy care on a patient with a newly inserted tracheostomy tube. Which of the following actions, if preformed by the nurse, indicates the need for further review of the procedure?

a) Places clean tracheostomy ties, and removes soiled ties after the new ties are in place
b) Puts on clean gloves; removes and discards the soiled dressing in a biohazard container
c) Cleans the wound and the plate with a sterile cotton tip moistened with hydrogen peroxide
d) Dries and reinserts the inner cannula or replaces it with a new disposable inner cannula

A

a) Places clean tracheostomy ties, and removes soiled ties after the new ties are in place

Explanation:
For a new tracheostomy, two people should assist with tie changes. The other actions, if performed by the nurse during tracheostomy care, are correct.

How well did you know this?
1
Not at all
2
3
4
5
Perfectly
32
Q

The nurse auscultates the PMI (point of maximal impulse) at which of the following anatomic locations?

a) 1 inch to the left of the xiphoid process
b) Left midclavicular line, fifth intercostal space
c) 2 inches to the left of the lower end of the sternum
d) Midsternum

A

b) Left midclavicular line, fifth intercostal space

Explanation:
The left ventricle is responsible for the apical impulse or the point of maximum impulse, which is normally palpable in the left midclavicular line of the chest wall at the fifth intercostal space. The right ventricle lies anteriorly, just beneath the sternum. Use of inches to identify the location of the PMI is inappropriate based on variations in human anatomy. Auscultation below and to the left of the xiphoid process will detect gastrointestinal sounds, but not the PMI.

How well did you know this?
1
Not at all
2
3
4
5
Perfectly
33
Q

Which of the following nursing roles may have prescriptive authority in their practice? (Select all that apply.)

A. Critical care nurse
B. Nurse practitioner
C. Certified clinical nurse specialist
D. Charge nurse

A

B,C

How well did you know this?
1
Not at all
2
3
4
5
Perfectly
34
Q

The circulating nurse is unsure if proper technique was followed when placing an object in the sterile field during a surgical procedure. What is the best action by the nurse?

a) Remove the entire sterile field from use.
b) Remove the item from the sterile field.
c) Ask another nurse to review the technique used.
d) Mark the patient’s chart for future review of infections.

A

a) Remove the entire sterile field from use.
Explanation:
If there is any doubt about the maintenance of sterility, the field should be considered not sterile. Because the object in question was placed in the sterile field, the sterile field must be removed from use. Removing the individual item is not appropriate, as the field was potentially contaminated. Reviewing the patient’s chart at a later date does not decrease the chance of infection. Although another nurse could observe the technique used to put objects in a sterile field, it does not solve the immediate concern.

How well did you know this?
1
Not at all
2
3
4
5
Perfectly
35
Q

The nurse is teaching a patient diagnosed with hypertension about the DASH diet. How many servings of meat, fish, and poultry should a patient consume per day?

a) 4 or 5
b) 7 or 8
c) 2 or 3
d) 2 or fewer

A

d) 2 or fewer
Explanation:
Two or fewer servings of meat, fish, and poultry are recommended in the DASH diet

How well did you know this?
1
Not at all
2
3
4
5
Perfectly
35
Q

A nurse is teaching a patient about valve replacement surgery. Which statement by the patient indicates an understanding of the benefit of an autograft replacement valve?

a) “The valve is from a tissue donor, and I will not need to take any blood thinning drugs with I am discharged.”
b) “The valve is made from my own heart valve, and I will not need to take any blood thinning drugs when I am discharged.”
c) “The valve is made from a pig tissue, and I will not need to take any blood-thinning drugs when I am discharged.”
d) “The valve is mechanical, and it will not deteriorate or need replacing.”

A

b) “The valve is made from my own heart valve, and I will not need to take any blood thinning drugs when I am discharged.”
Explanation:
Autografts (i.e., autologous valves) are obtained by excising the patient’s own pulmonic valve and a portion of the pulmonary artery for use as the aortic valve. Anticoagulation is unnecessary because the valve is the patient’s own tissue and is not thrombogenic. The autograft is an alternative for children (it may grow as the child grows), women of childbearing age, young adults, patients with a history of peptic ulcer disease, and people who cannot tolerate anticoagulation. Aortic valve autografts have remained viable for more than 20 years.

How well did you know this?
1
Not at all
2
3
4
5
Perfectly
36
Q

A nurse assesses arterial blood gas results for a patient in acute respiratory failure (ARF). Which of the following results are consistent with this disorder?

a) pH 7.35, PaCO2 48 mm Hg
b) pH 7.46, PaO2 80 mm Hg
c) pH 7.36, PaCO2 32 mm Hg
d) pH 7.28, PaO2 50 mm Hg

A

d) pH 7.28, PaO2 50 mm Hg

Explanation:
ARF is defined as a decrease in the arterial oxygen tension (PaO2) to less than 50 mm Hg (hypoxemia) and an increase in arterial carbon dioxide tension (PaCO2) to greater than 50 mm Hg (hypercapnia), with an arterial pH of less than 7.35.

How well did you know this?
1
Not at all
2
3
4
5
Perfectly
37
Q

Decreased pulse pressure reflects which of the following?

a) Elevated stroke volume
b) Reduced distensibility of the arteries
c) Tachycardia
d) Reduced stroke volume

A

d) Reduced stroke volume
Explanation:
Decreased pulse pressure reflects reduced stroke volume and ejection velocity or obstruction to blood flow during systole. Increased pulse pressure would indicate reduced distensibility of the arteries, along with bradycardia.

How well did you know this?
1
Not at all
2
3
4
5
Perfectly
39
Q

A 78-year-old woman is undergoing right hip surgery to repair a hip fracture. What nursing action is appropriate during the intraoperative phase?

a) Withhold pain medication due to decreased renal functioning.
b) Appropriately position the patient using adequate padding and support.
c) Discuss the need for higher doses of anesthetic agents with the anesthesiologist.
d) Maintain an operating room temperature of 18°C to prevent hypothermia.

A

b) Appropriately position the patient using adequate padding and support.
Explanation:
Adequate padding and support should be used to prevent positioning injuries. The older adult is has lower bone mass, which increases the risk of intraoperative positioning injuries. Pain medication can still be used, just in smaller doses, due to decreased liver and kidney functioning. For the same reason as pain medication, lower doses of anesthetic agents are used with the older adult. The operating room is usually maintained from 20°C to 24°C; 18°C is lower than the recommended temperature and can promote hypothermia in the older adult who already has impaired thermoregulation and is prone to hypothermia.

How well did you know this?
1
Not at all
2
3
4
5
Perfectly
40
Q

A postanesthesia care unit (PACU) nurse is caring for a patient with the following assessment data: pale, cool, moist skin; thready pulse of 122; blood pressure 78/60; urine output of 25 mL/h; temperature 99.2°F. What interventions by the nurse are appropriate? Select all that apply.

a) Apply oxygen per orders.
b) Frequently monitor neurological status.
c) Maintain a patent airway.
d) Raise the head of the bed 30 degrees.
e) Apply a warming blanket.
f) Administer blood products per orders.

A

a) Apply oxygen per orders., b) Frequently monitor neurological status., c) Maintain a patent airway., f) Administer blood products per orders.
Explanation:
The patient is demonstrating signs and symptoms of shock. The patient in shock may lose the ability to protect his or her airway. Frequently neurological assessment can provide information related to decrease oxygen to the brain. Administering the blood products may reverse the signs and symptoms of shock. There is an increased need for oxygen when in shock, so it is appropriate to apply oxygen. The head of the bed should not be elevated. The patient should be lying flat or in the Trendelenburg position. Applying a warming blanket when the patient is not hypothermic may cause vasodilation, which could further decrease blood pressure and perfusion to vital organs.

How well did you know this?
1
Not at all
2
3
4
5
Perfectly
41
Q

The purpose of postoperative leg exercises is to:

  1. Maintain muscle tone
  2. Promote venous return
  3. Assess range of motion
  4. Exercise fatigued muscles
A

Promote venous return

  1. Promotes normal venous return and circulatory
    blood flow
How well did you know this?
1
Not at all
2
3
4
5
Perfectly
42
Q

Age-related changes associated with the cardiac system include which of the following? Select all that apply.

a) Increased size of the left atrium
b) Endocardial fibrosis
c) Myocardial thinning
d) Increase in the number of SA node cells

A

a) Increased size of the left atrium, b) Endocardial fibrosis
Explanation:
Age-related changes associated with the cardiac system include endocardial fibrosis, increased size of the left atrium, decreased number of SA node cells, and myocardial thickening.

How well did you know this?
1
Not at all
2
3
4
5
Perfectly
42
Q

The nurse is caring for a patient prescribed Bumex (bumetanide) for the treatment of stage 2 hypertension. Which of the following indicates the patient is experiencing an adverse effect of the medication?

a) Electrocardiogram (EGG) tracing demonstrating peaked T waves
b) Blood glucose value of 160 mg/dL
c) Serum potassium value of 3.0 mEq/L
d) Urine output of 90 cc/mL 1 hour after medication administration

A

c) Serum potassium value of 3.0 mEq/L
Explanation:
Bumex is a loop diuretic that can cause fluid and electrolyte imbalances. Patients taking these medications may experience a low serum potassium level. ECG changes associated with an elevated serum potassium levels include peaked T waves. Diuresis is a desired effect postadministration of Bumex. The serum glucose level is elevated and requires intervention; however, this elevation is not associated with the administration of Bumex.

How well did you know this?
1
Not at all
2
3
4
5
Perfectly
42
Q

The nurse understands it is important to promote adequate tissue perfusion following cardiac surgery. Which of the following measures should the nurse complete to prevent deep venous thrombosis (DVT) and possible pulmonary embolism (PE) development? Select all that apply.

a) Encourage the crossing of the legs.
b) Avoid elevating the knees on the bed.
c) Initiate passive exercises.
d) Place pillows in the popliteal space.
e) Apply antiembolism stockings.

A

b) Avoid elevating the knees on the bed., c) Initiate passive exercises., e) Apply antiembolism stockings.
Explanation:
Preventative measures utilized to prevent venous stasis include: Application of sequential pneumatic compression wraps or antiembolic stockings; discouraging leg crossing; avoiding elevating the knees on the bed; omitting pillows in the popliteal space; beginning passive exercises followed by active exercises to promote circulation and prevent venous stasis.

How well did you know this?
1
Not at all
2
3
4
5
Perfectly
42
Q
A
How well did you know this?
1
Not at all
2
3
4
5
Perfectly
43
Q

The nurses on an acute care medical floor notice an increase in pressure ulcer formation in their patients. A nurse consultant decides to compare two types of treatment. The first is the procedure currently used to assess for pressure ulcer risk. The second uses a new assessment instrument to identify at-risk patients. Given this information, the nurse consultant exemplifies which career?

A. Clinical nurse specialist

B. Nurse administrator

C. Nurse educator

D. Nurse researcher

A

D. Nurse researcher

The nurse researcher investigates problems to improve nursing care and to further define and expand the scope of nursing practice. He or she often works in an academic setting, hospital, or independent professional or community service agency.

How well did you know this?
1
Not at all
2
3
4
5
Perfectly
43
Q

The nurse is caring for a patient in the ED who has a B-type natriuretic peptide (BNP) level of 115 pg/mL. The nurse understands that this finding is most suggestive of which of the following?

a) Heart failure
b) Myocardial infarction
c) Pulmonary edema
d) Ventricular hypertrophy

A

a) Heart failure
Explanation:
A BNP level greater than 100 pg/mL is suggestive of HF. Because this serum laboratory test can be quickly obtained, BNP levels are useful for prompt diagnosis of HF in settings such as the ED. Elevations in BNP can occur from a number of other conditions such as pulmonary embolus, myocardial infarction (MI), and ventricular hypertrophy. Therefore, the clinician correlates BNP levels with abnormal physical assessment findings and other diagnostic tests before making a definitive diagnosis of HF.

How well did you know this?
1
Not at all
2
3
4
5
Perfectly
44
Q

The nurse is caring for a patient in the ICU with a nasotracheal tube. Because of the tube placement, the nurse understands that the patient is at risk for developing which of the following?

a) Severe epistaxis
b) Orbital cellulitis
c) Sinus infection
d) Subperiosteal abscess

A

c) Sinus infection

Explanation:
Patients with nasotracheal and nasogastric tubes in place are at risk for development of sinus infections. Thus, accurate assessment of patients with these tubes is critical. Removal of the nasotracheal or nasogastric tube as soon as the patient’s condition permits allows the sinuses to drain, possibly avoiding septic complications. Severe epistaxis is not a complication of nasotracheal placement. Subperiosteal abscess and orbital cellulitis are complications of chronic rhinosinusitis.

How well did you know this?
1
Not at all
2
3
4
5
Perfectly
44
Q

During a routine assessment of a patient, the nurse notes that the nails are concave shaped. Which of the following is indicated by this finding?

a) Fungal infection
b) Long-standing cardiopulmonary disease
c) Poor circulation
d) Iron deficiency anemia

A

d) Iron deficiency anemia

Explanation:
The concave shape of the nails, referred to as spooning, is a sign of iron deficiency anemia. Clubbing of the nails, at greater than a 160-degree angle, suggests long-standing cardiopulmonary disease. Nails thicken when there is a fungal infection and also poor circulation.

How well did you know this?
1
Not at all
2
3
4
5
Perfectly
45
Q

Which of the following terms refers to a graft derived from one part of a patient’s body and used on another part of that same patient’s body?

a) Autograft
b) Allograft
c) Homograft
d) Heterograft

A

a) Autograft

Explanation:
Autografts of full-thickness and pedicle flaps are commonly used for reconstructive surgery, months or years after the initial injury. An allograft is a graft transferred from one human (living or cadaveric) to another human. A homograft is a graft transferred from one human (living or cadaveric) to another human. A heterograft is a graft obtained from an animal of a species other than that of the recipient.

How well did you know this?
1
Not at all
2
3
4
5
Perfectly
46
Q

A 77-year-old woman presents to the local community center for a blood pressure screening. The women’s blood pressure is recorded as 180/90 mm Hg. The woman has a history of hypertension, but she currently is not taking her medications. Which of the following questions is most appropriate for the nurse to ask the patient first?

a) “Why is it that you are not taking your medications?”
b) “Are you able to get to your pharmacy to pick up your medications?”
c) “Are you having trouble paying for your medication?”
d) “What medications are you prescribed?”

A

a) “Why is it that you are not taking your medications?”
Explanation:
It is important for the nurse to first ascertain if the reason why the patient is not taking her medications. Adherence to the therapeutic program may be more difficult for older adults. The medication regimen can be difficult to remember, and the expense can be a challenge. Monotherapy (treatment with a single agent), if appropriate, may simplify the medication regimen and make it less expensive. The other questions are appropriate, but the priority is to determine why the medication regimen is not being followed.

How well did you know this?
1
Not at all
2
3
4
5
Perfectly
47
Q

The nurse is performing chest auscultation for a pt with asthma. How does the nurse describe the high-pitched, sibilant, muscial sounds that are heard?

  1. Rales
  2. Crackles
  3. Wheezes
  4. Rhonchi
A

Wheezes

How well did you know this?
1
Not at all
2
3
4
5
Perfectly
48
Q

A patient is being mechanically ventilated in the ICU. The ventilator alarms begin to sound. The nurse should complete which of the following actions first?

a) Manually ventilate the patient.
b) Notify the respiratory therapist.
c) Troubleshoot to identify the malfunction.
d) Reposition the endotracheal (ET) tube.

A

c) Troubleshoot to identify the malfunction.

Explanation:
The nurse should first immediately attempt to identify and correct the problem and, if the problem cannot be identified and/or corrected, the patient must be manually ventilated with an Ambu bag. The respiratory therapist may be notified, but this is not the first action by the nurse. The nurse should not reposition the ET tube as a first response to an alarm.

How well did you know this?
1
Not at all
2
3
4
5
Perfectly
49
Q

The nurse is completing a physical assessment of a patient’s trachea. The nurse inspects and palpates the trachea for which of the following?

a) Color of the mucous membranes
b) Deviation from the midline
c) Evidence of exudate
d) Evidence of muscle weakness

A

b) Deviation from the midline

Explanation:

During the physical examination, the nurse must inspect and gently palpate the trachea to assess for placement and deviation from the midline. The trachea is normally in the midline as it enters the thoracic inlet behind the sternum, but it may be deviated by masses in the neck or mediastinum. Pulmonary disorders, such as a pneumothorax or pleural effusion, may also displace the trachea. The nurse examines the posterior pharynx and tonsils with a tongue blade and light, and notes any evidence of swelling, inflammation, or exudate, as well as changes in color of the mucous membranes. The nurse also examines the anterior, posterior, and lateral chest walls for any evidence of muscle weakness.

How well did you know this?
1
Not at all
2
3
4
5
Perfectly
49
Q

The nurse notes red, papular, round lesions on the patient’s back that blanch with light pressure. The appropriate action by the nurse is which of the following?

a) Notify the physician.
b) Document the finding
c) Apply barrier cream.
d) Turn and reposition patient.

A

b) Document the finding

Explanation:
Lesions that are red, papular, and round that are located on a patient’s trunk and blanch with pressure are typically cherry angiomas. As this lesion has no clinical significance, the appropriate action by the nurse is to document the findings.

How well did you know this?
1
Not at all
2
3
4
5
Perfectly
51
Q

The nurse is caring for a patient complaining of chest discomfort. The patient’s admitting diagnosis is left lower lobe pneumonia. Which of the following strategies will the nurse instruct the patient to use to help alleviate the discomfort?

a) Complete deep breathing exercises when chest discomfort occurs
b) Request narcotic medication when pain is experienced
c) Lying on the right side
d) Assume a left side-lying position while in bed

A

c) Lying on the right side

Explanation:
Pleuritic pain from irritation of the parietal pleura is sharp and seems to “catch” on inspiration; patients often describe it as being “like the stabbing of a knife.” Patients are more comfortable when they lay on the affected side because this splints the chest wall, limits expansion and contraction of the lung, and reduces the friction between the injured or diseased pleurae on that side. Pain associated with cough may be reduced manually by splinting the rib cage. The nurse would instruct the patient to lie on the left side, not the right, to decrease the pain. While pain medication may be administered, nonpharmacological therapies and non-narcotic interventions should be implemented first. Deep breathing exercises would not aid in decreasing the pain, but rather slowing the patient’s breathing and expanding the lungs.

How well did you know this?
1
Not at all
2
3
4
5
Perfectly
51
Q

The nurse is caring for a client with allergic rhinitis. The patient asks the nurse about measures to help decrease allergic symptoms. The best response by the nurse is which of the following?

a) “You should try to reduce exposure to irritants and allergens.”
b) “Be sure to receive your influenza vaccination each year.”
c) “You need to see your ear, nose, and throat specialist monthly.”
d) “Take over-the-counter (OTC) nasal congestions when you experience symptoms.”

A

a) “You should try to reduce exposure to irritants and allergens.”

Explanation:
The nurse instructs the patient with allergic rhinitis to avoid or reduce exposure to allergens and irritants, such as dusts, molds, animals, fumes, odors, powders, sprays, and tobacco smoke. Receiving an influenza vaccination each year is recommended for patients with infectious rhinitis. To prevent possible drug interactions, the patient is cautioned to read drug labels before taking any OTC medication. Patients with nasal septal deformities or nasal polyps may be referred to an ear, nose, and throat specialist.

How well did you know this?
1
Not at all
2
3
4
5
Perfectly
51
Q

A patient with infective endocarditis (IE) and a fever is admitted to the intensive care unit (ICU). Which of these physician orders should the nurse implement first?

a) Obtain a transesophageal echocardiogram.
b) Order blood cultures drawn from two sites.
c) Give acetaminophen (Tylenol) pro re nata (PRN) for fever higher than 100.3 degrees.
d) Administer ceftriaxone (Rocephin) 1 g IVPB q 12 hours.

A

b) Order blood cultures drawn from two sites.
Explanation:
Blood cultures (with each set including one aerobic and one anaerobic culture) drawn from different venipuncture sites over a 24-hour period (each set at least 12 hours apart), or every 30 minutes if the patient’s condition is unstable, should be obtained before administration of any antimicrobial agents. It is essential to obtain blood cultures before initiating antibiotic therapy to obtain accurate sensitivity results.

How well did you know this?
1
Not at all
2
3
4
5
Perfectly
52
Q

A patient is brought to the operating room for an elective surgery. What is the priority action by the circulating nurse?

a) Acquire ordered blood products.
b) Obtain a sponge and syringe count.
c) Verify consent.
d) Document start of surgery.

A

c) Verify consent.
Explanation:
Without consent, surgery cannot be performed. Documentation of the start of surgery can only happen once the surgery has started. Blood products must be administered within an allotted time frame and therefore should not be acquired unless needed. The sponge and syringe count is a safety issue that should be completed before surgery and while the wound is being sutured, but the patient has not consented, the surgery should not take place.

How well did you know this?
1
Not at all
2
3
4
5
Perfectly
53
Q

The nurse is reviewing discharge instructions with a patient who underwent a left groin cardiac catheterization 8 hours ago. Which of the following instructions should the nurse include?

a) “Do not bend at the waist, strain, or lift heavy objects for the next 24 hours.”
b) “You can take a tub bath or a shower when you get home.”
c) “If any discharge occurs at the puncture site, call 911 immediately.”
d) “Contact your primary care provider if you develop a temperature above 102°F.”

A

a) “Do not bend at the waist, strain, or lift heavy objects for the next 24 hours.”

Explanation:
The nurse should instruct the patient to complete the following: If the artery of the groin was used, for the next 24 hours, do not bend at the waist, strain, or lift heavy objects; the primary provider should be contacted if any of the following occur: swelling, new bruising or pain from your procedure puncture site, temperature of 101°F or more. If bleeding occurs, lie down (groin approach) and apply firm pressure to the puncture site for 10 minutes. Notify the primary provider as soon as possible and follow instructions. If there is a large amount of bleeding, call 911. The patient should not drive to the hospital.

How well did you know this?
1
Not at all
2
3
4
5
Perfectly
55
Q

Which of the following findings indicates that hypertension is progressing to target organ damage?

a) Urine output of 60 cc/mL over 2 hours
b) Chest x-ray showing pneumonia
c) Blood urea nitrogen (BUN) level of 12 mg/dL
d) Retinal blood vessel damage

A

d) Retinal blood vessel damage
Explanation:
Symptoms suggesting that hypertension is progressing to the extent that target organ damage is occurring must be detected early so that appropriate treatment can be initiated. All body systems must be assessed to detect any evidence of vascular damage. An eye examination with an ophthalmoscope is important because retinal blood vessel damage indicates similar damage elsewhere in the vascular system. The patient is questioned about blurred vision, spots in front of the eyes, and diminished visual acuity. The heart, nervous system, and kidneys are also carefully assessed. A BUN level and 60 cc/mL over 2 hours are normal findings. The presence of pneumonia does not indicate target organ damage.

How well did you know this?
1
Not at all
2
3
4
5
Perfectly
55
Q

A postoperative patient begins coughing forcefully when eating gelatin. The nurse notices an evisceration of the intestines. What should the nurse do first?

a) Notify the surgeon.
b) Cover the intestines with sterile, moist dressings.
c) Place the patient in low Fowler’s position.
d) Document the event.

A

c) Place the patient in low Fowler’s position.
Explanation:
Placing the patient in low Fowler’s position decreases further protrusion of the intestines. The nurse should cover the intestines with a sterile, moist dressing; notify the surgeon and document the event; but first and foremost the nurse should minimize further protrusion of the intestines.

How well did you know this?
1
Not at all
2
3
4
5
Perfectly
57
Q

A patient is undergoing a perineal surgical procedure. Which of the following actions by the nurse is appropriate?

a) Place the patient in Sims’ position.
b) Place the patient in a dorsal recumbent position.
c) Place the patient in lithotomy position.
d) Place the patient in the Trendelenburg position.

A

c) Place the patient in lithotomy position.
Explanation:
The lithotomy position is used for nearly all perineal, rectal, and vaginal surgeries. The Trendelenburg position is usually used for surgery on the lower abdomen and pelvis. Sims’ or lateral position is used for renal surgery. The dorsal recumbent position is the usual position for surgical procedures.

How well did you know this?
1
Not at all
2
3
4
5
Perfectly
57
Q

The nurse is performing an assessment of an immobilized client. Which assessment causes the nurse to take action?

  1. Heart rate 86
  2. Reddened area on sacrum
  3. Nonproductive cough
  4. Urine output of 50 mL/hour
A

2.Reddened area on sacrum

Rationale: The reddened area of the skin can lead to skin breakdown. The other options are within normal limits.

How well did you know this?
1
Not at all
2
3
4
5
Perfectly
58
Q

The nurse understands that an overall goal of hypertension management includes which of the following?

a) There are no complaints of sexual dysfunction.
b) There is no indication of target organ damage.
c) There is no complaint of postural hypotension.
d) The patient maintains a normal blood pressure reading.

A

b) There is no indication of target organ damage.
Explanation:
Prolonged blood pressure elevation gradually damages blood vessels throughout the body, particularly in target organs such as the heart, kidneys, brain, and eyes. The overall goal of management is that the patient does not experience target organ damage. The desired effects of antihypertensives are to maintain a normal BP. Postural hypotension and sexual dysfunction are side effects of certain antihypertension medications.

How well did you know this?
1
Not at all
2
3
4
5
Perfectly
59
Q

Of the following oxygen administration devices, which has the advantage of providing high oxygen concentration?

a) Nonrebreather mask
b) Venturi mask
c) Catheter
d) Face tent

A

a) Nonrebreather mask

Explanation:
The nonrebreather mask provides high oxygen concentration, but it is usually poorly fitting. However, if the nonrebreathing mask fits the patient snugly and both side exhalation ports have one-way valves, it is possible for the patient to receive 100% oxygen, making the nonrebreathing mask a high-flow oxygen system. The Venturi mask provides low levels of supplemental oxygen. The catheter is an inexpensive device that provides a variable fraction of inspired oxygen and may cause gastric distention. A face tent provides a fairly accurate fraction of inspired oxygen but is bulky and uncomfortable. It would not be the device of choice to provide high oxygen concentration.

How well did you know this?
1
Not at all
2
3
4
5
Perfectly
60
Q

The nurse is assessing the integumentary system of a female patient with Cushing’s syndrome. The nurse anticipates which of the following findings?

a) Hyperpigmentation
b) Jaundice
c) Hirsutism
d) Alopecia

A

c) Hirsutism

Explanation:
The nurse anticipates finding hirsutism, or excessive hair growth as Cushing’s syndrome causes hirsutism, especially in women. Alopecia, jaundice, and hyperpigmentation are not typical assessment findings in patients with Cushing’s syndrome.

How well did you know this?
1
Not at all
2
3
4
5
Perfectly
61
Q

A patient arrives at the ED with an exacerbation of left-sided heart failure and complains of shortness of breath. Which of the following is the priority nursing intervention?

a) Administer angiotensin-converting enzyme inhibitors
b) Administer angiotensin II receptor blockers
c) Assess oxygen saturation level
d) Administer diuretics

A

c) Assess oxygen saturation level
Explanation:
Assessment is priority to determine severity of the exacerbation. It is important to assess the oxygen saturation level of a heart failure patient, as below normal oxygen saturation level can be life-threatening. Treatment options vary according to the severity of the patient’s condition and may include supplemental oxygen, oral and IV medications, major lifestyle changes, implantation of cardiac devices, and surgical approaches. The overall goal of treatment of heart failure is to relieve patient symptoms and reduce the workload on the heart by reducing afterload and preload.

How well did you know this?
1
Not at all
2
3
4
5
Perfectly
62
Q

A patient with a myocardial infarction develops acute mitral valve regurgitation. The nurse knows to assess for which of the following manifestations that would indicate that the patient is developing pulmonary congestion?

a) Tachycardia
b) Hypertension
c) Shortness of breath
d) A loud, blowing murmur

A

c) Shortness of breath
Explanation:
Chronic mitral regurgitation is often asymptomatic, but acute mitral regurgitation (e.g., resulting from a myocardial infarction) usually manifests as severe congestive heart failure. Dyspnea, fatigue and weakness are the most common symptoms. Palpitations, shortness of breath on exertion, and cough from pulmonary congestion also occur. A loud, blowing murmur often is heard throughout ventricular systole at the heart’s apex. Hypertension may develop when reduced cardiac output triggers the renin-angiotensin-aldosterone cycle. Tachycardia is a compensatory mechanism when stroke volume decreases.

How well did you know this?
1
Not at all
2
3
4
5
Perfectly
62
Q

The PACU nurse notices that the patient is
shivering. This is most commonly caused by:
1. Cold irrigations used during surgery
2. Side effects of certain anesthetic agents
3. Malignant hypothermia, a serious condition
4. The use of a reflective blanket on the operating
room table

A

Side effects of certain anesthetic agents

Not always a sign of hypothermia but rather a
side effect of certain anesthetic agents

How well did you know this?
1
Not at all
2
3
4
5
Perfectly
63
Q

The nurse is instructing the pt in how to apply a corticosteroid cream to lesions on the arm. What intervention can the nurse instruct the pt to do to increase the absorpiton of the medication?

  1. Apply an occlusive dressing over the site after application
  2. Make sure the skin is slightly hydrated so that the medication can absorb through the skin cracks
  3. Apply a thick layer of cream over the lesions so that if some rubs off, there is more to absorb
  4. Apply the medication every 2 hours
A

Apply an occulsive dressing over the site after application

How well did you know this?
1
Not at all
2
3
4
5
Perfectly
64
Q

A patient presents to the emergency room complaining of chest pain. The patient’s orders include the following elements. Which order should the nurse complete first?

a) Troponin level
b) Oxygen 2 liters nasal cannula
c) 12-lead ECG
d) Aspirin 325 mg orally

A

c) 12-lead ECG
Explanation:
The nurse should complete the 12-lead ECG first. The priority is to determine if the patient is suffering an acute MI and implement appropriate interventions as quickly as possible. The other orders should be completed after the ECG.

How well did you know this?
1
Not at all
2
3
4
5
Perfectly
65
Q

When the nurse is assessing the older adult patient, what gerontologic changes in the respiratory system should he nurse be aware of?

  1. Decreased alveolar duct diameter
  2. Increased presence of muus
  3. Decreased gag reflex
  4. Increased prescence of collagen in alveolar walls
  5. Decreased presence of mucus
A

Decreased gag reflex, increased presence of collagen in alveolar walls, decreased presence of mucus

How well did you know this?
1
Not at all
2
3
4
5
Perfectly
66
Q

The nurse is assessing a pt with TEN. What assessment data would indicate that the pt may be progressing to keratoconjunctivitis (Select all that apply)

  1. Skin peeling on eyelids
  2. Pruritus of the eyes
  3. Burning of the eyes
  4. Dryness of the eyes
  5. Blurred optics discs
A

pruritus of the eyes, burning of the eyes, dryness of the eyes

How well did you know this?
1
Not at all
2
3
4
5
Perfectly
67
Q

The nurse is providing teaching to a patient with acne who is using isotretinoin (Accutane) therapy. Which of the following statements should the nurse make?

a) Contraceptives are not needed during treatment.
b) It is teratogenic in humans.
c) The side effects are irreversible.
d) The patient should take vitamin A supplements.

A

b) It is teratogenic in humans.

Explanation:
Accutane is teratogenic in humans, meaning that it can have an adverse effect on a fetus, causing central nervous system and cardiovascular defects, and structural abnormalities of the face. Contraceptives are needed during treatment. The patient should not take vitamin A supplements while taking this drug. Side effects are reversible with the withdrawal of the medication.

How well did you know this?
1
Not at all
2
3
4
5
Perfectly
69
Q

a patient comes into the emergency room complaining about chest pain that gets worse when taking deep breaths and lying down. After ruling out a myocardial infarction, a nurse would assess for which of the following diagnoses?

a) Rheumatic fever
b) Pericarditis
c) Cardiomyopathy
d) Mitral valve stenosis

A

b) Pericarditis
Explanation:
The primary symptom of pericarditis is pain, which is assessed by evaluating the patient in various positions. The nurse tries to identify whether pain is influenced by respiratory movements while holding an inhaled breath or holding an exhaled breath; by flexion, extension, or rotation of the spine, including the neck; by movements of shoulders and arms; by coughing; or by swallowing. Recognizing events that precipitate or intensify pain may help establish a diagnosis and differentiate pain of pericarditis from pain of myocardial infarction.

How well did you know this?
1
Not at all
2
3
4
5
Perfectly
69
Q

A patient asks about the purpose of withholding food and fluid before surgery. Which response by the nurse is appropriate?

a) It prevents aspiration and respiratory complications.
b) It decreases the risk of elevated blood sugars and slow wound healing.
c) It decreases urine output so that a catheter would not be needed.
d) It prevents overhydration and hypertension.

A

a) It prevents aspiration and respiratory complications.
Explanation:
The major purpose of withholding food and fluid before surgery is to prevent aspiration, which can lead to respiratory complications. Preventing overhydration, decreasing urine output, and decreasing blood sugar levels are not major purposes of withholding food and fluid before surgery.

How well did you know this?
1
Not at all
2
3
4
5
Perfectly
70
Q

Which of the following comfort techniques does a nurse teach to a patient with pleurisy to assist with splinting the chest wall?

a) Use a prescribed analgesic
b) Elevate the head of the bed
c) Use a heat application
d) Turn onto the affected side

A

d) Turn onto the affected side

Explanation:
The nurse teaches the patient to splint the chest wall by turning onto the affected side in order to reduce the stretching of the pleurae and decrease pain.

How well did you know this?
1
Not at all
2
3
4
5
Perfectly
71
Q

Which of the following is a potential complication of a low pressure in the endotracheal tube (ET) cuff?

a) Pressure necrosis
b) Tracheal bleeding
c) Aspiration pneumonia
d) Tracheal ischemia

A

c) Aspiration pneumonia

Explanation:
Low pressure in the cuff can increase the risk for aspiration pneumonia. High cuff pressure can cause tracheal bleeding, ischemia, and pressure necrosis.

How well did you know this?
1
Not at all
2
3
4
5
Perfectly
72
Q

The nurse is caring for a patient diagnosed with unstable angina receiving IV heparin. The patient is placed on bleeding precautions. Bleeding precautions include which of the following measures?

a) Avoiding subcutaneous (SQ) injections
b) Avoiding the use of nail clippers
c) Using an electric toothbrush
d) Avoiding continuous BP monitoring

A

d) Avoiding continuous BP monitoring
Explanation:
The patient receiving heparin is placed on bleeding precautions, which can include: applying pressure to the site of any needle punctures for a longer time than usual, avoiding intramuscular injections, avoiding tissue injury and bruising from trauma or constrictive devices (e.g. continuous use of an automatic BP cuff). SQ injections are permitted; a soft toothbrush should be used, and the patient may use nail clippers, but with caution.

How well did you know this?
1
Not at all
2
3
4
5
Perfectly
73
Q

Which action will a public health nurse include when planning ways to decrease the incidence of rheumatic fever in the community?

a) Teach individuals of the community to seek medical treatment for streptococcal pharyngitis.
b) Provide prophylactic antibiotics to individuals with a family history of rheumatic fever.
c) Educate individuals of the community about the importance of monitoring temperature when infections occur.
d) Encourage susceptible groups in the community to receive immunizations with streptococcal vaccine.

A

a) Teach individuals of the community to seek medical treatment for streptococcal pharyngitis.
Explanation:
Prevention of acute rheumatic fever is dependent upon effective antibiotic treatment of streptococcal pharyngitis. Family history is not a risk factor for rheumatic fever. There is no immunization that is effective in decreasing the incidence of rheumatic fever. Education about monitoring temperature will not decrease the incidence of rheumatic fever

How well did you know this?
1
Not at all
2
3
4
5
Perfectly
75
Q

A pt is complaining of severe itching that intensifies at night. The nurse decides to assess the skin using a magnifying glass and penlight to look for the “itch mite”. What skin condition does the nurse anticipate finding?

  1. Contact dermatitis
  2. Pediculosis
  3. Scabies
  4. Tinea corporis
A

Scabies

How well did you know this?
1
Not at all
2
3
4
5
Perfectly
76
Q

Why is a client with fever often predisposed to pressure ulcers?

  1. Pain perception is diminished.
  2. Medications given to relieve fever cause edema.
  3. The client may be too weak to change position.
  4. Increased metabolism causes increased oxygen needs that cannot be met.
A
  1. Increased metabolism causes increased oxygen needs that cannot be met; Increased metabolism causes increased oxygen needs that cannot be met; therefore, a client with a fever is predisposed to pressure ulcers. Answers 1 and 2 are false statements. Answer 3 may be a cause of pressure ulcers and may occur in clients with fever, but it is not directly related.
How well did you know this?
1
Not at all
2
3
4
5
Perfectly
77
Q

Black wounds are treated with debridement. Which type of debridement is most selective and least damaging?

  1. Debridement with scissors
  2. Debridement with wet to dry dressings
  3. Mechanical debridement
  4. Chemical debridement
A
  1. Chemical debridement; Chemical debridement is either done with enzyme agents or autolytic agents. Answer 1 is a type of sharp debridement. Answers 2 and 3 are mechanical and less precise than chemical.
How well did you know this?
1
Not at all
2
3
4
5
Perfectly
78
Q

The nurse is assessing a patient who, following an extensive surgical procedure, is at risk for developing acute respiratory distress syndrome (ARDS). The nurse assesses for which early, most common sign of ARDS?

a) Bilateral wheezing
b) Rapid onset of severe dyspnea
c) Inspiratory crackles
d) Cyanosis

A

b) Rapid onset of severe dyspnea

Explanation:
The acute phase of ARDS is marked by a rapid onset of severe dyspnea that usually occurs less than 72 hours after the precipitating event

How well did you know this?
1
Not at all
2
3
4
5
Perfectly
79
Q

The nurse is teaching a patient about the correct use of topical concentrated corticosteroids. The nurse includes which of the following? Select all that apply.

a) Avoid prolonged use.
b) Hypertrichosis is normal.
c) Apply to face.
d) Apply to intertriginous areas.

A

a) Avoid prolonged use., c) Apply to face.

Explanation:
The nurse should teach the patient to avoid prolonged use, which could lead to hypertrichosis (excessive hair growth) and/or steroid-induced acne. The nurse should also tell the patient to avoid applying the corticosteroid to the face and to intertriginous areas.

How well did you know this?
1
Not at all
2
3
4
5
Perfectly
80
Q

The nurse has administered the preanesthetic medication. What action should the nurse take next?

a) Educate the patient on discharge instructions.
b) Review the patient’s list of home medications.
c) Obtain the patient’s signature on the consent form.
d) Place the patient on bed rest with the side rails up.

A

d) Place the patient on bed rest with the side rails up.
Explanation:
The preanesthetic medication can make the patient lightheaded and dizzy. Safety is a priority. The consent form should be signed before the patient is medicated. Consents signed after the patient is medicated are not legal. Reviewing the home medications and educating the patient should take place before the patient is medicated.

How well did you know this?
1
Not at all
2
3
4
5
Perfectly
81
Q

The nurse is caring for a patient who is scheduled for a bronchoscopy. The nurse understands that it is important to provide the required information and appropriate explanations for any diagnostic procedure to a patient with a respiratory disorder in order to do which of the following?

a) Aid the caregivers of the patient
b) Ensure adequate rest periods
c) Manage decreased energy levels
d) Manage respiratory distress

A

c) Manage decreased energy levels

Explanation:
In addition to the nursing management of individual tests, patients with respiratory disorders require informative and appropriate explanations of any diagnostic procedures they will experience. Nurses must remember that for many of these patients, breathing may in some way be compromised and energy levels may be decreased. For that reason, explanations should be brief, yet complete, and may need to be repeated later after a rest period. The nurse must also ensure adequate rest periods before and after the procedures. After invasive procedures, the nurse must carefully assess for signs of respiratory distress.

How well did you know this?
1
Not at all
2
3
4
5
Perfectly
81
Q

What action by the nurse best encompasses the preoperative phase?

a) Educating the patients on signs and symptoms of infection
b) Documenting the application of sequential compression devices (SCD)
c) Shaving the patient using a straight razor
d) Monitoring vital signs every 15 minutes

A

a) Educating the patients on signs and symptoms of infection
Explanation:
Educating the patient on prevention or recognition of complications begins in the preoperative phase. Applying SCD and frequent vital sign monitoring happens after the preoperative phase. Only electric clippers should be used to remove hair.

How well did you know this?
1
Not at all
2
3
4
5
Perfectly
83
Q

The nurse is conducting a service project for a local elderly community group on the topic of hypertension. The nurse will relay that risk factors and cardiovascular problems related to hypertension include which of the following? Select all that apply.

a) Obesity (BMI ≥ 30 kg/m2)
b) Age ≥55 in men
c) Decreased low-density lipoprotein (LDL) levels.
d) Elevated high-density lipoprotein (HDL) cholesterol
e) Smoking

A

a) Obesity (BMI ≥ 30 kg/m2), b) Age ≥55 in men, e) Smoking
Explanation:
Major risk factors (in addition to hypotension) include smoking, dyslipidemia (high LDL, low HDL cholesterol), diabetes mellitus, impaired renal function, obesity, physical inactivity, age (older than 55 years for men, 65 years for women), and family history of cardiovascular disease.

How well did you know this?
1
Not at all
2
3
4
5
Perfectly
85
Q

hoose the statements that correctly match the hypertensive medication with its side effect. Select all that apply.

a) With ACE inhibitors, assess for bradycardia.
b) With thiazide diuretics, monitor serum potassium levels.
c) Direct vasodilators may cause headache and tachycardia.
d) With adrenergic inhibitors, cough is a common side effect.
e) Beta-blockers may cause sedation.

A

b) With thiazide diuretics, monitor serum potassium levels., c) Direct vasodilators may cause headache and tachycardia.
Explanation:
Thiazide diuretics may deplete potassium; many clients will need potassium supplementation. Angiotensin-converting enzyme (ACE) inhibitors can induce a mild to severe dry cough. Beta-blockers may induce decreased heart rate; pulse rate should be assessed b

How well did you know this?
1
Not at all
2
3
4
5
Perfectly
86
Q

The nurse is differentiating between a macule and a papule when evaluating a patient’s skin lesion. The nurse determines that the lesion is a papule when which of the following is noted?

a) Greater than 1 cm in diameter
b) Flat skin color change
c) Elevated and palpable
d) Circumscribed border

A

c) Elevated and palpable

Explanation:
The nurse determines that the lesion is a papule versus a macule when the lesion is noted to be elevated and palpable. Macules are flat, nonpalpable skin color changes. Both macules and papules have circumscribed borders. Macules are less than 1 cm in diameter and papules are less than 0.5 cm in diameter.

How well did you know this?
1
Not at all
2
3
4
5
Perfectly
87
Q

The nurse needs to collect a sputum specimen to identify the presence of tuberculosis (TB). Which nursing action(s) is/are indicated for this type of specimen? Select all that apply.

  1. Collect the specimen in the evening.
  2. Send the specimen immediately to the laboratory.
  3. Ask the client to spit into the sputum container.
  4. Offer mouth care before and after collection of the sputum specimen.
  5. Collect a specimen for 3 consecutive days.
A
  1. Send the specimen immediately to the laboratory.
  2. Offer mouth care before and after collection of the sputum specimen.
  3. Collect a specimen for 3 consecutive days.

Rationale: The sputum specimen should be sent immediately to the laboratory. The client should be provided mouth care before and after the specimen is collected. The sputum specimen should be collected for three consecutive days. Option 1 is incorrect because the sputum specimen is collected in the morning not in the evening. Option 3 is incorrect because the term spit indicates that saliva is being examined. The client needs to cough up or expectorate mucus or sputum.

How well did you know this?
1
Not at all
2
3
4
5
Perfectly
88
Q

A nurse is caring for a patient after a thoracentesis. Which of the following signs if noted in the patient should be reported to the physician immediately?

a) “Patient is becoming agitated and complains of pleuritic pain.”
b) “Patient has subcutaneous emphysema around needle insertion site.”
c) “Patient has an oxygen saturation level of 93%.”
d) “Patient is drowsy and complains of headache.”

A

c) “Patient is becoming agitated and complains of pleuritic pain.”

Explanation:
After a thoracentesis, the nurse monitors the patient for pneumothorax or recurrence of pleural effusion. Signs and symptoms associated with pneumothorax depend on its size and cause. Pain is usually sudden and may be pleuritic. The patient may have only minimal respiratory distress with slight chest discomfort and tachypnea with a small simple or uncomplicated pneumothorax. As the pneumothorax enlarges, the patient may become anxious and develop dyspnea with increased use of the accessory muscles.

How well did you know this?
1
Not at all
2
3
4
5
Perfectly
88
Q

Production of melanin is controlled by a hormone secreted from which of the following glands?

a) Parathyroid
b) Hypothalamus
c) Adrenal
d) Thyroid

A

b) Hypothalamus

Explanation:
The production of melanin is controlled by a hormone secreted from the hypothalamus of the brain called melanocyte-stimulating hormone. Production of melanin is not controlled by the thyroid, adrenal, or parathyroid gland.

How well did you know this?
1
Not at all
2
3
4
5
Perfectly
89
Q

The nurse is caring for a patient admitted with a diagnosis of bacterial pharyngitis. The nurse anticipates the patient will be ordered which of the following medications?

a) Tylenol with codeine
b) Robitussin DM
c) Tylenol
d) Penicillin

A

d) Penicillin

Explanation:
Treatment of choice for bacterial pharyngitis is penicillin. Penicillin V potassium taken for 5 days is the regimen of choice. Traditionally, penicillin was administered as a single injection; however, oral forms are used more often and are as effective and less painful than injections. Penicillin injections are recommended only if there is a concern that the patient will not comply with therapy. Robitussin DM may be used as an antitussive. For severe sore throats aspirin or Tylenol, or Tylenol with codeine may be given.

How well did you know this?
1
Not at all
2
3
4
5
Perfectly
90
Q

A nurse is caring for a patient who had an aortic balloon valvuloplasty. The nurse would inspect the surgical insertion site closely for which of the following complications?

a) Evisceration
b) Bleeding and wound dehiscence
c) Thrombosis and infection
d) Bleeding and infection

A

d) Bleeding and infection

Explanation:
Possible complications of an aortic balloon valvuloplasty include aortic regurgitation, emboli, ventricular perforation, rupture of the aortic valve annulus, ventricular dysrhythmia, mitral valve damage, infection, and bleeding from the catheter insertion sites.

How well did you know this?
1
Not at all
2
3
4
5
Perfectly
91
Q

A patient presents to the ED complaining of anxiety and chest pain after shoveling heavy snow that morning. The patient says that he has not taken nitroglycerin for months but did take three nitroglycerin tablets and although the pain is less, “They did not work all that well.” The patient shows the nurse the nitroglycerin bottle and the prescription was filled 12 months ago. The nurse anticipates which of the following physician orders?

a) Chest x-ray
b) Ativan 1 mg orally
c) Serum electrolytes
d) Nitroglycerin SL

A

d) Nitroglycerin SL
Explanation:
Nitroglycerin is volatile and is inactivated by heat, moisture, air, light, and time. Nitroglycerin should be renewed every 6 months to ensure full potency. The client’s tablets were expired and the nurse should anticipate administering nitroglycerin to assess if the chest pain subsides. The other choices may be ordered at a later time, but the priority is to relieve the patient’s chest pain.

How well did you know this?
1
Not at all
2
3
4
5
Perfectly
93
Q

High or increased compliance occurs in which of the following conditions?

a) Pneumothorax
b) Pleural effusion
c) ARDS (acute respiratory distress syndrome)
d) Emphysema

A

d) Emphysema

Explanation:
High or increased compliance occurs if the lungs have lost their elasticity and the thorax is over-distended as in emphysema. Conditions associated with decreased compliance include pneumothorax, pleural effusion, and ARDS.

How well did you know this?
1
Not at all
2
3
4
5
Perfectly
95
Q
A
How well did you know this?
1
Not at all
2
3
4
5
Perfectly
96
Q

Evidence-based practice is defined as:

A. Nursing care based on tradition

B. Scholarly inquiry of nursing and biomedical research literature

C. A problem-solving approach that integrates best current evidence with clinical practice Correct

D. Quality nursing care provided in an efficient and economically sound manner

A

C. A problem-solving approach that integrates best current evidence with clinical practice Correct

Evidence-based practice integrates best current evidence with clinical expertise and patient/family preferences and values for delivery of optimal health care.

How well did you know this?
1
Not at all
2
3
4
5
Perfectly
97
Q

A client has a wound infection. What local human response should the nurse expect to identify?

A

Edema; Chemical mediators increase the permeability of small blood vessels, thereby causing fluid to move into the interstitial compartment, resulting in local edema.

How well did you know this?
1
Not at all
2
3
4
5
Perfectly
98
Q

A pt comes to the ER complaining of a knifelike pain when taking a deep breath. What does this type of pain likely indicate to the nurse?

  1. Bacterial penumonia
  2. Bronchogenic carcinoma
  3. Lung infarction
  4. Pleurisy
A

Pleurisy

How well did you know this?
1
Not at all
2
3
4
5
Perfectly
100
Q

The nurse is caring for patient experiencing an acute MI (STEMI). The nurse anticipates the physician will prescribe alteplase (Activase). Prior to administering this medication, which of the following questions is most important for the nurse to ask the patient?

a) “How many sublingual nitroglycerin tabs did you take?”
b) “What time did your chest pain start today?”
c) “What is your pain level on a scale of 1 to 10?”
d) “Do your parents have a history of heart disease?”

A

b) “What time did your chest pain start today?”
Explanation:
The patient may be a candidate for thrombolytic (fibrolytic) therapy. These medications are administered if the patient’s chest pain lasts longer than 20 minutes, unrelieved by nitroglycerin, ST-segment elevation in the at least two leads that face the same area of the heart, less than 6 hours from onset of pain. The most appropriate question for the nurse to ask is in relationship to when the chest pain began. The other questions would not aid in determining if the patient is a candidate for thrombolytic therapy.

How well did you know this?
1
Not at all
2
3
4
5
Perfectly
101
Q

A patient has had a 12-lead -ECG completed as part of an annual physical examination. The nurse notes an abnormal Q wave on an otherwise unremarkable ECG. The nurse recognizes this finding indicates which of the following?

a) Variant angina
b) An evolving MI
c) A past MI
d) A cardiac dysrhythmia

A

c) A past MI
Explanation:
An abnormal Q wave may be present without ST-segment and T-wave changes, which indicates an old, not acute, MI

How well did you know this?
1
Not at all
2
3
4
5
Perfectly
101
Q

The nurse is working with the parents of a seriously
ill newborn. Surgery has been proposed for the
infant, but the chances of success are unclear. In
helping the parents resolve this ethical conflict, the
nurse knows that the first step is:

  1. Exploring reasonable courses of action
  2. Identifying people who can solve the difficulty
  3. Clarifying values related to the cause of the
    dilemma
  4. Collecting all available information about the
    situation.
A
  1. Collecting all available information about the
    situation. .

Incorporate as much information as possible
from a variety of sources such as laboratory and
test results; the clinical state of the patient; current
literature about the condition; and the patient’s
religious, cultural, and family situation.

How well did you know this?
1
Not at all
2
3
4
5
Perfectly
101
Q

Proper technique for performing a wound culture includes what?

  1. Cleansing the wound prior to obtaining the specimen.
  2. Swabbing for the specimen in the area with the largest collection of drainage.
  3. Removing crusts or scabs with sterile forceps and then culturing the site beneath.
  4. Waiting 8 hours following a dose of antibiotic to obtain the specimen.
A
  1. Cleansing the wound prior to obtaining the specimen; Wound culture specimens should be obtained from a cleaned area of the wound. Microbes responsible for infection are more likely to be found in viable tissue.

Collected drainage contains old and mixed organisms. An appropriate specimen can be obtained without causing the client the discomfort of debriding. The nurse does not generally debride a wound to obtain a specimen. Once systemic antibiotics have been begun, the interval following a does will not significantly affect the concentration of wound organisms.

How well did you know this?
1
Not at all
2
3
4
5
Perfectly
102
Q

The nurse is caring for a patient prescribed warfarin (Coumadin) orally. The nurse reviews the patient’s prothrombin time (PT) level to evaluate the effectiveness of the medication. The nurse should also evaluate which of the following laboratory values?

a) International normalized ratio (INR)
b) Partial thromboplastic time (PTT)
c) Sodium
d) Complete blood count (CBC)

A

a) International normalized ratio (INR)
Explanation:
The INR, reported with the PT, provides a standard method for reporting PT levels and eliminates the variation of PT results from different laboratories. The INR, rather than the PT alone, is used to monitor the effectiveness of warfarin. The therapeutic range for INR is 2 to 3.5, although specific ranges vary based on diagnosis. The other laboratory values are not used to evaluate the effectiveness of Coumadin.

How well did you know this?
1
Not at all
2
3
4
5
Perfectly
103
Q

The nurse is caring for a patient in the ICU who is receiving mechanical ventilation. Which of the following nursing measures are implemented in an effort to reduce the patient’s risk of developing ventilator-associated pneumonia (VAP)?

a) Maintaining the patient in a high Fowler’s position
b) Cleaning the patient’s mouth with chlorhexidine daily
c) Turning and repositioning the patient every 4 hours
d) Ensuring that the patient remains sedated while intubated

A

b) Cleaning the patient’s mouth with chlorhexidine daily

Explanation:
The five key elements of the VAP bundle include the following: elevation of the head of the bed (30 to 45 degrees: semi-Fowler’s position), daily “sedation vacations,” and assessment of readiness to extubate (see below); peptic ulcer disease prophylaxis (with histamine-2 receptor antagonists, such as ranitidine [Zantac]); deep venous thrombosis (DVT) prophylaxis; and daily oral care with chlorhexidine (0.12% oral rinses). The patient should be turned and repositioned every 2 hours to prevent complications of immobility and atelectasis and to optimize lung expansion.

How well did you know this?
1
Not at all
2
3
4
5
Perfectly
104
Q

What action during a surgical procedure requires immediate intervention by the circulating nurse?

a) The surgeon reaching within the sterile field to obtain equipment
b) The scrub nurse calling the blood bank to obtain blood products
c) The anesthesiologist monitoring blood gas levels
d) The registered nurse’s first assistant suturing the surgical wound

A

b) The scrub nurse calling the blood bank to obtain blood products
Explanation:
The scrub nurse is “scrubbed” in and should only come in contact with sterile equipment. Using the phone to call the blood bank is the responsibility of the circulating nurse and it would break the sterility of the scrub nurse. The surgeon has “scrubbed” and should only touch within sterile fields. The anesthesiologist should monitor blood gas levels as needed, and it is appropriate for the registered nurse first assistant to suture the surgical wound.

How well did you know this?
1
Not at all
2
3
4
5
Perfectly
106
Q

The nurse would identify which of the following vitamin deficiencies to prevent the complication of hemorrhaging during surgery?

a) Magnesium
b) Zinc
c) Vitamin K
d) Vitamin A

A

c) Vitamin K
Explanation:
Vitamin K is important for normal blood clotting. Vitamin A and zinc deficiencies would affect the immune system, whereas a magnesium deficiency would delay wound healing.

How well did you know this?
1
Not at all
2
3
4
5
Perfectly
107
Q

The nurse is assessing a patient admitted with infective endocarditis. Which of the following manifestations would the nurse expect to find?

a) Involuntary muscle movements of the extremities
b) Raised red rash on the trunk and face
c) Bruising on the palms of the hands and soles of the feet
d) Small painful lesions on the pads of the fingers and toes

A

d) Small painful lesions on the pads of the fingers and toes
Explanation:
Primary presenting symptoms of infective endocarditis are fever and a heart murmur. In addition small, painful nodules (Osler nodes) may be present in pads of fingers or toes.

How well did you know this?
1
Not at all
2
3
4
5
Perfectly
108
Q

The nurse is caring for a patient with herpes zoster. The nurse describes the lesions in the patient’s chart as which of the following?

a) Pustules
b) Vesicles
c) Wheals
d) Cysts

A

b) Vesicles

Explanation:
The lesions form herpes zoster are vesicles, defined as circumscribed, elevated, palpable, lesions that contain serous fluid and are less than 0.5 cm in diameter. Wheals are elevated masses with transient, irregular borders. Pustules are pus-filled lesions. Cysts are encapsulated fluid-filled or semisolid masses in the subcutaneous tissue or dermis.

How well did you know this?
1
Not at all
2
3
4
5
Perfectly
109
Q

The nurse is caring for a patient following a wedge resection. While the nurse is assessing the patient’s chest tube drainage system, constant bubbling is noted in the water seal chamber. This finding indicates which of the following problems?

a) Tension pneumothorax
b) Air leak
c) Tidaling
d) Increased drainage

A

b) Air leak

Explanation:
The nurse needs to observe for air leaks in the drainage system; they are indicated by constant bubbling in the water seal chamber, or by the air leak indicator in dry systems with a one-way valve. Tidaling is fluctuation of the water level in the water seal that shows effective connection between the pleural cavity and the drainage chamber and indicates that the drainage system remains patent.

How well did you know this?
1
Not at all
2
3
4
5
Perfectly
111
Q

The nurse teaches the patient who demonstrates herpes zoster (shingles) which of the following?

a) Once a patient has had shingles, they will not have it a second time.
b) The infection results from reactivation of the chickenpox virus.
c) No known medications affect the course of shingles.
d) A person who has had chickenpox can contract it again upon exposure to a person with shingles.

A

b) The infection results from reactivation of the chickenpox virus.

Explanation:
It is assumed that herpes zoster represents a reactivation of latent varicella (chickenpox) virus and reflects lowered immunity. It is believed that the varicella zoster virus lies dormant inside nerve cells near the brain and spinal cord and is reactivated with weakened immune systems and cancers. A person who has had chickenpox is immune and, therefore, not at risk of infection after exposure to patients with herpes zoster. Some evidence indicates that infection is arrested if oral antiviral agents are administered within 24 hours of the initial eruption.

How well did you know this?
1
Not at all
2
3
4
5
Perfectly
113
Q

The nurse auscultates the lung sounds of a patient during a routine assessment. The sounds produced are harsh and cracking, sounding like two pieces of leather being rubbed together. The nurse would be correct in documenting this finding as which of the following?

a) Crackles
b) Sonorous wheezes
c) Sibilant wheezes
d) Pleural friction rub

A

d) Pleural friction rub

Explanation:
A pleural friction rub is heard secondary to inflammation and loss of lubricating pleural fluid. Crackles are soft, high-pitched, discontinuous popping sounds that occur during inspiration. Sonorous wheezes are deep, low-pitched rumbling sounds heard primarily during expiration. Sibilant wheezes are continuous, musical, high-pitched, whistlelike sounds heard during inspiration and expiration.

How well did you know this?
1
Not at all
2
3
4
5
Perfectly
115
Q

Which of the following interventions does a nurse implement for patients with empyema?

a) Do not allow visitors with respiratory infections
b) Encourage breathing exercises
c) Institute droplet precautions
d) Place suspected patients together

A

b) Encourage breathing exercises

Explanation:
The nurse instructs the patient in lung-expanding breathing exercises to restore normal respiratory function.

How well did you know this?
1
Not at all
2
3
4
5
Perfectly
115
Q

A patient admitted to the coronary care unit (CCU) diagnosed with a STEMI is anxious and fearful. Which of the following medications will the nurse administer to relieve the patient’s anxiety and decrease cardiac workload?

a) Norvasc (amlodipine)
b) Tenormin (atenolol)
c) IV morphine
d) IV nitroglycerin

A

c) IV morphine
Explanation:
IV morphine is the analgesic of choice for treatment of an acute MI. It is given to reduce pain and treat anxiety. It also reduces preload and afterload, which decreases the workload of the heart. IV nitroglycerin is given to alleviate chest pain. Administration of Tenormin and Norvasc are not indicated in this situation.

How well did you know this?
1
Not at all
2
3
4
5
Perfectly
115
Q

A pt has contact dermatitis on the hand and the nurse observes an area that is thickened and rough btw the thumb and forefinger. What does the nurse know that this is significant of related to repeated scratching and rubbing?

  1. Atrophy
  2. Lichenification
  3. Keloid
  4. Scales
A

Lichenfication

How well did you know this?
1
Not at all
2
3
4
5
Perfectly
116
Q

The scope of nursing practice is legally defined by:
1. State Nurse Practice Acts
2. Professional nursing organizations
3. Hospital policy and procedure manuals
4. Health care providers in the employing
institutions

A
  1. State Nurse Practice Acts

Determines the legal boundaries within each state

How well did you know this?
1
Not at all
2
3
4
5
Perfectly
117
Q

The nurse is preparing to assist the health care provider with the removal of a patient’s chest tube. Which of the following instructions will the nurse correctly give the patient?

a) “During the removal of the chest tube, do not move because it will make the removal more painful.”
b) “Exhale forcefully while the chest tube is being removed.”
c) “While the chest tube is being removed, raise your arms above your head.”
d) “When the tube is being removed, take a deep breath, exhale, and bear down.”

A

d) “When the tube is being removed, take a deep breath, exhale, and bear down.”

Explanation:
When assisting in the chest tube’s removal, instruct the patient to perform a gentle Valsalva maneuver or to breathe quietly. The chest tube is then clamped and quickly removed. Simultaneously, a small bandage is applied and made airtight with petrolatum gauze covered by a 4 × 4-inch gauze pad and thoroughly covered and sealed with nonporous tape. The other options are incorrect instructions for the patient.

How well did you know this?
1
Not at all
2
3
4
5
Perfectly
118
Q

A patient presents to the ED complaining of severe coughing episodes. The patient states the “episodes are more intense at night.” The nurse should suspect which of the following conditions based on the patient’s primary complaint?

a) Bronchitis
b) Left-sided heart failure
c) Emphysema
d) Chronic obstructive pulmonary disorder (COPD)

A

b) Left-sided heart failure

Explanation:
Coughing at night may indicate the onset of left-sided heart failure or bronchial asthma. A cough in the morning with sputum production may indicate bronchitis. A cough that worsens when the patient is supine suggests postnasal drip (rhinosinusitis). Coughing after food intake may indicate aspiration of material into the tracheobronchial tree. A cough of recent onset is usually from an acute infection. A cough that occurs more frequently at night is not associated with COPD, emphysema, or bronchitis.

How well did you know this?
1
Not at all
2
3
4
5
Perfectly
119
Q

In relation to the structure of the larynx, the cricoid cartilage is which of the following?

a) The largest of the cartilage structures
b) The valve flap of cartilage that covers the opening to the larynx during swallowing
c) Used in vocal cord movement with the thyroid cartilage
d) The only complete cartilaginous ring in the larynx

A

d) The only complete cartilaginous ring in the larynx

Explanation:
The cricoid cartilage is the only complete cartilaginous ring in the larynx (located below the thyroid cartilage). The arytenoid cartilages are used in vocal cord movement with the thyroid cartilage. The thyroid cartilage is the largest of the cartilage structures; part of it forms the Adam’s apple. The epiglottis is the valve flap of cartilage that covers the opening to the larynx during swallowing.

120
Q

The nurse is completing a preoperative assessment. The nurse notices the patient is tearful and constantly wringing hands. The patient states, “I’m really nervous about this surgery. Do you think it will be ok?” What is the nurse’s best response?

a) “You have nothing to worry about; you have the best surgical team.”
b) “What family support do you have after the surgery?”
c) “What are your concerns?”
d) “No one has ever died from the procedure you are having.”

A

c) “What are your concerns?”
Explanation:
Asking the patient about their concerns is an open-ended therapeutic technique. It allows the patient to guide the conversation and address their emotional state. Asking about family support is changing the subject and is nontherapeutic. Discussing the surgical team and the low death rate associated with a procedure is minimizing the patient’s feelings and is nontherapeutic.

121
Q

Target organ damage from untreated/undertreated hypertension includes which of the following? Select all that apply.

a) Hyperlipidemia
b) Stroke
c) Diabetes
d) Retinal damage
e) Heart failure

A

b) Stroke, d) Retinal damage, e) Heart failure
Explanation:
Target organ systems include cardiac, cerebrovascular, peripheral vascular, renal, and the eye. Hyperlipidemia and diabetes are risk factors for development of hypertension.

123
Q

A nurse is conducting a heath history on a patient with a primary diagnosis of mitral stenosis. Which of the following disorders reported by the patient is the most common cause of mitral stenosis?

a) Congestive heart failure
b) Rheumatic endocarditis
c) Atrial fibrillation
d) Myocardial infarction

A

b) Rheumatic endocarditis
Explanation:
Mitral stenosis is most often caused by rheumatic endocarditis, which progressively thickens the mitral valve leaflets and chordate tendineae. Leaflets often fuse together. Eventually, the mitral valve orifice narrows and progressively obstructs blood flow into the ventricle.

123
Q

The anesthesiologist administered a transsacral conduction block. Which of the following documentation by the nurse is consistent with the anesthesia being administered?

a) No movement in right lower leg
b) Denies sensation to perineum and lower abdomen
c) Yelling and pulling at equipment
d) Unresponsive to verbal or tactile stimuli

A

b) Denies sensation to perineum and lower abdomen
Explanation:
A transsacral block produces anesthesia of the perineum, and occasionally, the lower abdomen. Yelling and pulling at equipment can be related to the excitement phase of general anesthesia. Unresponsive to verbal or tactile stimuli and no movement in the right lower leg are not consistent with a transsacral conduction block.

123
Q

You are at the scene of an accident and find the victim has a bleeding lower leg wound. After flushing the wound with water and covering it with a clean dressing, you find the dressing has been saturated with blood. Which of the following would be the best action to take in this case?

  1. Lower the extremity while applying pressure to the wound.
  2. Take off the first dressing and apply another clean or sterile dressing.
  3. Encircle the client’s ankle with your hands and apply pressure.
  4. Reinforce the first layer of dressing with a second layer of dressing.
A
  1. Reinforce the first layer of dressing with a second layer of dressing; To control severe bleeding, apply direct pressure to the wound and elevate the extremity. If the dressing becomes saturated, apply a second layer. Removing the first dressing may disturb blood clots and increase the bleeding.
124
Q

A patient is postoperative day 3 for surgical repair of an open abdominal wound and traumatic amputation of the right lower leg following a motorcycle crash. What is the highest priority nursing intervention?

a) Assessing WBC count, temperature, and wound appearance
b) Educating patient on safe bed-to-chair transfer procedures
c) Administering pain medications within 1 hour of the patient’s request
d) Obtaining dietary consultation for improved wound healing

A

a) Assessing WBC count, temperature, and wound appearance
Explanation:
The patient has an increased risk for infection related to the surgical wound classification of dirty. Assessing the WBC count, temperature, and wound appearance will allow the nurse to intervene at the earliest sign of infection. The patient will have special nutritional needs for wound healing and need education on safe transfer procedures but the need to monitor for infection is a higher priority. The patient should receive pain medication as soon as possible after asking but the latest literature suggest that pain medication should be given on a schedule versus “as needed.”

125
Q

An asymptomatic patient questions the nurse about the diagnosis of mitral regurgitation and inquires about continuing an exercise routine. Which of the following is the most appropriate nursing response?

a) Continue the exercise routine unless symptoms such as shortness of breath or fatigue develop.
b) Avoid strenuous cardiovascular exercise.
c) Continue the exercise routine but take ample rest after exercising.
d) Avoid any type of exercise.
Show Answer

A
127
Q

The ability of the cardiac muscle to shorten in response to an electrical impulse is termed which of the following?

a) Repolarization
b) Diastole
c) Contractility
d) Depolarization

A

c) Contractility

Explanation:
Contractility is the ability of the cardiac muscle to shorten in response to an electrical impulse. Depolarization is the electrical activation of a cell caused by the influx of sodium into the cell while potassium exits the cell. Repolarization is the return of the cell to the resting state, caused by reentry of potassium into the cell while sodium exits the cell. Diastole is the period of ventricular relaxation resulting in ventricular filling.

128
Q

Which of the following is an age-related change associated with the respiratory system?

a) Decreased size of the airway
b) Decreased thickening of alveolar membranes
c) Increased elasticity of alveolar sacs
d) Increased chest muscle mass

A

a) Decreased size of the airway

Explanation:
Age-related changes that occur in the respiratory system are a decrease in the size of the airway, decreased chest muscle mass, increased thickening of the alveolar membranes, and decreased elasticity of the alveolar sacs.

128
Q

The nurse teaches the patient which of the following guidelines regarding lifestyle modifications for hypertension?

a) Maintain adequate dietary intake of fruits and vegetables
b) Stop alcohol intake
c) Limit aerobic physical activity to 15 minutes, three times per week
d) Reduce smoking to no more than four cigarettes per day

A

a) Maintain adequate dietary intake of fruits and vegetables
Explanation:
Guidelines include adopting the dietary approaches to stop hypertension (DASH) eating plan: consume a diet rich in fruits, vegetables, and low-fat dairy products with a reduced content of saturated and total fat, dietary sodium reduction: reduce dietary sodium intake to no more than 100 mmol/day (2.4 g sodium or 6 g sodium chloride), and physical activity: engage in regular aerobic physical activity such as brisk walking (at least 30 min/day, most days of the week), Moderate alcohol consumption: limit consumption to no more than two drinks (eg, 24 oz beer, 10 oz wine, or 3 oz 80-proof whiskey) per day in most men and to no more than 1 drink per day in women and lighter-weight people. Tobacco: should be avoided because anyone with high blood pressure is already at increased risk for heart disease, and smoking amplifies this risk.

129
Q

Thirty minutes after application is initiated, the client requests that the nurse leave the heating pad in place. The nurse explains to the client that:

  1. Heat application for longer than thirty minutes can actually cause the opposite effect (constriction) of the one desired (dilation)
  2. It will be acceptable to leave the pad in place for another thirty minutes
A
  1. Heat application for longer than thirty minutes can actually cause the opposite effect (constriction) of the one desired (dilation); The heating pads need to be removed. After 30 minutes of heat application, the blood vessels in the area will begin to exhibit the rebound effect resulting in vasoconstriction.

Lowering the temperature, but still delivering heat -dry or moist- will not prevent the rebound effect. The visual appearance of the site on inspection (option 3) does not indicate if rebound is occurring.

131
Q

The nurse is educating a community group regarding types of surgery. A member of the group asks the nurse to describe a type of surgery that is curative. What response by the nurse is true?

a) A face-lift
b) The placement of gastrostomy tube
c) A biopsy
d) The excision of a tumor

A

d) The excision of a tumor
Explanation:
An example of a curative surgical procedure is the excision of a tumor. A biopsy, a face-lift, and the placement of a gastrostomy tube are not examples of curative surgical procedures.

132
Q

A patient presents to the ED with a suspected allergic reaction. The patient is experiencing laryngeal edema causing obstruction and is demonstrating retractions in the neck during inspirations. Which of the following is the nurse’s priority intervention?

a) Prepare for endotracheal intubation with mechanical ventilation.
b) Prepare to administer subcutaneous epinephrine and corticosteroids.
c) Apply 100% oxygen via a face mask.
d) Prepare for immediate tracheostomy.

A

b) Prepare to administer subcutaneous epinephrine and corticosteroids.

Explanation:
The use of accessory muscles to maximize airflow is often manifested by retractions in the neck during inspirations and is an ominous sign of impending respiratory distress. The patient’s obstruction is caused by edema resulting from an allergic reaction, and treatment should include immediate administration of subcutaneous epinephrine and a corticosteroid. The other interventions may be indicated for a patient with a laryngeal obstruction; however, in this instance the most appropriate intervention to treat the patient’s laryngeal edema is the administration of the medications.

133
Q

When performing endotracheal suctioning, the nurse applies suctioning while withdrawing and gently rotating the catheter 360 degrees for which of the following time periods?

a) 20 to 25 seconds
b) 30 to 35 seconds
c) 10 to 15 seconds
d) 0 to 5 seconds

A

c) 10 to 15 seconds

Explanation:
In general, the nurse should apply suction no longer than 10 to15 seconds because hypoxia and dysrhythmias may develop, leading to cardiac arrest. Applying suction for 30 to 35 seconds is hazardous and may result in the patient’s developing hypoxia, which can lead to dysrhythmias and, ultimately, cardiac arrest. Applying suction for 20 to 25 seconds is hazardous and may result in the patient’s developing hypoxia, which can lead to dysrhythmias and, ultimately, cardiac arrest. Applying suction for 0 to 5 seconds would provide too little time for effective suctioning of secretions.

134
Q

The nurse understands that patient education related to antihypertensive medication should include all of the following instructions except which of the following?

a) Avoid hot baths, exercise, and alcohol within 3 hours of taking vasodilators.
b) Avoid over the counter (OTC) cold, weight reduction, and sinus medications.
c) If a dosage of medication is missed, double up on the next one to catch up.
d) Do not stop antihypertensive medication abruptly.

A

c) If a dosage of medication is missed, double up on the next one to catch up.
Explanation:
Doubling doses could cause serious hypotension (HTN) and is not recommended. Medications should be taken as prescribed. Hot baths, strenuous exercise, and excessive alcohol are all vasodilators and should be avoided. Many OTC preparations can precipitate HTN. Stopping antihypertensives abruptly can precipitate a severe hypertensive reaction and is not recommended.

135
Q

Which respiratory volume is the maximum volume of air that can be inhaled after maximal expiration?

a) Residual volume
b) Expiratory reserve volume
c) Inspiratory reserve volume
d) Tidal volume

A

c) Inspiratory reserve volume

Explanation:
The maximum volume of air that can be inhaled after a normal inhalation is termed inspiratory reserve volume. Inspiratory reserve volume is normally 3,000 mL. Tidal volume is the volume of air inhaled and exhaled with each breath. Expiratory reserve volume is the maximum volume of air that can be exhaled forcibly after a normal exhalation. Residual volume is the volume of air remaining in the lungs after a maximum exhalation.

136
Q

Which of the following skin conditions is caused by staphylococci, streptococci, or multiple bacteria?

a) Impetigo
b) Scabies
c) Pediculosis capitis
d) Poison ivy

A

a) Impetigo

Explanation:
Impetigo is seen at all ages, but is particularly common among children living under poor hygienic conditions. Scabies is caused by the itch mite. Pediculosis capitis is caused by head lice. Poison ivy is a contact dermatitis caused by the oleoresin given off by a particular form of ivy.

137
Q

A patient has undergone hernia repair surgery without complications. In the immediate postoperative period, which of the following actions by the nurse is most appropriate?

a) Monitor vital signs every 15 minutes
b) Assessing pupillary response every 5 minutes
c) Measuring urinary output every 15 minutes
d) Obtaining arterial blood gas every 5 minutes

A

a) Monitor vital signs every 15 minutes
Explanation:
The pulse rate, blood pressure, and respiration rate are recorded at least every 15 minutes for the first hour and every 30 minutes for the next 2 hours. Obtaining an arterial blood gas every 5 minutes is painful to the patient unless a special device is inserted to obtain arterial blood samples. Without complications, this is not indicated for the patient. Urinary output is monitored frequently but usually measured hourly. While it may be necessary to assess pupillary response during the immediate postoperative phase, it does not need to be done every 5 minutes.

137
Q

All of the following measures are used to assess for
deep vein thrombosis except:
1. Checking for a positive Homans sign
2. Asking the patient about the presence of calf
pain
3. Observing the dorsal aspect of lower extremities
for redness, warmth, and tenderness
4. Measuring the circumference of each leg daily,
placing the tape measure at the midpoint of the
knee

A
  1. Measuring the circumference of each leg daily, placing the tape measure at the midpoint of the knee

Need to measure bilateral calf circumference

139
Q

The nurse is assessing the periwound skin of an African American for inflammation. The nurse determines that inflammation is present when which of the following is noted?

a) Blue-green hue
b) White patches
c) Red coloration
d) Purple-gray cast

A

d) Purple-gray cast

Explanation:
The nurse determines that inflammation is present when a purple-gray cast is noted. Inflammation in light-skinned people is noted by erythema or a redness of the skin.

141
Q

A patient with restrictive cardiomyopathy (RCM) is taking digoxin. Because of the risk of increased sensitivity, the nurse should carefully assess the patient for which of the following manifestations?

a) Tachypnea and dyspnea
b) Anorexia and confusion
c) Edema and orthopnea
d) Abdominal pain and diarrhea

A

b) Anorexia and confusion
Explanation:
Patients with RCM have increased sensitivity to digoxin, and the nurse must anticipate that low doses will be prescribed and assess for digoxin toxicity. The most common manifestations of digoxin toxicity are gastrointestinal (anorexia, nausea, and vomiting), cardiac (rhythm disturbances and heart block), and central nervous system (CNS) disturbances (confusion, headache, weakness, dizziness, and blurred or yellow vision).

143
Q

The nurse is taking a respiratory history for a pt who has come into the clinic with a chronic cough. What information should the nurse obtain from the pt? (Select all that apply)

  1. Financial ability to pay the bill
  2. Social support
  3. Previous history of lung disease in the patient or family
  4. Occupational and environmental influences
  5. Previous history of smoking
A

Previous hx of lung disease in the pt or family, occupational and environmental influences, previous hx of smoking

144
Q

A practitioner orders a wound to be packed with a wet-to-damp gauze dressing. What should the nurse explain to the client is the primary reason for this type of dressing?

A

Packing the wound with wet-to-damp dressings allows epidermal cells to migrate more rapidly across the bed of the wound surface than dry dressings, thereby facilitating healing.

Wet-to-damp dressings will also wick exudate up and away from the base of the wound and help to increase resistance to a wound infection. 

145
Q

A patient admitted to the hospital following a motor vehicle crash has suffered a flail chest. A nurse assesses the patient for what most common clinical manifestation of flail chest?

a) Cyanosis
b) Hypertension
c) Paradoxical chest movement
d) Wheezing

A

c) Paradoxical chest movement

Explanation:
During inspiration, as the chest expands, the detached part of the rib segment (flail segment) moves in a paradoxical manner (pendelluft movement) in that it is pulled inward during inspiration, reducing the amount of air that can be drawn into the lungs. On expiration, because the intrathoracic pressure exceeds atmospheric pressure, the flail segment bulges outward, impairing the patient’s ability to exhale. The mediastinum then shifts back to the affected side. This paradoxical action results in increased dead space, a reduction in alveolar ventilation, and decreased compliance.

147
Q

A nurse caring for pt with a pulmonary embolism understands a high ventilation/perfusion ration may exist. What does this mean for the pt?

  1. Perfusion exceeds ventilation
  2. There is an absence of perfusion and ventilation
  3. Ventilation exceeds perfusion
  4. Ventilation matches perfusion
A

Ventilation exceeds perfusion

148
Q

A patient with fractured skull after falling from a ladder requires surgery. The nurse should anticipate transporting the patient to surgery during what time frame?

a) Immediately
b) In 1 day
c) In 48–72 hours
d) In 1 week

A

a) Immediately
Explanation:
Emergent surgery occurs when the patient requires immediate attention. A fractured skull is an indication for emergent surgery. An urgent surgery occurs when the patient requires prompt attention, usually within 24–30 hours. Any surgery scheduled beyond 30 hours is classified as required or elective and a fractured skull does not meet the requirements for elective or required surgery.

149
Q

The nurse is caring for a patient following a thoracotomy. Which of the following findings requires immediate intervention by the nurse?

a) Moderate amounts of colorless sputum
b) Chest tube drainage of 190 mL/hr
c) Pain of 5 on a 1 to 10 pain scale
d) Heart rate: 112 bpm

A

b) Chest tube drainage of 190 mL/hr

Explanation:
The nurse should monitor and document the amount and character of drainage every 2 hours. The nurse will notify the primary provider if drainage is 150 mL/hr or greater. The other findings are normal following a thoracotomy; no intervention is required.

150
Q

A primary care provider is going to perform a thoracentesis. The nurse’s role will include which action?

  1. Place the client supine in the Trendelenburg position.
  2. Position the client in a seated position with elbows on the overbed table.
  3. Instruct the UAP to measure vital signs.
  4. Administer an opioid analgesic.
A
  1. Position the client in a seated position with elbows on the overbed table.

Rationale: The puncture site is usually on the posterior chest. The client should be positioned leaning forward. This will allow the ribs to separate for exposure of the site. Option 1 is incorrect. The client should not be placed in the Trendelenburg position because the site would not be exposed. Option 3 is incorrect since changes in vital signs do not routinely occur with this procedure. Option 4 is incorrect. The client does not need to be medicated for pain with this procedure.

152
Q

Which of the following sedative medications is effective for treating pruritus?

a) Benzoyl peroxide
b) Tetracycline
c) Hydroxyzine (Atarax)
d) Fexofenadine (Allegra)

A

c) Hydroxyzine (Atarax)

Explanation:
Atarax is a sedating medication effective in the treatment of pruritus. Benzoyl peroxide, Allegra, and tetracycline are not effective in treating pruritus

153
Q

The client is only comfortable lying on the right side or left side (not on the back or stomach). List at least four potential sites of pressure ulcers the nurse must assess.

A

These are important areas to assess. Potential ulcer sites for side-lying clients include:

  1. Ankles
  2. Knees
  3. Trochanters
  4. Ilia
  5. Shoulders
  6. Ears
154
Q

Which of the following are primary risk factors for pressure ulcers? Select all that apply.

  1. Low-protein diet
  2. Insomnia
  3. Lengthy surgical procedures
  4. Fever
  5. Sleeping on a waterbed
A

1, 3, & 4; Risk factors for pressure ulcers include a low-protein diet, lengthy surgical procedures, and fever.

Protein is needed for adequate skin health and healing. During surgery, the client is on a hard surface and may not be well protected from pressure on bony prominences. Fever increases skin moisture, which can lead to skin breakdown, plus the stress on the body from the cause of the fever could impair circulation and skin integrity. Insomnia (option 2) would generally involve restless sleeping, which transfers pressure to different parts of the body and would reduce chances of skin breakdown. A waterbed (option 5) distributes pressure more evenly than a regular mattress and, thus, actually reduces the chance of skin breakdown. 

156
Q

The nurse is caring for a patient in the ICU who required emergent endotracheal (ET) intubation with mechanical ventilation. The nurse receives an order to obtain arterial blood gases (ABGs) following the procedure. The nurse recognizes that ABGs should be obtained at which timeframe following the initiation of mechanical ventilation?

a) 15 minutes
b) 20 minutes
c) 10 minutes
d) 25 minutes

A

b) 20 minutes

Explanation:
The nurse records minute volume and obtains ABGs to measure carbon dioxide partial pressure (PaCO2), pH, and PaO2 after 20 minutes of continuous mechanical ventilation.

156
Q

When interpreting the results of a Mantoux test, the nurse explains to the patient that a reaction occurs when the intradermal injection site shows which of the following signs?

a) Redness and induration
b) Bruising
c) Tissue sloughing
d) Drainage

A

a) Redness and induration

Explanation:
A reaction occurs when both induration and erythema (redness) are present

156
Q

A client’s family asks you to explain some keloid scars that the client developed. The best explanation of the keloid scars would be that keloid scars are:

  1. Due to a relatively rare inherited tendency.
  2. Caused by an abnormal amount of collagen being laid down in scar formation.
  3. Most common in pale-skinned people of Northern European ancestry.
  4. Caused by repeated and abrupt early disruption of eschar being formed.
A
  1. Caused by an abnormal amount of collagen being laid down in scar formation; Keloid scars are due to an abnormal amount of collagen being laid down in scar formation in the maturation phase, and they are more apt to occur in a dark-skinned person.
157
Q

A pt has a moisture-retentive dressing for the treatment of a sacral decubitus ulcer. How long should the nurse leave the dressing in place before replacing it?

  1. 4-6 hr
  2. 8 h
  3. 12-24 hr
  4. 24-36 hr
A

12-24 hr

158
Q

When caring for an obese client 4 to 5 days post-surgery, who has nausea and occasional vomiting and is not keeping fluids down well, which of the following would you be most concerned about?

  1. Post surgical hemorrhage and anemia
  2. Wound dehiscence and evisceration
  3. Impaired skin integrity and decubitus ulcers
  4. Loss of motility and paralytic illeus
A
  1. Wound dehiscence and evisceration; Wound dehiscence is most likely to occur 4 to 5 days postoperatively, and risk factors include obesity, poor nutrition, multiple trauma, failure of suturing, excessive coughing, vomiting, and dehydration.
159
Q

The nurse examines a pt and notices a herpes simplex/zoster skin lesion. How does the nurse document this lesion?

  1. Macule
  2. Papule
  3. Vesicle
  4. Wheal
A

Vesicle

161
Q

The nurse notes that a patient has round red macules over the lower extremities. The nurse documents this finding as which of the following?

a) Petechiae
b) Spider angioma
c) Telangiectasia
d) Ecchymosis

A

a) Petechiae

Explanation:
Petechiae are associated with bleeding tendencies or emboli to the skin. Spider angioma is associated with liver disease, pregnancy, and vitamin B deficiency. Ecchymosis is associated with trauma and bleeding tendencies. Telangiectasia is associated with venous pressure states.

163
Q

A nurse on the surgical team has been assigned the role of scrub nurse. What action by the scrub nurse is appropriate?

a) Handing instruments to the surgeon and assistants
b) Leading the surgical team in a debriefing session
c) Keeping all records and adjusting lights
d) Coordinating activities of other personnel

A

a) Handing instruments to the surgeon and assistants
Explanation:
The responsibilities of a scrub nurse are to assist the surgical team by handing instruments to the surgeon and assistants, preparing sutures, receiving specimens for laboratory examination, and counting sponges and needles. Responsibilities of a circulating nurse include leading the surgical team in a debriefing session, keeping records, adjusting lights, and coordinating activities of other personnel.

164
Q

A nurse assesses a postoperative patient to have the protrusion of abdominal organs through the surgical incision. Which term, documented by the nurse, best describes the assessment findings?

a) Erythema
b) Hernia
c) Dehiscence
d) Evisceration

A

d) Evisceration
Explanation:
Evisceration is a surgical emergency. A hernia is a weakness in the abdominal wall. Dehiscence refers to the partial or complete separation of wound edges. Erythema refers to the redness of tissue.

166
Q

The nurse is performing an assessment for a pt with CHF. The nurse asks if the pt has difficulty breathing in any position other than upright. What is the nurse referring to?

  1. Dyspnea
  2. Orthopnea
  3. Tachypnea
  4. BradypneaO
A

Orthopnea

167
Q

The nurse is caring for a female client who has had 25 mg of oral hydrochlorothiazide added to her medication regimen for the treatment of hypertension (HTN). Which of the following instructions should the nurse give the patient?

a) “You may develop dry mouth or nasal congestion while on this medication.”
b) “You may drink alcohol while taking this medication.”
c) “Increase the amount of fruits and vegetables you eat.”
d) “Take this medication before going to bed.”

A

c) “Increase the amount of fruits and vegetables you eat.”
Explanation:
Thiazide diuretics cause loss of sodium, potassium, and magnesium. The patient should be encouraged to eat fruits and vegetables which are high in potassium. Diuretics cause increased urination; the patient should not take the medication prior to going to bed. Thiazide diuretics to not cause dry mouth or nasal congestion. Postural hypotension (side effect) may be potentiated by alcohol.

168
Q

The nurse assesses a dark-skinned pt who has cherry-red nail beds, lips, and oral mucosa. What does this assessment data indicate the pt may be experiencing?

  1. Anemia
  2. CO posioning
  3. Polycythemia
  4. ShockCO
A

CO poisoning

170
Q

A medical student, scheduled to observe surgery, enters the unrestricted surgical zone wearing jeans, a t-shirt, and tennis shoes. What is the best action by the nurse?

a) Educate the medical student on required attire for each surgical zone.
b) Provide the medical student a cap and mask.
c) Immediately escort the medical student out of the area.
d) No action is needed.

A

a) Educate the medical student on required attire for each surgical zone.
Explanation:
It would be best to educate the medical student on the required attire for each surgical zone. Since the student will be observing a surgery, the student will need to dress appropriately in each zone to decrease the risk of introducing pathogens. The unrestricted zone allows for street clothes; therefore, the student does not need to be removed. If no action is taken by the nurse, the student could enter the semirestricted or restricted zone without appropriate attire. Providing a cap and mask does not address the need to change out of the street clothes to observe the surgery.

172
Q
A
173
Q

A pt is being educated in the use of an incentive spirometry prior to having a surgical procedure. What should the nurse be sure to include in the education?

  1. Have the pt lie in a supine position during the use of the spirometry
  2. Encourage the pt to try to stop couhing during and after use of spirometry
  3. Inform the pt that using the spirometer is not necessary if the pt is experiencing pain
  4. Encourage the pt to take approximately 10 breaths per hours, while awake
A

Encourage the pt to take approximately 10 breaths/hour, while awake

174
Q

Which of the following is the term for the transfer of heat from the body to a cooler object in contact with it?

a) Conduction
b) Evaporation
c) Radiation
d) Lichenification

A

a) Conduction

Explanation:
Conduction is the transfer of heat from the body to a cooler object in contact with it. Radiation is the transfer of heat to another object of lower temperature situated at a distance. Lichenification is the leather thickening of the skin. Convection consists of movement of warm air molecules away from the body.

175
Q

A patient suspected of developing acute respiratory distress syndrome (ARDS) is experiencing anxiety and agitation due to increasing hypoxemia and dyspnea. A nurse would implement which of the following interventions to improve oxygenation and provide comfort for the patient?

a) Force fluids for the next 24 hours
b) Administer small doses of pancuronium (Pavulon)
c) Position the patient in the prone position
d) Assist the patient up to a chair

A

c) Position the patient in the prone position

Explanation:
The patient is extremely anxious and agitated because of the increasing hypoxemia and dyspnea. It is important to reduce the patient’s anxiety because anxiety increases oxygen expenditure. Oxygenation in patients with ARDS is sometimes improved in the prone position. Rest is essential to limit oxygen consumption and reduce oxygen needs.

176
Q

When the nurse is assessing the skin of a patient with tinea pedis, a linear crack is noted. This is documented by the nurse as which of the following?

a) Scale
b) Ulcer
c) Fissure
d) Erosion

A

c) Fissure

Explanation:
Linear cracks in the skin are documented as fissures. Scales are flakes secondary to desquamated, dead epithelium. Erosions are defined as loss of superficial epidermis that does not extend into the dermis. Ulcers have skin loss that extends past the epidermis.

177
Q

The nurse has completed a teaching session on the self-administration of sublingual nitroglycerin. Which of the following patient statements indicates that the patient teaching has been effective?

a) “Side effects of nitroglycerin include, flushing, throbbing headache, and hypertension”.
b) “I can take nitroglycerin prior to having sexual intercourse so I won’t develop chest pain”.
c) “After taking two tablets with no relief, I should call emergency medical services.”
d) “I can put the nitroglycerin tablets in my daily pill dispenser with my other medications”.

A

b) “I can take nitroglycerin prior to having sexual intercourse so I won’t develop chest pain”.
Explanation:
Nitroglycerin can be taken in anticipation of any activity that may produce pain. Because nitroglycerin increases tolerance for exercise and stress when taken prophylactically (i.e. before angina-producing activity, such as exercise, stair-climbing, or sexual intercourse), it is best taken before pain develops. The client is instructed to take three tablets 5 minutes apart and if the chest pain is not relieved emergency medical services should be contacted. Nitroglycerin is very unstable; it should be carried securely in its original container (e.g., capped dark glass bottle); tablets should never be removed and stored in metal or plastic pillboxes. Side effects of nitroglycerin includes: flushing, throbbing headache, hypotension, and tachycardia.

178
Q

The nurse is auscultating the heart of a patient diagnosed with mitral valve prolapse. Which of the following is often the first and only manifestation of mitral valve prolapse?

a) Extra heart sound
b) Fatigue
c) Syncope
d) Dizziness

A

a) Extra heart sound
Explanation:
Often, the first and only sign of mitral valve prolapse is identified when a physical examination of the heart reveals an extra heart sound referred to as a mitral click. Fatigue, dizziness, and syncope are other symptoms of mitral valve prolapsed.

180
Q

The nurse is caring for a patient with a suspected malignancy of the skin. The nurse anticipates that the patient will undergo which of the following diagnostic tests?

a) Patch test
b) Skin scraping
c) Tzanck smear
d) Biopsy

A

d) Biopsy

Explanation:
Biopsies are performed on skin nodules, plaques, blisters, and other lesions to rule out malignancy and to establish an exact diagnosis. Skin scrapings are used to diagnose spores and hyphae. A Tzanck smear is a test used to examine cells from blistering skin conditions such as herpes zoster. A patch test is used to identify substances to which the patient has developed an allergy.

181
Q

A patient is admitted to the intensive care unit (ICU) with a diagnosis of hypertension emergency/crisis. The patient’s blood pressure (BP) is 200/130 mm Hg. The nurse is preparing to administer IV Nitropress (nitroprusside). Upon assessment, which of the following patient findings requires immediate intervention by the nurse?

a) Chest pain score of 3/10 (on a scale of 1 to 10)
b) Urine output of 40 cc/mL over the last hour
c) Nausea and severe headache
d) Left arm numbness and weakness

A

d) Left arm numbness and weakness
Explanation:
Hypertensive emergencies are acute, life-threatening BP elevations that require prompt treatment in an intensive care setting because of the serious target organ damage that may occur. The finding of left arm numbness and weakness may indicate the patient is experiencing neurological symptoms associated with an ischemic stroke because of the severely elevated BP and requires immediate interventions. A urine output of 40 mL/h is within normal limits. The other findings are likely caused by the hypertension and require intervention, but they do not require action as urgently as the neurologic changes.

183
Q

What finding by the nurse may indicate that the pt has chronic hypoxia?

  1. Crackles
  2. Peripheral edema
  3. Clubbing of the fingers
  4. Cyanosis
A

Clubbing of the fingers

184
Q

The nurse is caring for a patient newly diagnosed with hypertension. Which of the following statements if made by the patient indicates the need for further teaching?

a) “When getting up from bed, I will sit for a short period prior to standing up.”
b) “If I take my blood pressure and it is normal, I don’t have to take my BP pills.”
c) “I think I’m going to sign up for a yoga class twice a week to help reduce my stress.”
d) “I will consult a dietician to help get my weight under control.”

A

b) “If I take my blood pressure and it is normal, I don’t have to take my BP pills.”
Explanation:
The patient needs to understand the disease process and how lifestyle changes and medications can control hypertension. The patient must take his/her medication as directed. A normal BP indicates the medication is producing its desired effect. The other responses do not indicate the need for further teaching.

185
Q

The nurse is caring for a pt with a pulmonary disorder. What observation by the nurse is indicative of a very late symptom of hypoxia?

  1. Cyanosis
  2. Dyspnea
  3. Restlessness
  4. ConfusionC
A

Cyanosis

187
Q

The nurse is caring for a patient presenting to the emergency department (ED) complaining of chest pain. Which of the following electrocardiographic (ECG) findings would be most concerning to the nurse?

a) ST elevations
b) Frequent premature atrial contractions (PACs)
c) Isolated premature ventricular contractions (PVCs)
d) Sinus tachycardia

A

a) ST elevations
Explanation:
The first signs of an acute MI are usually seen in the T wave and ST segment. The T wave becomes inverted; the ST segment elevates (usually flat). An elevation in ST segment in two contiguous leads is a key diagnostic indicator for MI (i.e. ST elevation myocardial infarction, STEMI). This patient requires immediate invasive therapy or fibrinolytic medications. Although the other ECG findings require intervention, elevated ST elevations require immediate and definitive interventions.

188
Q

When the balloon on the distal tip of a pulmonary artery catheter is inflated and a pressure is measured, the measurement obtained is referred to as which of the following?

a) Cardiac output
b) Pulmonary artery pressure
c) Central venous pressure
d) Pulmonary artery wedge pressure

A

d) Pulmonary artery wedge pressure
Explanation:
When the balloon is inflated, the tip of the catheter floats into smaller branches of the pulmonary artery until it can no longer be passed and the pressure is recorded, reflecting left atrial pressure and left ventricular end-diastolic pressure. Central venous pressure is measured in the right atrium. Pulmonary artery pressure is measured when the balloon tip is not inflated. Cardiac output is determined through thermodilution involving injection of fluid into the pulmonary artery catheter

189
Q

The nurse is assigned the care of a patient with a chest tube. The nurse should ensure that which of the following items is kept at the patient’s bedside?

a) A bottle of sterile water
b) A set of hemostats
c) An incentive spirometer
d) An Ambu bag

A

a) A bottle of sterile water

Explanation:
It is essential that the nurse ensure that a bottle of sterile water is readily available at the patient’s bedside. If the chest tube and drainage system become disconnected, air can enter the pleural space, producing a pneumothorax. To prevent the development of a pneumothorax, a temporary water seal can be established by immersing the chest tube’s open end in a bottle of sterile water. There is no need to have an Ambu bag, incentive spirometer, or a set of hemostats at the bedside.

190
Q

Which of the following is the priority nursing diagnosis for the patient undergoing a laryngectomy?

a) Ineffective airway clearance
b) Impaired verbal communication
c) Anxiety and depression
d) Imbalanced nutrition: Less than body requirements

A

a) Ineffective airway clearance

Explanation:
The priority nursing diagnosis is Ineffective airway clearance, utilizing the ABCs. Imbalanced nutrition: Less than body requirement, impaired verbal communication, and anxiety and depression are all potential nursing diagnoses, but they are not a priority diagnosis.

190
Q

The examination for registered nurse licensure is exactly the same in every state in the United States. This examination:

A. Guarantees safe nursing care for all patients

B. Ensures standard nursing care for all patients

C. Ensures that honest and ethical care is provided

D. Provides a minimal standard of knowledge for a registered nurse in practice

A

D. Provides a minimal standard of knowledge for a registered nurse in practice

Registered nurse (RN) candidates must pass the NCLEX-RN® that the individual State Boards of Nursing administer. Regardless of educational preparation, the examination for RN licensure is exactly the same in every state in the United States. This provides a standardized minimum knowledge base for nurses.

190
Q

The nurse is providing instruction to a patient with acne. The nurse promotes avoidance for which of the following foods? Select all that apply.

a) Ice cream
b) Bananas
c) Chocolate
d) Onions

A

a) Ice cream, c) Chocolate

Explanation:
The nurse should promote avoidance of foods associated with flare-up of acne including chocolate, cola, fried foods, and milk products

191
Q

The nurse is preparing to perform chest physiotherapy (CPT) on a patient. Which of the following patient statements would indicate the procedure is contraindicated.

a) “I just finished eating my lunch, I’m ready for my CPT now.”
b) “I received my pain medication 10 minutes ago, let’s do my CPT now.”
c) “I just changed into my running suit; we can do my CPT now.”
d) “I have been coughing all morning and am barely bringing anything up.”

A

a) “I just finished eating my lunch, I’m ready for my CPT now.”

Explanation:
When performing CPT, the nurse ensures that the patient is comfortable, is not wearing restrictive clothing, and has not just eaten. The nurse gives medication for pain, as prescribed, before percussion and vibration and splints any incision and provides pillows for support, as needed. A goal of CPT is for the patient to be able to mobilize secretions; the patient who is having an unproductive cough is a candidate for CPT.

192
Q

A patient diagnosed with acute respiratory distress syndrome (ARDS) is restless and has a low oxygen saturation level. If the patient’s condition does not improve and the oxygen saturation level continues to decrease what procedure will the nurse expect to assist with in order to assist the patient to breathe easier?

a) Increase oxygen administration
b) Administer a large dose of furosemide (Lasix) IVP stat
c) Schedule the patient for pulmonary surgery
d) Intubate the patient and control breathing with mechanical ventilation

A

d) Intubate the patient and control breathing with mechanical ventilation

Explanation:
A patient with ARDS may need mechanical ventilation to assist with breathing while the underlying cause of the pulmonary edema can be corrected. The other options are not appropriate.

194
Q

The nurse is caring for a patient newly diagnosed with coronary artery disease (CAD). While developing a teaching plan for the patient to address modifiable risk factors for CAD, the nurse will include which of the following? Select all that apply.

a) Obesity
b) Decreased LDL level
c) Alcohol use
d) Drug use
e) Elevated blood pressure

A

a) Obesity, e) Elevated blood pressure
Explanation:
Hypertension, obesity, hyperlipidemia, tobacco use, diabetes mellitus, metabolic syndrome, and physical inactivity are modifiable risk factor for CAD. Alcohol and drug use are not included in the list of modifiable risk factors for CAD.

195
Q

An obese patient is undergoing abdominal surgery. A surgical resident states, “The amount of fat we have to cut through is disgusting” during the procedure. What is the best response by the nurse?

a) Report the resident to the attending surgeon.
b) Ignore the comment.
c) Discuss concerns regarding the comments with the charge nurse.
d) Inform the resident that all communication needs to remain professional.

A

d) Inform the resident that all communication needs to remain professional.
Explanation:
The nurse must advocate for the patient, especially when the patient cannot speak for themselves. By informing the resident that all communication needs to be professional, the nurse is addressing the comment at that moment in time, advocating for the patient. Ignoring the comment is not appropriate. The nurse may need to address the concerns of unprofessional communication with the attending surgeon or the charge nurse if the behavior continues. The best action is to address the behavior when it is happening.

196
Q

Which of the following ventilation–perfusion ratios is exhibited when a patient is diagnosed with pulmonary emboli?

a) Silent unit
b) Low ventilation–perfusion ratio
c) Dead space
d) Normal perfusion to ventilation ratio

A

c) Dead space

Explanation:
When ventilation exceeds perfusion a dead space exists (high ventilation–perfusion ratios). An example of a dead space is pulmonary emboli, pulmonary infarction, and cardiogenic shock. A low ventilation–perfusion ratio exists in pneumonia or with a mucus plug. A silent unit occurs in pneumothorax or ARDS.

198
Q

The nurse is caring for a 56-year-old male patient who had an implantable cardioverter defibrillator (ICD) implanted 2 days prior. The patient tells the nurse “My wife and I can never have sex again now that I have this ICD.” The nurse’s best response is which of the following?

a) “Sex is permitted following the implantation of an ICD.”
b) “You really should speak to your wife about your concerns.”
c) “I will be sure to share your concerns with the physician.”
d) “You seem apprehensive about resuming sexual activity.”

A

d) “You seem apprehensive about resuming sexual activity.”
Explanation:
The patient treated with an electronic device experiences not only lifestyle and physical changes but also emotional changes. At different times during the healing process, the patient may feel angry, depressed, fearful, anxious, or a combination of these emotions. It is imperative for the nurse to observe the patient’s response to the device and provide the patient and family members with emotional support and teaching as indicated. Identifying that the patient appears apprehensive about resuming sexual activity acknowledges the patient’s concerns while allowing for further discussion. The remaining responses ignore the patient’s feelings and do not facilitate an ongoing conversation or explore the patient’s concern.

199
Q

A college student presents to the health clinical with signs and symptoms of viral rhinitis (common cold). The patient states, “I’ve felt terrible all week; what can I do to feel better?” Which of the following is the best response the nurse can give?

a) “Your symptoms should go away soon, just try to get some rest.”
b) “You should rest, increase your fluids, and take Ibuprofen.”
c) “Have you tried a topic nasal decongestant; they work well.”
d) “Antibiotics will be prescribed, which will make you feel better.”

A

b) “You should rest, increase your fluids, and take Ibuprofen.”

Explanation:
Management of viral rhinitis consists of symptomatic therapy that includes adequate fluid intake, rest, prevention of chilling, and use of expectorants as needed. Warm saltwater gargles soothe the sore throat, and nonsteroidal anti-inflammatory drugs (NSAIDs), such as aspirin or ibuprofen, relieve aches and pains. Antibiotics are not prescribed because they do not affect the virus causing the patient’s signs and symptoms. Topical nasal decongestants should be used with caution. The symptoms of viral rhinitis may last from 1 to 2 weeks.

200
Q

The nurse is caring for a patient who is scheduled for a lobectomy. Following the procedure, the nurse will plan care based on which of the following?

a) The patient will return to the nursing unit with two chest tubes.
b) The patient will require sedation until the chest tube (s) are removed.
c) The patient will return from surgery with no drainage tubes.
d) The patient will require mechanical ventilation following surgery.

A

a) The patient will return to the nursing unit with two chest tubes.

Explanation:
The nurse should plan for the patient to return to the nursing unit with two chest tubes intact. During a lobectomy, the lobe is removed, and the remaining lobes of the lung are re-expanded. Usually, two chest catheters are inserted for drainage. The upper tube is for air removal; the lower one is for fluid drainage. Sometimes, only one catheter is needed. The chest tube is connected to a chest drainage apparatus for several days.

202
Q

Which of the following teaching interventions should the nurse include in the plan of care for a patient with valvular heart disease who is experiencing pulmonary congestion?

a) Teaching patients to take nitroglycerin if shortness of breath develops
b) Teaching patients to report a weight gain of 3 pounds in 1 week
c) Teaching patients to rest and sleep in a chair or sit in bed with head elevated
d) Teaching patients to drink at least 2 L of fluid daily and monitor urine output

A

c) Teaching patients to rest and sleep in a chair or sit in bed with head elevated
Explanation:
Patients who experience symptoms of pulmonary congestion are advised to rest and sleep sitting in a chair or bed with the head elevated. In addition, the nurse educates the patient to take a daily weight and report gains of 3 pounds in 1 day or 5 pounds in 1 week to the primary provider. The nurse may assist the patient with planning activity and rest periods to achieve an acceptable lifestyle.

204
Q

The nurse is caring for a female patient following a tonsillectomy and adenoidectomy. Two hours following the procedure, the patient begins to vomit large amounts of dark blood in frequent intervals and is tachycardic and febrile. After notifying the surgeon, the nurse will do which of the following?

a) Prepare for a needle aspiration.
b) Stay with the patient and monitor her closely.
c) Prepare for a needle aspiration.
d) Obtain a light, mirror, gauze, curved hemostats.

A

d) Obtain a light, mirror, gauze, curved hemostats.

Explanation:
If the patient vomits large amounts of dark blood at frequent intervals, or if the pulse rate and temperature rise, or the patient becomes restless, the nurse notifies the surgeon immediately. The nurse should have the following items ready for examination of the surgical site for bleeding: a light, a mirror, gauze, curved hemostats, and a waste basin. It is not necessary for the nurse to stay at the patient’s bedside. Needle aspiration is a procedure considered for patients experiencing a peritonsillar abscess. Although oral suctioning may be needed at some point of care, it is not a priority at this time.

205
Q

A postoperative patient is being discharged home following minor surgery. The PACU nurse is reviewing discharge instructions with the patient and his or her spouse. What action by the nurse is appropriate? Select all that apply.

a) Educate on activity limitations.
b) Discuss wound care.
c) Have the patient sign his or her advance directive form.
d) Have the spouse review when to notify the physician.
e) Provide information on health promotion topics.

A

a) Educate on activity limitations., b) Discuss wound care., d) Have the spouse review when to notify the physician., e) Provide information on health promotion topics.
Explanation:
The nurse should provide education on activity limitations, wound care, and review complications that require notification to the physician. The nurse should also provide information regarding health promotion topics, such as, weight management and smoking cessation. The patient should not make any major decisions or sign any legal forms due to the effects of anesthesia.

207
Q

A pt hs been recieving 100% O2 therapy by way of nonrebreather mask for several days. Now the pt complains of tinging in the fingers and shortness of breath, is extremely restless, and describes pain beneath the breastbone. What should the nurse expect?

  1. Oxygen-induced hypoventilation
  2. Oxygen toxicity
  3. Oxygen-induced atelectasis
  4. Hypoxia
A

Oxygen toxicity

208
Q

A 55-year-old man newly diagnosed with hypertension returns to his physician’s office for a routine follow-up appointment after several months of treatment with Lopressor (metoprolol). During the nurse’s initial assessment the patient’s blood pressure (BP) is recorded as 180/90 mm Hg. The patient states he does not take his medication as prescribed. The best response by the nurse is which of the following?

a) “Be certain to discuss your noncompliance with your medication regimen with the physician.”
b) “Your hypertension must be treated with medications; you need to take your Lopressor every day.”
c) “It is very important for you to take your medication as prescribed, or you could experience a stroke.”
d) “The medication you were prescribed may cause sexual dysfunction; are you experiencing this side effect?”

A

d) “The medication you were prescribed may cause sexual dysfunction; are you experiencing this side effect?”
Explanation:
The nurse needs to understand why the patient is not taking his medication. Lopressor is a beta-blocker. All patients should be informed that beta-blockers might cause sexual dysfunction and that other medications are available if problems with sexual function occur. The other statements, although true, are nontherapeutic and would not elicit why the patient was not taking his medications as prescribed.

209
Q

The nurse is caring for a male patient diagnosed with rhinosinusitis. The physician has ordered the patient to receive four sprays of budesonide (Rhinocort) in each nostril every morning. The nurse informs the patient that a common side effect of this medication includes which of the following?

a) Watery eyes
b) Headache
c) Epistaxis
d) Arthralgia

A

c) Epistaxis

Explanation:
Common side effects of budesonide include epistaxis, pharyngitis, cough, nasal irritation, and bronchospasms.

210
Q

Which of the following diagnostic tests is used to confirm the diagnosis of maxillary and frontal sinusitis?

a) Sinus x-rays
b) Sinus aspirates
c) MRI
d) CT scan

A

b) Sinus aspirates

Explanation:
To confirm the diagnosis of maxillary and frontal sinusitis and identify the pathogen, sinus aspirates may be obtained. Flexible endoscopic culture techniques and swabbing of the sinuses have been used for this purpose. Sinus x-rays and CT scans may be obtained for patients with frontal headaches, in refractory cases, and if complications are suspected.

211
Q

Which type of ventilator has a pre-sent volume of air to be delivered with each inspiration?

a) Time cycled
b) Negative pressure
c) Pressure cycled
d) Volume cycled

A

d) Volume cycled

Explanation:
With volume-cycled ventilation, the volume of air to be delivered with each inspiration is present. Negative pressure ventilators exert a negative pressure on the external chest. Time-cycled ventilators terminate or control inspiration after a preset time. When the pressure-cycled ventilator cycles on, it delivers a flow of air (inspiration) until it reaches a present pressure, and then cycles off, and expiration occurs passively.

213
Q

The nurse recognizes that the treatment for a non-ST elevation myocardial infarction (NSTEMI) differs from that of a patient with a STEMI, in that a STEMI is more frequently treated with which of the following?

a) IV heparin
b) Percutaneous coronary intervention (PCI)
c) IV nitroglycerin
d) Thrombolytics

A

b) Percutaneous coronary intervention (PCI)
Explanation:
The patient with a STEMI is often taken directly to the cardiac catheterization laboratory for an immediate PCI. Superior outcomes have been reported with the use of PCI compared to thrombolytics. IV heparin and IV nitroglycerin are used to treat NSTEMI.

215
Q

Officially, hypertension is diagnosed when the patient demonstrates a systolic blood pressure greater than ______ mm Hg and a diastolic blood pressure greater than _____ mm Hg over a sustained period.

a) 110, 60
b) 130, 80
c) 120, 70
d) 140, 90

A

d) 140, 90
Explanation:
According to the categories of blood pressure levels established by the Joint National Committee (JNC) VI, stage 1 hypertension is demonstrated by a systolic pressure of 140 to 159, or a diastolic pressure of 90 to 99. Pressure of 130 systolic and 80 diastolic falls within the normal range for an adult. Pressure of 110 systolic and 60 diastolic falls within the normal range for an adult. Pressure of 120 systolic and 70 diastolic falls within the normal range for an adult.

216
Q

Which of the following actions would place a client at the greatest risk for a shearing force injury to the skin?

  1. Walking without shoes
  2. Sitting in Fowler’s position
  3. Lying supine in bed
  4. Using a heating pad
A
  1. Sitting in Fowler’s position; None of the other movements or situations creates the combination of friction and pressure with downward movement seen in bedridden clients positioned in Fowler’s position.
217
Q

The nurse is providing discharge instructions for a patient following laryngeal surgery. The nurse instructs the patient to avoid which of the following?

a) Wearing a scarf over the stoma
b) Coughing
c) Wearing a plastic bib while showering
d) Swimming

A

d) Swimming

Explanation:
Swimming is not recommended because a person with a laryngectomy can drown without submerging his or her face. Special precautions are needed in the shower to prevent water from entering the stoma. Wearing a loose-fitting plastic bib over the tracheostomy or simply holding a hand over the opening is effective. The nurse also suggests that the patient wear a scarf over the stoma to make the opening less obvious. The nurse encourages the patient to cough every 2 hours to promote effective gas exchange.

218
Q

The purposes of the nursing history for the patient
who is to have surgery include all of the following
except:
1. Deciding whether surgery is indicated
2. Identifying the patient’s perception and
expectations about surgery
3. Obtaining information about the patient’s past
experience with surgery
4. Understanding the impact surgery has on the
patient’s and family’s emotional health

A

Deciding whether surgery is indicated

That is a medical decision and the responsibility
of the provider

220
Q

The nurse is attempting to ambulate a patient who underwent shoulder surgery earlier in the day. The patient is refusing to ambulate. What action by the nurse is most appropriate?

a) Use multiple staff members to remove the patient from the bed.
b) Delegate the task to the unlicensed assistive personnel.
c) Reinforce the importance of early mobility in preventing complications.
d) Document the patient’s refusal.

A

c) Reinforce the importance of early mobility in preventing complications.

Explanation:
The patient may be refusing to ambulate because of fear or pain. Educating on the importance of mobility in preventing complications may encourage the patient to ambulate. The nurse should try all reasonable measures (pain control, education) before documenting the patient’s refusal to ambulate. If the patient is already refusing to ambulate delegating the task to the unlicensed assistive personnel is not an appropriate action. The patient should not be forcefully removed from the bed.

221
Q

During a preadmission assessment, for what diagnosis would the nurse to expect to find decreased tactile fremitus and hyperresonant percussion sounds?

  1. Bronchitis
  2. Emphysema
  3. Atelectasis
  4. Pulmonary edema
A

Emphysema

222
Q

The nurse is assessing an African American patient and notes a streak of pigmentation in the patient’s fingernails. The nurse determines that this finding indicates which of the following?

a) Chronic anemia
b) Normal variation
c) Smoker’s fingernails
d) Melanoma

A

b) Normal variation

Explanation:
Variation in skin pigment levels in patients with dark skin, such as pigmented streaks in the nails, pigmented spots on the sclera of the eye, and pigmented crease across the bridge of the nose, are considered normal color variations.

223
Q

Which statement, if made by the client or family member, would indicate the need for further teaching?

  1. If a skin area gets red but then the red goes away after turning, I should report it to the nurse.
  2. Putting foam pads under the heels or other bony areas can help decrease pressure.
  3. If a person cannot turn himself in bed, someone should help them change position q4h.
  4. The skin should be washed with only warm water (not hot) and lotion put on while it is still a little wet.
A
  1. If a person cannot turn himself in bed, someone should help them change position q4h; Immobile and dependent persons should be repositioned at least every 2 hours, not every 4, so this client or family member requires additional teaching.

Warm water and moisturizing damp skin are correct techniques for skin care. Red areas that do not return to normal skin color should be reported. It would also be correct to use a foam pad to help relieve pressure.

224
Q

A confused patient who fell out of bed because side
rails were not used is an example of which type of
liability?
1. Felony
2. Battery
3. Assault
4. Negligence

A
  1. Negligence
  2. Conduct that falls below the standards of care
226
Q

A 66-year-old client presents to the emergency room (ER) complaining of a severe headache and mild nausea for the last 6 hours. Upon assessment, the patient’s BP is 210/120 mm Hg. The patient has a history of HTN for which he takes 1.0 mg clonidine (Catapres) twice daily for. Which of the following questions is most important for the nurse to ask the patient next?

a) “Are you having chest pain or shortness of breath?”
b) “Have you taken your prescribed Catapres today?”
c) “Do you have a dry mouth or nasal congestion?”
d) ”Did you take any medication for your headache?”

A

b) “Have you taken your prescribed Catapres today?”
Explanation:
The nurse must ask if the patient has taken his prescribed Catapres. Patients need to be informed that rebound hypertension can occur if antihypertensive medications are suddenly stopped. Specifically, a side effect of Catapres is rebound or withdrawal hypertension. Although the other questions may be asked, it is most important to inquire if the patient has taken his prescribed HTN medication given the patient’s severely elevated BP.

228
Q

The nurse answers the call light of a male patient. The patient is complaining of an irritating tickling sensation in the throat, a salty taste, and a burning sensation in the chest. Upon further assessment, the nurse notes a tissue with bright red, frothy blood at the bedside. The nurse can assume the source of the blood is likely from which of the following?

a) The stomach
b) The rectum
c) The lungs
d) The nose

A

c) The lungs

Explanation:
Blood from the lung is usually bright red, frothy, and mixed with sputum. Initial symptoms include a tickling sensation in the throat, a salty taste, a burning or bubbling sensation in the chest, and perhaps chest pain, in which case the patient tends to splint the bleeding side. This blood has an alkaline pH (>7.0). Blood from the stomach is vomited rather than expectorated, may be mixed with food, and is usually much darker; often referred to as “coffee ground emesis.” This blood has an acid pH (

229
Q

A client’s wound is draining thick yellow material. The nurse correctly describes the drainage as:

  1. Sanguineous
  2. Serous-sanguineous
  3. Serous
  4. Purulent
A
  1. Purulent; Drainage is described as purulent. Sanguineous and Serous-sanguineous contain blood. Serous is clear and watery.
230
Q

A patient is postoperative day 1 from abdominal surgery. The patient is receiving 0.9% normal saline at 75 mL/h, has a nasogastric tube to low wall suction with 200 mL every 8 hours of light yellow fluid, and a wound drain with 50 mL of dark red drainage every 8 hours. The 24-hour urine output total 2430 mL. What action by the nurse is most appropriate?

a) Assess for edema.
b) Document the findings and reassess in 24 hours.
c) Discontinue the nasogastric tube suctioning.
d) Assess for signs and symptoms of fluid volume deficit.

A

d) Assess for signs and symptoms of fluid volume deficit.
Explanation:
The patient’s 24 hour intake is 1,800 mL (75x24). The patient’s 24 hour output is 3180 mL [(200 × 3) + (50 × 3) + 2430]. Since the output is significantly higher than the intake the patient is at risk for fluid volume deficit. The nurse should not discontinue the nasogastric suctioning without a physician’s order. The findings should be documented and reassessed but the nurse needs to take more action to prevent complication. Edema is usually associated with fluid volume excess.

232
Q

A patient is being assessed for acute laryngitis. The nurse recognizes that clinical manifestations of acute laryngitis include which of the following?

a) A nonedematous uvula
b) A throat that feels worse in the evening
c) Hoarseness
d) A moist cough

A

c) Hoarseness

Explanation:
Signs of acute laryngitis include hoarseness or aphonia and severe cough. Other signs of acute laryngitis include a dry cough, and a throat that feels worse in the morning. If allergies are present, the uvula will be visibly edematous.

233
Q

The nurse is providing discharge instructions to a patient with pulmonary sarcoidosis. The nurse concludes that the patient understands the information if the patient correctly states which of the following early signs of exacerbation?

a) Weight loss
b) Shortness of breath
c) Fever
d) Headache

A

b) Shortness of breath

Explanation:
Early signs and symptoms of pulmonary sarcoidosis may include dyspnea, cough, hemoptysis, and congestion. Generalized symptoms include anorexia, fatigue, and weight loss.

235
Q

A new scrub technician is being orientated to the operating room. The scrub technician states to the nurse, “You can skip the fire safety information because I have worked in hospitals for the last 10 years.” What is the best response by the nurse?

a) “The operating room has some unique circumstances that increases the chances of fire.”
b) “I know this information is not exciting but I’m required to cover this information with you.”
c) “OK, but you will be required to review the hospital’s policy on fire safety on your own.”
d) “This is a requirement of your job, just tough through it.”

A

a) “The operating room has some unique circumstances that increases the chances of fire.”
Explanation:
The operating room environment has some unique characteristics that do increase the chance of fires, such as drapes that allow oxygen concentration. By engaging the new employee to understand the underlying reason for fire safety in the operating room, the new employee will develop a greater understanding and appreciation for fire safety. If fire safety is only presented as a requirement for the job then the employee may not understand the importance of fire safety. The hospital’s policy for fire safety is broad; the employee would need to review the fire safety policies specifically for the operating room

236
Q

A nurse is witnessing a patient sign the consent form for surgery. After the patient signs the consent form, the patient starts asking questions regarding the risks and benefits of a surgical procedure. What action by the nurse is most appropriate?

a) Place the consent form in the patient’s medical record.
b) Notify the nurse manager of the patient’s questions.
c) Answer the patient’s questions.
d) Request that the surgeon come and answer the questions.

A

d) Request that the surgeon come and answer the questions.
Explanation:
It is the physician’s responsibility to provide information pertaining to risks and benefits of surgery. It is not the responsibility of the nurse or nurse manager to discuss risks and benefits. The consent form should not be placed in the medical record until all questions are answered fully for the patient.

237
Q

A patient is scheduled for an invasive procedure. What is the priority documentation needed regarding the procedure?

a) The medication reconciliation form
b) A signed consent form from the patient
c) A health history obtained by the primary physician
d) Prescriptions for postoperative medications

A

b) A signed consent form from the patient
Explanation:
A signed consent is required and is important for initiating invasive procedures. The nurse should therefore check for the patient’s signed consent form. A health history, medication reconciliation, and postoperative prescriptions are good items to have, but are not required documentation before performing an invasive procedure.

238
Q

Advanced practice registered nurses generally:

A. Function independently

B. Function as unit directors

C. Work in acute care settings

D. Work in the university setting

A

A. Function independently

Advanced practice registered nurse functions independently as a clinician, educator, case manager, consultant, and researcher within his or her area of practice to plan or improve the quality of nursing care for the patient and family.

239
Q

A postoperative patient, with an open abdominal wound is currently taking corticosteroids. The physician orders a wound culture of the abdominal wound even though there are no signs and symptoms of infection. What action by the nurse is appropriate?

a) Request the order be discontinued without obtaining the specimen.
b) Hold the order until purulent drainage is noted.
c) Obtain the wound culture specimen.
d) Use an antibiotic cleaning agent before obtaining the specimen.

A

c) Obtain the wound culture specimen.
Explanation:
Corticosteroids may mask the presence of infection by impairing the normal inflammatory response. The culture should be obtained even though the patient is not demonstrating traditional signs and symptoms of infection. The order should not be discontinued or held until purulent drainage is noted because the infection could worsen and the patient will possibly develop sepsis. An antibiotic cleaning agent should not be used before obtaining the specimen because it will alter the growth of the organisms.

240
Q

The nurse is conducting an admission history and physical examination of a patient with a history of contact dermatitis. The nurse assesses if the patient utilizes which of the following medication classifications?

a) Saline irrigations
b) Antivirals
c) Corticosteroids
d) Antifungals

A

c) Corticosteroids

Explanation:
Corticosteroids are utilized for contact dermatitis. Antifungals, antivirals, and saline irrigations are not utilized in the treatment of contact dermatitis

242
Q

When assessing an African American patient for cyanosis, the nurse should evaluate which of the following areas?

a) Fingernails
b) Oral mucosa
c) Nose
d) Sclera

A

b) Oral mucosa

Explanation:
In a person with dark skin, the skin usually assumes a grayish cast. To detect cyanosis, observe oral muscosa, conjuctivae, and nail beds

243
Q

The herpes simplex virus type 1 (HSV-1), which produces a cold sore (fever blister), has an incubation period of which of the following?

a) 2 to 12 days
b) 3 to 6 months
c) 20 to 30 days
d) 0 to 3 months

A

a) 2 to 12 days

Explanation:
HSV-1 is transmitted primarily by direct contact with infected secretions. The incubation period is about 2 to 12 days. The time period 0 to 3 months exceeds the incubation period. The time period 20 to 30 days exceeds the incubation period. The time period 3 to 6 months exceeds the incubation period.

244
Q

The nurse is conducting a community education program on basal cell carcinoma (BCC). Which of the following statements should the nurse make?

a) It is a malignant proliferation arising from the epidermis.
b) It begins as a small, waxy nodule with rolled translucent, pearly borders.
c) It metastasizes by blood or lymphatic system.
d) It is more invasive than squamous cell carcinoma (SCC).

A

b) It begins as a small, waxy nodule with rolled translucent, pearly borders.

Explanation:
BCC usually begins as a small, waxy nodule with rolled, translucent, pearly borders. It is less invasive than SCC. It does not metastasize by the blood or lymphatic system. SCC is a malignant proliferation arising from the epidermis.

244
Q

The nurse is assisting with the collection of a Tzanck smear. What is the suspected diagnosis of the pt?

  1. Fungal infection
  2. Herpes Zoster
  3. Psoriasis
  4. Seborrheic dermatosis
A

Herpes Zoster

246
Q

A critical care nurse is using a computerized decision support system to correctly position her ventilated patients to reduce pneumonia caused by accumulated respiratory secretions. This is an example of which Quality and Safety in the Education of Nurses (QSEN) competency?

A. Patient-centered care

B. Safety

C. Teamwork and collaboration

D. Informatics

A

D. Informatics

Using decision support systems is one example of using and gaining competency in informatics.

248
Q

The nurse is caring for a patient with extensive respiratory disease. Which of the following is a late sign of hypoxia in the patient?

a) Hypotension
b) Restlessness
c) Somnolence
d) Cyanosis
Show Answer

A

d) Cyanosis

Explanation:
Cyanosis is a late sign of hypoxia. Hypoxia may cause restlessness and an initial rise in blood pressure that is followed by hypotension and somnolence.

249
Q

A pt is being treated for chronic venous stasis ulcers of the lower extremities. What medication does the nurse understand will increase peripheral blood flow by decreasing the viscosity of the blood and assist with the healing of ulcers?

  1. Heparin
  2. Warfarin
  3. Aspirin
  4. Pentoxifylline (Trental)
A

Pentoxifylline

251
Q

A patient involved in a motor vehicle crash suffered a blunt injury to the chest wall and was brought to the emergency department. The nurse assesses the patient for which clinical manifestation that would indicate the presence of a pneumothorax?

a) Sucking sound at the site of injury
b) Decreased respiratory rate
c) Bloody, productive cough
d) Diminished breath sounds

A

a) Sucking sound at the site of injury

.Explanation:

Open pneumothorax is one form of traumatic pneumothorax. It occurs when a wound in the chest wall is large enough to allow air to pass freely in and out of the thoracic cavity with each attempted respiration. Because the rush of air through the wound in the chest wall produces a sucking sound, such injuries are termed sucking chest wounds

252
Q

A middle-aged male presents to the ED complaining of severe chest discomfort. Which of the following patient findings is most indicative of a possible MI?

a) Intermittent nausea and emesis for 3 days
b) Cool, clammy, diaphoretic, and pale appearance
c) Chest discomfort not relieved by rest or nitroglycerin
d) Anxiousness, restlessness, and lightheadedness

A

c) Chest discomfort not relieved by rest or nitroglycerin
Explanation:
Chest pain or discomfort not relieved by rest or nitroglycerin is associated with an acute MI. The other findings, although associated with ACS (acute coronary syndrome) or MI, may also occur with angina and, alone, are not indicative of an MI.

254
Q

The nurse is caring for a patient with an endotracheal tube (ET). Which of the following nursing interventions is contraindicated?

a) Ensuring that humidified oxygen is always introduced through the tube
b) Deflating the cuff prior to tube removal
c) Deflating the cuff routinely
d) Checking the cuff pressure every 6 to 8 hours

A

c) Deflating the cuff routinely

Explanation:
Routine cuff deflation is not recommended because of the increased risk for aspiration and hypoxia. The cuff is deflated before the ET is removed. Cuff pressures should be checked every 6 to 8 hours. Humidified oxygen should always be introduced through the tube.

255
Q

Crohn’s disease is a condition of malabsorption caused by which of the following pathophysiological processes?

a) Gastric resection
b) Disaccharidase deficiency
c) Inflammation of all layers of intestinal mucosa
d) Infectious disease

A

c) Inflammation of all layers of intestinal mucosa
Explanation:
Crohn’s disease, also known as regional enteritis, can occur anywhere along the GI tract, but most commonly at the distal ileum and in the colon. Infectious disease causes problems such as small bowel bacterial overgrowth leading to malabsorption. Disaccharidase deficiency leads to lactose intolerance. Postoperative malabsorption occurs after gastric or intestinal resection.

257
Q

The nurse is discussing immediate postoperative communication strategies with a patient scheduled for a total laryngectomy. Which of the following information will the nurse include?

a) “After surgery, you will have to use an electric larynx to communicate.”
b) “After surgery you will have a sore throat, but will be able to speak.”
c) “A speech therapist will evaluate you and recommend a system of communication after surgery.”
d) “You can use writing or a communication board to communicate.”

A

d) “You can use writing or a communication board to communicate.”

Explanation:
If a total laryngectomy is scheduled, the patient must understand that the natural voice will be lost, but that special training can provide a means for communicating. The patient needs to know that until training is started, communication will be possible by using the call light, by writing, or by using a special communication board. The use of an electronic device is a long-term postoperative goal. The speech therapist will evaluate the patient prior to surgery and a method of immediate postoperative communication will be established.

258
Q

A parent of a 16-year-old patient asks the nurse, “How could the surgeon operate without my consent?” What is the best response given by the nurse?

a) “The surgical procedure being performed does not require consent.”
b) “Two doctors decided your child needed the surgery, therefore we did not need to get consent.”
c) “Your child had life-threatening injuries that required immediate surgery.”
d) “We obtained consent from your child after your child requested the surgery.”

A

b) “Your child had life-threatening injuries that required immediate surgery.”
Explanation:
In an emergency, it may be necessary for the surgeon to operate as a lifesaving measure without the patient’s or parent’s informed consent. Informed consent must be obtained before any invasive procedure. A minor cannot consent for a surgical procedure. Two doctors’ opinions do not overrule the need to obtain informed consent.

260
Q

A nurse is preparing instructions for a patient with a lung abscess regarding dietary recommendations. Which of the following statements would be included in the plan of care?

a) “You must consume a diet low in calories, such as skim milk, fresh fruits, and vegetables.”
b) “You must consume a diet high in carbohydrates, such as bread, potatoes, and pasta.”
c) “You must consume a diet low in fat by limiting dairy products and concentrated sweets.”
d) “You must consume a diet rich in protein, such as chicken, fish, and beans.”

A

d) “You must consume a diet rich in protein, such as chicken, fish, and beans.”

Explanation:
For a patient with a lung abscess the nurse encourages a diet that is high in protein and calories to ensure proper nutritional intake. A carbohydrate-dense diet or diets with limited fats are not advisable for a patient with a lung abscess.

262
Q

Which of the following terms refers to a condition characterized by destruction of the melanocytes in circumscribed areas of the skin?

a) Lichenification
b) Telangiectases
c) Vitiligo
d) Hirsutism

A

c) Vitiligo

Explanation:
Vitiligo results in the development of white patches that may be localized or widespread. Hirsutism is the condition of having excessive hair growth. Lichenification refers to a leathery thickening of the skin. Telangiectases refer to red marks on the skin caused by stretching of the superficial blood vessels.

263
Q

When teaching a patient with rheumatic carditis and a history of recurrent rheumatic fever, which of the following statements made by the patient indicates that teaching has been successful?

a) “I may have to take prophylactic antibiotics for up to 10 years.”
b) “I will avoid milk, yogurt and other dairy products.”
c) “I will avoid any kind of activity.”
d) “I will take nonsteroidal anti-inflammatory medication (NSAIDs) every day.”

A

a) “I may have to take prophylactic antibiotics for up to 10 years.”
Explanation:
Antibiotic prophylaxis for recurrent rheumatic fever with rheumatic carditis may require 10 or more years of antibiotic coverage (e.g., penicillin G intramuscularly (IM) every 4 weeks, penicillin V orally twice a day (BID), sulfadiazine orally daily, or erythromycin orally BID. Patients with a history of rheumatic fever are susceptible to infective endocarditis and should be asked to take prophylactic antibiotics before any invasive procedure, including dental work. Steroids are prescribed to suppress the inflammatory response and aspirin to control the formation of blood clots around heart valves. Activities that require minimal activity are recommended to reduce the work of the myocardium and counteract the boredom of weeks of bed rest.

264
Q

The nurse is instructing the parents of a child with head lice. Which of the following statements should the nurse include?

a) Shampoo with piperonyl butoxide (RID).
b) Wash clothes in cold water.
c) Shampoo with Kwell.
d) Disinfect brushes and combs with bleach.

A

a) Shampoo with piperonyl butoxide (RID).

Explanation:
The nurse’s instructions should include shampooing with RID, washing clothes in hot water, and disinfecting brushes and combs with RID shampoo

265
Q

Which of the following is a true statement regarding air pressure variances?

a) Air is drawn through the trachea and bronchi into the alveoli during inspiration.
b) Air flows from a region of lower pressure to a region of higher pressure during inspiration.
c) The thoracic cavity becomes smaller during inspiration.
d) The diaphragm contracts during inspiration.

A

a) Air is drawn through the trachea and bronchi into the alveoli during inspiration.

Explanation:
During inspiration, movements of the diaphragm and intercostal muscles enlarge the thoracic cavity, thereby lowering the pressure inside the thorax to a level below that of atmospheric pressure. As a result, air is drawn through the trachea and bronchi into the alveoli.

266
Q

A nurse is teaching a patient who is awaiting a heart transplant. Which of the following statements indicate the patient understands what is required to help minimize rejection?

a) “There is no risk of rejection if the donor heart is an exact match.”
b) “I will receive medication before and during surgery which will eliminate the risk of rejection.”
c) “I will need medication following surgery to prevent rejection and if my body does not reject the new heart I will not have to take any medication at home.”
d) “I will need to take three different types of medications for the rest of my life to help prevent rejection.”

A

d) “I will need to take three different types of medications for the rest of my life to help prevent rejection.”
Explanation:
Patients who have had heart transplants are constantly balancing the risk of rejection with the risk of infection. They must adhere to a complex regimen of diet, medications, activity, follow-up laboratory studies, biopsies of the transplanted heart (to diagnose rejection), and clinic visits. There are three classes of medications that are prescribed for a transplant patient to help minimize rejection: corticosteroids (e.g., prednisone), calcineurin inhibitors (tacrolimus, cyclosporin), and antiproliferative agents (mycophenolate mofetil [CellCept], azathioprine [Imuran], or sirolimus [Rapamune]).

267
Q

The nurse is observing a patient during an exercise stress test (bicycle). Which of the following findings indicates a positive test and the need for further diagnostic testing?

a) BP changes; 148/80 mm Hg to 166/90 mm Hg
b) Dizziness and leg cramping
c) Heart rate changes; 78 bpm to 112 bpm
d) ST-segment changes on the ECG

A

d) ST-segment changes on the ECG
Explanation:
During the test, the following are monitored: two or more ECG leads for heart rate, rhythm, and ischemic changes; BP; skin temperature; physical appearance; perceived exertion; and symptoms, including chest pain, dyspnea, dizziness, leg cramping, and fatigue. The test is terminated when the target heart rate is achieved or if the patient experiences signs of myocardial ischemia. Further diagnostic testing, such as a cardiac catheterization, may be warranted if the patient develops chest pain, extreme fatigue, a decrease in BP or pulse rate, serious dysrhythmias or ST-segment changes on the ECG during the stress test. The other findings would not warrant the testing to be stopped.

268
Q

A nurse is caring for patients with a variety of wounds. Which would will most likely heal by primary intention?

  1. Cut in the skin from a kitchen knife
  2. Excoriated perineal area
  3. Abrasion of the skin
  4. Pressure ulcer
A
  1. Cut in the skin from a kitchen knife; A cut in the skin by a sharp instrument with minimal tissue loss can heal by primary intention when the wound edges are lightly pulled together (approximated).

Excoriations, abrasions, and pressure ulcers heal by secondary, not primary. Secondary intention healing occurs when wound edges are not approximated because of full-thickness tissue loss; the wound is left open until it fills with new tissue. Abrasions and excoriations are injuries to the surface of the skin.

269
Q

Graduate nurses must pass a licensure examination
administered by the:
1. State Boards of Nursing
2. National League for Nursing
3. Accredited school of nursing
4. American Nurses Association

A

State Boards of Nursing

270
Q

An emergency room nurse is assessing a patient who is complaining of dyspnea. Which of these signs would indicate the presence of a pleural effusion?

a) Mottling of the skin upon inspection
b) Resonance upon percussion
c) Wheezing upon auscultation
d) Decreased chest wall excursion upon palpation

A

d) Decreased chest wall excursion upon palpation

Explanation:
Symptoms of pleural effusion are shortness of breath, pain, assumption of a position that decreases pain, absent breath sounds, decreased fremitus, a dull, flat sound on percussion, and decreased chest wall excursion. The nurse may also hear a friction rub. Chest radiography and computed tomography (CT) scan show fluid in the involved area.

271
Q

Nurses at a community hospital are in an education program to learn how to use a new pressure-relieving device for patients at risk for pressure ulcers. This is which type of education?

A. Continuing education

B. Graduate education

C. In-service education

D. Professional Registered Nurse Education

A

C. In-service education

In-service education programs are instruction or training provided by a health care agency or institution. An in-service program is held in the institution and is designed to increase the knowledge, skills, and competencies of nurses and other health care professionals employed by the institution.

272
Q

A client has a diabetic stasis ulcer on the lower leg. The nurse uses a hydrocolloid dressing to cover it. The procedure for application includes:

  1. Cleaning the skin and wound with betadine
  2. Removing all traces of residues for the old dressing
  3. Choosing a dressing no more than quarter-inch larger than the wound size
  4. Holding in place for one minute to allow it to adhere
A
  1. Holding in place for one minute to allow it to adhere; The skin is cleansed with normal saline or a mild cleanser. Residue of old dressings will dissolve. The dressing size is to be 3-4 cm (1.5 inches) larger than the size of the wound.
273
Q

What physiological conditions are contraindicated for using heat as a therapy? (Select all that apply.)

  1. The first 24 hours of injury
  2. Active hemorrhage
  3. Noninflammatory edema
  4. Localized malignant tumor
A

All of the above; Heat causes vasodilatation and increases blood flow to the affected area bringing oxygen, nutrients, antibodies, and leukocytes. A possible disadvantage of heat is that it increases capillary permeability, which allows extracellular fluid and substances to pass through the capillary walls and may result in edema or an increase in preexisting edema.
Contraindications include: the first 24 hours of injury, active hemorrhage, noninflammatory edema, localized malignant tumor, and skin disorder that causes redness or blisters.

275
Q

The nurse notes that the patient demonstrates generalized pallor and recognizes that this finding may be indicative of

a) Albinism
b) Anemia
c) Vitiligo
d) Local arterial insufficiency

A

b) Anemia

Explanation:
In the light-skinned individual, generalized pallor is a manifestation of anemia. In brown- and black-skinned individuals, anemia is demonstrated as a dull skin appearance. Albinism is a condition of total absence of pigment in which the skin appears whitish pink. Vitiligo is a condition characterized by the destruction of the melanocytes in circumscribed areas of skin, resulting in patchy, milky white spots. Local arterial insufficiency is characterized by marked localized pallor.

276
Q

Which of the following is the principal hardening ingredient of the hair and nails?

a) Merkel cell
b) Melanin
c) Sebaceous gland
d) Keratin

A

d) Keratin

Explanation:
Keratin is the principal hardening ingredient of the hair and nails. Melanin is the substance responsible for the coloration of the skin. A Merkel cell is located in the epidermis and plays a role in transmission of sensory message. A sebaceous gland secretes sebum to keep the skin soft and pliable.

278
Q

Which of the following is the most reliable and accurate method for delivering precise concentrations of oxygen through noninvasive means?

a) T-piece
b) Partial-rebreathing mask
c) Venturi mask
d) Nasal cannula

A

c) Venturi mask

Explanation:
The Venturi mask is the most reliable and accurate method for delivering a precise concentration of oxygen through noninvasive means. The mask is constructed in a way that allows a constant flow of room air blended with a fixed flow of oxygen. Nasal cannula, T-piece, and partial-rebreathing masks are not the most reliable and accurate methods of oxygen administration.

279
Q

The nurse is assessing the fingernails of a pt at the clinic. The nurse observes the pitting on the surface of the nail. What disorder is this finding indicative of?

  1. Psoriasis
  2. Vitiligo
  3. Diabetes
  4. Melanoma
A

Psoriasis

280
Q

A harsh grating sound caused by abrasion of the pericardial surfaces during the cardiac cycle is termed which of the following?

a) Ejection click
b) Murmur
c) Friction rub
d) Opening snap

A

c) Friction rub

Explanation:
In pericarditis, a harsh, grating sound that can be heard in both systole and diastole is called a friction rub. A murmur is created by the turbulent flow of blood. A cause of the turbulence may be a critically narrowed valve. An opening snap is caused by high pressure in the left atrium with abrupt displacement of a rigid mitral valve. An ejection click is caused by very high pressure within the ventricle, displacing a rigid and calcified aortic valve.

281
Q

An obese patient is scheduled for open abdominal surgery. What priority education should the nurse provide this patient?

a) Venous thromboembolism prevention
b) Prevention of respiratory complications
c) Prevention of wound dehiscence
d) Wound care and infection prevention

A

b) Prevention of respiratory complications
Explanation:
All answers are correct but the obese patient has an increased susceptibility to respiratory complications, and maintaining a patent airway would be the priority.

282
Q

Hypertension that can be attributed to an underlying cause is termed which of the following?

a) Secondary
b) Primary
c) Essential
d) Isolated systolic

A

a) Secondary
Explanation:
Secondary hypertension may be caused by a tumor of the adrenal gland (eg, pheochromocytoma). Primary hypertension has no known underlying cause. Essential hypertension has no known underlying cause. Isolated systolic hypertension is demonstrated by readings in which the systolic pressure exceeds 140 mm Hg and the diastolic measurement is normal or near normal (less than 90 mm Hg).

283
Q

In order to be effective, percutaneous transluminal coronary angioplasty (PTCA) must be performed within what time frame, beginning with arrival at the emergency department after diagnosis of myocardial infarction (MI)?

a) 60 minutes
b) 30 minutes
c) 9 days
d) 6 to 12 months

A

a) 60 minutes
Explanation:
The 60-minute interval is known as “door-to-balloon time” for performance of PTCA on a diagnosed MI patient. The 30-minute interval is known as “door-to-needle time” for administration of thrombolytics post MI. The time frame of 9 days refers to the time for onset of vasculitis after administration of streptokinase for thrombolysis in an acute MI patient. The 6- to 12-month time frame refers to the time period during which streptokinase will not be used again in the same patient for acute MI.

284
Q

The nurse is reviewing the laboratory results for a patient diagnosed with coronary artery disease (CAD). The patient’s low-density lipoprotein (LDL) level is 115 mg/dL. The nurse interprets this value as which of the following?

a) High
b) Critically high
c) Low
d) Within normal limits

A

a) High
Explanation:
The normal LDL range is 100 mg/dL to 130 mg/dL. A level of 115 mg/dL is considered to be high. The goal of treatment is to decrease the LDL level below 100 mg/dL (less than 70 mg/dL for very high-risk patients).

286
Q

The nurse is conducting a preoperative assessment on a patient scheduled for gallbladder surgery. The patient reports having a frequent cough producing green sputum for 3 days and denies fever. Upon auscultation, the nurse notes rhonchi throughout the right lung with an occasional expiratory wheeze. Respiratory rate is 20, temperature is 99.8 taken orally, heart rate is 87, and blood pressure is 124/70. What is the nurse’s best action?

a) Notify the surgeon to possibly delay the surgery.
b) Notify the primary physician about the assessment findings.
c) Document the findings and continue the patient through the preoperative phase.
d) Wait 1 hour and complete the assessment again.

A

a) Notify the surgeon to possibly delay the surgery.
Explanation:
A respiratory infection can delay a nonemergent surgical procedure because the infection can increase the risk for respiratory complications. Therefore, the nurse should notify the surgeon about delaying the surgery. The primary physician may be called to care for the assessment findings but that should be done only after the surgeon has been notified. Continuing through the preoperative phase without notifying the surgeon and waiting 1 hour is not appropriate.

287
Q

You are caring for an assigned client and notice a superficial ulcer on the client’s buttock that appears as a shallow crater involving the epidermis and the dermis. Which of the following stages would you say best describes this break in skin integrity?

  1. Stage I
  2. Stage II
  3. Stage III
  4. Stage IV
A
  1. Stage II; Stage I pressure ulcer involves a nonblanchable erythema of intact skin, while a stage II involves a partial-thickness skin loss involving epidermis, dermis, or both, with the ulcer being superficial and presenting as an abrasion, blister, or shallow crater.
288
Q

A scrub nurse is diagnosed with a skin infection to the right forearm. What is the priority action by the nurse?

a) Request role change to circulating nurse.
b) Return to work after being on antibiotics for 24 hours.
c) Report the infection to an immediate supervisor.
d) Ensure the infection is covered with a dressing.

A

c) Report the infection to an immediate supervisor.
Explanation:
The infection needs to be reported immediately because of the asepsis environment of the operating room. The usual barriers may not protect the patient when an infection is present. The employee will need to follow the policy of the operating room regarding infections. Covering the infections with a dressing may be necessary but the infection must first be reported. The scrub nurse may still be able to work depending on the policy; therefore, returning to work after 24 hours is not the priority action. Even if the nurse requests a role change to circulating nurse, the policy for infections in the operating room must be followed; therefore, it must first be reported.

289
Q

The nurse is teaching health promotion to a class on osteoporosis prevention. The nurse determines that the participants understand the teaching when they identify that the amount of sun exposure needed to synthesize sufficient vitamin D is which of the following?

a) 90 to 120 minutes twice a week
b) 5 to 30 minutes twice a week
c) 60 to 90 minutes weekly
d) 30 to 60 minutes weekly

A

b) 5 to 30 minutes twice a week

Explanation:
It is estimated that most people need 5 to 30 minutes of sun exposure twice a week to synthesize enough vitamin D to prevent osteoporosis

290
Q

The clinical finding of pink frothy sputum may be an indication of which of the following?

a) Pulmonary edema
b) An infection
c) A lung abscess
d) Bronchiectasis

A

a) Pulmonary edema

Explanation:
Profuse, frothy pink material, often welling up into the throat, may indicate pulmonary edema. Foul-smelling sputum and bad breath may indicate a lung abscess, bronchiectasis, or an infection caused by fusospirochetal or other anaerobic organisms.

291
Q

The nurse is caring for a patient who underwent a laryngectomy. Which of the following interventions will the nurse initially complete in an effort to meet the patient’s nutritional needs?

a) Encourage sweet foods.
b) Liberally season foods.
c) Initiate enteral feedings.
d) Offer plenty of thin liquids.

A

c) Initiate enteral feedings.

Explanation:
Postoperatively, the patient may not be permitted to eat or drink for at least 7 days. Alternative sources of nutrition and hydration include IV fluids, enteral feedings through a nasogastric or gastrostomy tube, and parenteral nutrition. Once the patient is permitted to resume oral feedings, thin liquids are offered, and sweet food are avoided because they cause increased salivation and decrease the patient’s appetite. The patient’s taste sensations are altered for a while after surgery because inhaled air passes directly into the trachea, bypassing the nose and the olfactory end organs. In time, however, the patient usually accommodates to this change and olfactory sensation adapts; thus, seasonings are based on personal preferences.

292
Q

A patient who has an altered level of consciousness is receiving tube feedings. Patients receiving tube feeding should be placed in which of the following positions?

a) Trendelenburg
b) Side-lying
c) Semi-Fowler’s or higher
d) Supine

A

c) Semi-Fowler’s or higher

Explanation:
Patients receiving tube feedings are positioned with the head of the bed at 30 degrees or higher during feedings and for 30 to 45 minutes after tube feedings. Patients receiving oral feedings are positioned with the head of the bed in an upright position for 30 to 45 minutes after feedings. For patients with a nasogastric or gastrostomy tube, the placement of the tube and residual gastric volume must be checked before each feeding.

293
Q

A patient’s orders include a wound dressing using an autolytic debriding agent. The nurse providing discharge instructions to the patient should include which of the following?

a) Keep the dressing very wet at all times.
b) Do not use a dressing 6 hr/day.
c) Cleanse the wound with Dakin’s solution.
d) The wound may have a foul odor.

A

d) The wound may have a foul odor.

Explanation:
During autolytic débridement therapy a foul odor will be produced by the breakdown of cellular debris. This odor does not indicate that the wound is infected. During autolytic débridement therapy the wound is kept moist at all times. The wound should be cleansed with normal saline.

294
Q

A pt is to recieve an oxygen concentration of 70%. What is the best way for the nurse to deliver this concentration?

  1. A nasal cannula
  2. A oropharyngeal catheter
  3. A partial rebreather mask
  4. A Venturi mask
A

A partial rebreather mask

295
Q

A patient is undergoing thoracic surgery. What priority education should the nurse provide to assist in preventing respiratory complications?

a) Splint the incision site using a pillow during deep breathing and coughing exercises.
b) Pain medication should be taken before completing deep breathing and coughing exercises.
c) Deep breathing and coughing exercises may be used as relaxation techniques.
d) Deep breathing and coughing exercises should be completed every 8 hours.

A

a) Splint the incision site using a pillow during deep breathing and coughing exercises.
Explanation:
Splinting the incision site will help decrease pain and support the incision. This will increase compliance with the deep breathing and coughing exercises that assist with the prevention of respiratory complications. Pain medication should be taken regularly and not only before deep breathing and coughing exercises. Deep breathing and coughing exercises should be done at least every 2 hours, more frequently if possible. While some patients will find the exercises relaxing, most patients find it painful to complete the exercises.

296
Q

The nurse correctly identifies which of the following data as an example of BP and HR measurements in a patient with postural hypotension?

a) Supine: BP 120/70 mm Hg, HR 70 bpm; sitting: BP 100/55 mm Hg, HR 90 bpm; standing: BP 98/52 mm Hg, HR 94 bpm
b) Supine: BP 130/70 mm Hg, HR 80 bpm; sitting: BP 128/70 mm Hg, HR 80 bpm; standing: BP 130/68 mm Hg, HR 82 bpm
c) Supine: BP 140/78 mm Hg, HR 72 bpm; sitting: BP 145/78 mm Hg, HR 74 bpm; standing: BP 144/78 mm Hg, HR 74 bpm
d) Supine: BP 114/82 mm Hg, HR 90 bpm; sitting: BP 110/76 mm Hg, HR 95 bpm; standing: BP 108/74 mm Hg, HR 98 bpm

A

a) Supine: BP 120/70 mm Hg, HR 70 bpm; sitting: BP 100/55 mm Hg, HR 90 bpm; standing: BP 98/52 mm Hg, HR 94 bpm
Explanation:
Postural (orthostatic) hypotension is a sustained decrease of at least 20 mm Hg in systolic BP or 10 mm Hg in diastolic BP within 3 minutes of moving from a lying or sitting to a standing position. The following is an example of BP and HR measurements in a patient with postural hypotension: supine: BP 120/70 mm Hg, HR 70 bpm; sitting: BP 100/55 mm Hg, HR 90 bpm; standing: BP 98/52 mm Hg, HR 94 bpm. Normal postural responses that occur when a person moves from a lying to a standing position include (1) a HR increase of 5 to 20 bpm above the resting rate; (2) an unchanged systolic pressure, or a slight decrease of up to 10 mm Hg; and (3) a slight increase of 5 mm Hg in diastolic pressure.

297
Q

A client has a pressure ulcer with a shallow, partial skin thickness, eroded area but no necrotic areas. The nurse would treat the area with which dressing?

  1. Alginate
  2. Dry Gauze
  3. Hydrocolloid
  4. No dressing indicated.
A
  1. Hydrocolloid; Hydrocolloid dressings protect shallow ulcers and maintain an appropriate healing environment.

Alginates (option 1) are used for wounds with significant drainage; dry gauze (option 2) will stick to granulation tissue, causing more damage. A dressing is needed to protect the wound and enhance healing.

298
Q

The nurse caring for a patient notes that the patient’s lower extremities are covered with very dry skin and that the horny layer of the skin has become thickened. The nurse notes the finding as being which of the following?

a) Lichenification
b) Acantholysis
c) Dermatitis
d) Pyodermas

A

a) Lichenification

Explanation:
The nurse should note this as being lichenification. Dermatitis is an inflammation of the skin. Acantholysis is a separation of the epidermal cells from each other, and pyodermas is a bacterial skin infection.

299
Q

When conducting a skin assessment, the nurse notes a purple macular lesion on the patient’s right upper extremity. The nurse differentiates the lesion as a petechia or ecchymosis based on which of the following?

a) Erythema
b) Exudate
c) Location
d) Size

A

d) Size

Explanation:
The nurse differentiates between a petechial and an ecchymosis based on the size of the involved area. Other differentiating factors include shape, color, and etiology.

300
Q

The nurse is caring for a patient receiving radiation therapy for laryngeal cancer. A late complication of radiation therapy includes which of the following?

a) Laryngeal necrosis
b) Dysphasia
c) Xerostomia
d) Pain

A

a) Laryngeal necrosis

Explanation:
Late complications of radiation therapy include laryngeal necrosis, edema, and fibrosis. Pain, xerostomia, and dysphasia are not late complications of radiation therapy.

301
Q

During a community health fair, a nurse is teaching a group of seniors about health promotion and infection prevention. Which intervention would best promote infection prevention for senior citizens who are at risk of pneumococcal and influenza infections?

a) Drink six glasses of water daily
b) Exercise daily
c) Take all prescribed medications
d) Receive vaccinations

A

d) Receive vaccinations

Explanation:
Identifying the patients who are at risk for pneumonia provides a means to practice preventive nursing care. The nurse encourages patients at risk of pneumococcal and influenza infections to receive vaccinations against these infections.

302
Q

The nurse is caring for a client who is prescribed diuretic medication for the treatment of hypertension. The nurse recognizes that which of the following medications conserves potassium?

a) Spironolactone (Aldactone)
b) Chlorthalidone (Hygroton)
c) Chlorothiazide (Diuril)
d) Furosemide (Lasix)

A

a) Spironolactone (Aldactone)
Explanation:
Aldactone is known as a potassium-sparing diuretic. Lasix causes loss of potassium from the body. Diuril causes mild hypokalemia. Hygroton causes mild hypokalemia.

303
Q

The nurse is conducting discharge teaching for a patient with diverticulosis. Which of the following should the nurse include in the teaching?

a) Drink 8 to 10 glasses of fluid daily.
b) Avoid unprocessed bran.
c) Use laxatives weekly.
d) Avoid daily exercise.

A

a) Drink 8 to 10 glasses of fluid daily.
Explanation:
The nurse should instruct a patient with diverticulosis to drink at least 8 to 10 large glasses of fluid every day. The patient should include unprocessed bran in the diet because it adds bulk and should avoid the use of laxatives or enemas except when recommended by the physician. In addition, the patient should exercise regularly if his or her current lifestyle is somewhat inactive.

304
Q

A patient is scheduled for elective surgery. To prevent the complication of hypotension and cardiovascular collapse, the nurse should report the use of what medication?

a) Hydrochlorothiazide (HydroDIURIL)
b) Prednisone (Deltasone)
c) Erythromycin (Ery-Tab)
d) Warfarin (Coumadin)

A

b) Prednisone (Deltasone)
Explanation:
Patients who have received corticosteroids are at risk of adrenal insufficiency. Insufficiency related to corticosteroids can cause circulatory collapse and hypotension. Hydrochlorothiazide and erythromycin can cause respiratory complications. Warfarin will increase the risk of bleeding.

306
Q

The nurse is conducting a community education program on malignant melanoma. When the participants identify which of the following risk factors does the nurse know that they understand the teaching?

a) Mediterranean descent
b) Dark skin
c) History of suntans
d) Family history of pancreatic cancer

A

d) Family history of pancreatic cancer

Explanation:
A family history of pancreatic cancer is a risk factor for malignant melanoma. Additional risk factors include fairer skin, freckles, blue-eyes, blond hair, Celtic or Scandinavian descent, history of sunburns, previous melanoma, family history of melanoma, and a family or personal history of multiple atypical nevi.

307
Q

A student nurse who is employed as a nursing
assistant may perform any functions that:

  1. Have been learned in school
  2. Are expected of a nurse at that level
  3. Are identified in the position’s job description
  4. Require technical rather than professional skill
A
  1. Are identified in the position’s job description
  2. Need to perform only those tasks that appear in
    the job description for a nurse’s aide or assistan
308
Q

All of the following patients are at risk for
developing serious fluid and electrolyte imbalances
during and after surgery except:
1. Patient F, who is 1 year old and having a cleft
palate repair
2. Patient H, who is 79 years old and has a history
of congestive heart failure
3. Patient G, who is 55 years old and has a history
of chronic respiratory disease
4. Patient E, who is 81 years old and having
emergency surgery for a bowel obstruction after
4 days of vomiting and diarrhea

A

Patient G, who is 55 years old and has a history
of chronic respiratory disease

All of the other patients are predisposed to an
imbalance either to existing loses, fluid overload, or
the inability to obtain PO fluids.

309
Q

A patient is undergoing surgery with a brachial plexus block to the right wrist. The patient voices concerns about anesthesia awareness. What is the best response by the nurse?

a) “Because of the type of anesthesia used, you may be aware of what is going on around you.”
b) “Anesthesia awareness is not a concern with type of surgery you are having.”
c) “Advances in medicines used decrease the chance of anesthesia awareness. What are your major concerns?”
d) “The entire surgical team will monitor for anesthesia awareness and treat it appropriately.”

A

a) “Because of the type of anesthesia used, you may be aware of what is going on around you.”
Explanation:
Anesthesia awareness is a complication of general anesthesia. The patient is undergoing surgery with a local conduction block, not general surgery. Honest discussion of awareness is needed so patients know what to expect while they are in the operating room. Although the entire surgical team should be monitoring for anesthesia awareness, it is not relevant to the surgical procedure being performed. Telling the patient that anesthesia awareness is not a concern is dismissive of the patient’s feelings.

310
Q

A 76-year-old patient had surgery for an abdominal hernia. The PACU nurse assesses that the patient is confused and is trying to climb out of the bed and pull at the cardiac monitor lines. At this time, what interventions by the nurse are appropriate? Select all that apply.

a) Ambulate the patient.
b) Administer opioid pain medication per orders.
c) Reorient the patient.
d) Assess for urine output.
e) Assess for hypoxia.
f) Apply wrist restraints.

A

c) Reorient the patient., d) Assess for urine output., e) Assess for hypoxia.
Explanation:
The nurse should provide reassurance and reorient the patient as needed. Hypoxia and urinary retention may cause acute confusion in the older adult postoperative patient, so it would be appropriate for the nurse to assess for hypoxia and urine output. Opioid pain medications may cause further confusion; consultation with the physician about the type and dosage of the pain medication should occur. Ambulating the patient may be a safety issue, especially if the patient is bleeding or hypoxic. Applying wrist restraints should only be used as a last resort.

311
Q

When working with an older person, you would keep in mind that the older person is most likely to experience which of following changes with aging?

  1. Thinning of the epidermis
  2. Thickening of the epidermis
  3. Oiliness of the skin
  4. Increased elasticity of the skin
A
  1. Thinning of the epidermis, The epidermis thins with aging, and there is decreased strength and elasticity of the skin, increased dryness and scaliness of the skin, and diminished pain perception due to decreased sensation of pressure and light touch.
313
Q

Contemporary nursing requires that the nurse has knowledge and skills for a variety of professional roles and responsibilities. Which of the following are examples? (Select all that apply.)

A. Caregiver

B. Autonomy and accountability

C. Patient advocate

D. Health promotion

E. Lobbyist

A

A,B,C,D

314
Q

You find that your newly assigned client has very shiny skin on their legs, has little or no leg hair, and the client reports that their skin damages easily. You would suspect that these signs and symptoms are related to:

  1. Overuse of caustic products to strip the leg hair.
  2. Chronic neurological pathology.
  3. Impaired peripheral arterial circulation.
  4. Inherited reduction in sweat glands and hair follicles.
A
  1. Impaired peripheral arterial circulation; Shiny skin on the legs, reduction in or absence of leg hair, and skin that damages easily is often related to impaired peripheral arterial circulation.
315
Q

Which of the following should be incorporated into the patient teaching plan to prevent deep vein thrombosis?

a) Use of blanket rolls for elevation of the lower extremities
b) Hourly leg exercises
c) Fluid restriction
d) Prolonged dangling at the edge of the bed

A

b) Hourly leg exercises
Explanation:
The benefits of early amb) Hourly leg exercises

Explanation:

The benefits of early ambulation and hourly leg exercises in preventing deep vein thrombosis cannot be overemphasized. It is important to avoid the use of blanket rolls, pillow rolls, or any form of elevation that constricts vessels under the knees. Prolonged dangling can be dangerous and is not recommended in susceptible patients because the pressure under the knees can impede circulation. Dehydration adds to the risk of thrombosis formation.bulation and hourly leg exercises in preventing deep vein thrombosis cannot be overemphasized. It is important to avoid the use of blanket rolls, pillow rolls, or any form of elevation that constricts vessels under the knees. Prolonged dangling can be dangerous and is not recommended in susceptible patients because the pressure under the knees can impede circulation. Dehydration adds to the risk of thrombosis formation.

317
Q

A patient asks why there is a drain pulling fluid from the surgical wound. What is the best response by the nurse?

a) “It will cut down on the number of dressing changes needed.”
b) “The drain will remove the necrotic tissue.”
c) “Most surgeons use wound drains now.”
d) “It assists in preventing infection.”

A

d) “It assists in preventing infection.”

Explanation:
A wound drain assists in preventing infection by removing the medium in which bacteria would grow. The purpose of the wound drain is not to remove necrotic tissue or to decrease the number of dressing changes. Stating that most surgeons use wound drains does not answer the patient‘s question appropriately.

318
Q

A patient with a recent myocardial infarction was admitted to the hospital with a new diagnosis of mitral valve regurgitation. Which of the following assessment data obtained by the nurse should be immediately communicated to the health care provider?

a) The patient has 4+ peripheral edema in both legs.
b) The patient has crackles audible throughout the lungs.
c) The patient has a loud systolic murmur all across the precordium.
d) The patient has a palpable thrill felt over the left anterior chest.

A

b) The patient has crackles audible throughout the lungs.
Explanation:
Acute mitral regurgitation, resulting from a myocardial infarction, usually manifests as severe congestive heart failure. Dyspnea, fatigue and weakness are the most common symptoms. Palpitations, shortness of breath on exertion and cough from pulmonary congestion also occur. Crackles that are audible throughout the lungs indicate that the patient is experiencing severe left ventricular failure with pulmonary congestion and need immediate interventions, such as diuretics.

319
Q

The nurse is conducting a gastrointestinal assessment. When the patient complains of the presence of mucus and pus in his stools, the nurse assesses for additional signs/symptoms of which of the following disease/conditions?

a) Disorders of the colon
b) Ulcerative colitis
c) Intestinal malabsorption
d) Small-bowel disease

A

b) Ulcerative colitis
Explanation:
The presence of mucus and pus in the stools suggests ulcerative colitis. Watery stools are characteristic of small-bowel disease. Loose, semisolid stools are associated more often with disorders of the colon. Voluminous, greasy stools suggest intestinal malabsorption.

321
Q

The nurse is caring for a patient with recurrent hemoptysis who has undergone a bronchoscopy. Immediately following the procedure, the nurse should complete which of the following?

a) Ensure the patient remains moderately sedated to decrease anxiety.
b) Assess the patient for a cough reflex.
c) Instruct the patient that bed rest must be maintained for 2 hours.
d) Offer the patient ice chips.

A

b) Assess the patient for a cough reflex.

Explanation:
After the procedure, the patient must take nothing by mouth until the cough reflex returns, because the preoperative sedation and local anesthesia impair the protective laryngeal reflex and swallowing. Once the patient demonstrates a cough reflex, the nurse may offer ice chips and eventually fluids. The patient is sedated during the procedure, not afterward. The patient is not required to maintain bed rest following the procedure.

322
Q

What is the priority action by the scrub nurse when the surgeon is starting to close the surgical wound?

a) Obtain a sponge count.
b) Label the tissue specimen.
c) Prepare the needed sutures.
d) Handing needed equipment to the surgeon.

A

a) Obtain a sponge count.
Explanation:
Standards call for the scrub nurse and the circulating nurse to obtain a sponge count at the beginning of the surgery when the surgical wound is being sutured and when the skin is being sutured. Tissue specimens should be labeled when obtained. The sutures should be ready prior to the surgeon needing them. While the scrub nurse hands equipment to the surgeon, the sponge count is a higher priority action.

323
Q

A patient comes into the hospital with a Tegaderm dressing in place on his buttocks. The nurse documents this as being which type of dressing?

a) Inactive
b) Interactive
c) Active
d) Passive

A

d) Passive

Explanation:
There are three categories of wound dressings: active, passive and interactive. The nurse labels Tegaderm as being a passive dressing, a dressing that acts as a protective, moist environment for natural healing. Interactive dressings absorb wound exudate and include hydrocolloids. Active dressings decrease healing time to improve the healing process and include biologic skin substitutes.

324
Q

The nurse is triaging the surgical patients. Which patient would the nurse document as urgent for surgical care?

a) A patient with severe bleeding
b) A patient scheduled for cosmetic surgery
c) A patient needing cataract surgery
d) A patient with an acute gallbladder infection

A

d) A patient with an acute gallbladder infection
Explanation:
An acute gallbladder infection is considered an urgent surgical procedure. Cosmetic surgery and cataract surgery are not considered urgent surgical procedures. Severe bleeding could be considered an emergent surgical procedure.

326
Q

Health care reform will bring changes in the emphasis of care. Which of the following models is expected from health care reform?

A. Moving from an acute illness to a health promotion, illness prevention model

B. Moving from illness prevention to a health promotion model

C. Moving from an acute illness to a disease management model

D. Moving from a chronic care to an illness prevention model

A

A. Moving from an acute illness to a health promotion, illness prevention model

Health care reform also affects how health care is delivered. There is greater emphasis on health promotion, disease prevention, and management of illness.

327
Q

Which of the following terms will the nurse use to document the inability of a patient to breathe easily unless positioned upright?

a) Dyspnea
b) Hemoptysis
c) Orthopnea
d) Hypoxemia

A

c) Orthopnea

Explanation:
Orthopnea is the term used to describe a patient’s inability to breathe easily except in an upright position. Orthopnea may be found in patients with heart disease and, occasionally, in patients with COPD. Patients with orthopnea are placed in a high Fowler’s position to facilitate breathing. Dyspnea refers to labored breathing or shortness of breath. Hemoptysis refers to expectoration of blood from the respiratory tract. Hypoxemia refers to low oxygen levels in the blood.

329
Q

The nurse auscultates crackles in a pt with a respiratory disorder. With what disorder would crackles be commonly heard?

  1. Asthma
  2. Bronchospasm
  3. Collapsed alveoli
  4. Pulmonary fibrosis
A

Collapsed alveoli

330
Q

A nurse reviewing a patient’s echocardiogram report reads the following statements: “The heart muscle is asymmetrically thickened and has an increase in overall size and mass, especially along the septum. The ventricular walls are thickened reducing the size of the ventricular cavities. Several areas of the myocardium have evidence of scaring.” The nurse knows these manifestations are indicative of which type of cardiomyopathy?

a) Hypertrophic
b) Dilated
c) Restrictive
d) Arrhythmogenic right ventricular cardiomyopathy

A

a) Hypertrophic
Explanation:
In hypertrophic cardiomyopathy (HCM), the heart muscle asymmetrically increases in size and mass, especially along the septum. It often affects nonadjacent areas of the ventricle. The increased thickness of the heart muscle reduces the size of the ventricular cavities and causes the ventricles to take a longer time to relax after systole. The coronary arteriole walls are also thickened, which decreases the internal diameter of the arterioles. The narrow arterioles restrict the blood supply to the myocardium, causing numerous small areas of ischemia and necrosis. The necrotic areas of the myocardium ultimately fibrose and scar, further impeding ventricular contraction. Because of the structural changes, HCM had also been called idiopathic hypertrophic subaortic stenosis (IHSS) or asymmetric septal hypertrophy (ASH). RCM is characterized by diastolic dysfunction caused by rigid ventricular walls that impair ventricular stretch and diastolic filling. Arrhythmogenic right ventricular cardiomyopathy (ARVC) occurs when the myocardium of the right ventricle is progressively infiltrated and replaced by fibrous scar and adipose tissue.

331
Q

Which of the following diagnostic tests is used to examine cells from herpes zoster?

a) Skin biopsy
b) Tzanck smear
c) Patch testing
d) Skin scrapings

A

b) Tzanck smear

Explanation:
A Tzanck smear is a test used to examine cells from blistering skin conditions such as herpes zoster. Biopsies are performed on skin nodules, plaques, blisters, and other lesions to rule out malignancy and to establish an exact diagnosis. Skin scraping is used to diagnose spores and hyphae. A patch test is used to identify substances to which the patient has developed an allergy.

332
Q

Which of the following is the primary lesion associated with acne caused by sebum blockage in hair follicles?

a) Carbuncle
b) Comedones
c) Furuncle
d) Striae

A

b) Comedones

Explanation:
A comedone is the primary lesion of acne, caused by sebum blockage in the hair follicle. A furuncle is a localized skin infection of a single hair follicle. A carbuncle is a localized skin infection involving several hair follicles. Striae are bandlike streaks on the skin, distinguished by color, texture, depression, or elevation from the tissue in which they are found.

333
Q

The nurse is assessing a patient’s skin when the patient points out a mole. The nurse brings the mole to the physician’s attention when which of the following is noted?

a) Uniform light brown color
b) Diameter exceeding 6 mm
c) Symmetric in appearance
d) Distinct borders

A

b) Diameter exceeding 6 mm

Explanation:
The nurse brings the mole to the physician’s attention when characteristics of melanoma are detected such as diameter exceeding 6 mm. Other characteristics of melanoma include asymmetric appearance; irregular, indistinct borders; and red, white, or blue coloration.

334
Q

During the preoperative assessment, the patient states he is allergic to avocados, bananas, and hydrocodone (Vicodin). What is the priority action by the nurse?

a) Notify the dietary department.
b) Notify the surgical team to remove all latex-based items.
c) Notify the nurse manager to follow up on the procedure.
d) Notify the physician regarding postoperative pain medications.

A

b) Notify the surgical team to remove all latex-based items.
Explanation:
Allergies to avocados and bananas may indicate an allergy to latex. Although it is necessary to notify the dietary department and physician, it is not an immediate threat, as the patient is NPO (nothing by mouth) and pain medication will be ordered postoperatively. The nurse manager does not need to be notified of the patient’s allergies.

335
Q

nurse is caring for a patient with end-stage lung disease. The patient wants to go home on oxygen and be comfortable. The family wants the patient to have a new surgical procedure. The nurse explains the risk and benefits of the surgery to the family and discusses the patient’s wishes with the family. The nurse is acting as the patient’s:

A. Educator

B. Advocate

C. Caregiver

D. Case manager

A

B. Advocate

An advocate protects the patient’s human and legal right to make choices about his or her care. An advocate may also provide additional information to help a patient decide whether or not to accept a treatment or find an interpreter to help family members communicate their concerns.

336
Q

Your client has a pressure ulcer over the sacral area that is believed to be due to shearing force. The client’s family asks you to explain shearing force. You would be most accurate if you tell the family that shearing force involves:

  1. A tearing of the muscle tissue due to a considerable downward force.
  2. A sudden break in skin integrity due to being pulled against the bed linens.
  3. A superficial skin fold getting pinched, and tissues irritated by the pressure.
  4. Superficial skin surface relatively unmoving in relation to the bed surface.
A
  1. A superficial skin fold getting pinched, and tissues irritated by the pressure; Shearing force is a combination of friction and pressure with skin surface unmoving in relation to the bed surface, while deeper tissue attached to the skeleton tends to move with the body.
337
Q

A PACU nurse receives a postoperative patient who received general anesthesia with a hard plastic oral airway in place. The patient has clear lung sounds, even and unlabored respirations of 16, and an oxygen saturation of 98%. The patient is minimally responsive to painful stimuli. What action by the nurse is most appropriate?

a) Continue with frequent patient assessments.
b) Remove the oral airway.
c) Notify the physician of impaired neurological status.
d) Obtain vital signs, including pulse oximetry, every 5 minutes.

A

a) Continue with frequent patient assessments.
Explanation:
An immediate postoperative patient may be transferred to the PACU with a hard plastic oral airway. The airway should not be removed until the patient is showing signs of gagging or choking. The neurological status is appropriate for a patient that received general anesthesia. There is no information provided that requires the patient to have vitals taken more frequently than the standard 15 minutes. The nurse should continue with frequent patient assessments.

338
Q

Which of the following results in decreased gas exchange in older adults?

a) The elasticity of the lungs increases with age.
b) The alveolar walls become thicker.
c) The number of alveoli decreases with age.
d) The alveolar walls contain fewer capillaries.

A

d) The alveolar walls contain fewer capillaries.

Explanation:
Although the number of alveoli remains stable with age, the alveolar walls become thinner and contain fewer capillaries, resulting in decreased gas exchange. The lungs also lose elasticity and become stiffer. Lungs elasticity does not increase with age, and number of alveoli does not decrease with age.

339
Q

The nurse is caring for a patient being weaned from the mechanical ventilator. Which of the following patient findings would require the termination of the weaning process?

a) Heart rate less than 100 bpm
b) PaO2 greater than 60 mm Hg with a FiO2 less than 40%
c) Vital capacity of 12 mL/kg
d) Blood pressure increase of 20 mm Hg from baseline

A

d) Blood pressure increase of 20 mm Hg from baseline

Explanation:
In collaboration with the primary provider, the nurse would terminate the weaning process if adverse reactions occur, including a heart rate increase of 20 beats/min, systolic BP increase of 20 mm Hg, a decrease in oxygen saturation to less than 90%, respiratory rate less than 8 or greater than 20 breaths/min, ventricular dysrhythmias, fatigue, panic, cyanosis, erratic or labored breathing, and paradoxical chest movement. A vital capacity of 10 to 15 mL/kg, maximum inspiratory pressure (MIP) at least –20 cm H2O, tidal volume: 7to –9 mL/kg, minute ventilation: 6 L/min, and rapid/shallow breathing index below 100 breaths/min/L; PaO2 greater than 60 mm Hg with FiO2 less than 40% are criteria if met by the patient indicates that the patient is ready to be weaned from the ventilator. A normal vital capacity is
10 to 15 mL/kg.

340
Q

A nurse is caring for a patient post cardiac surgery. Upon assessment, the patient appears restless and is complaining of nausea and weakness. The patient’s ECG reveals peaked T waves. The nurse reviews the patient’s serum electrolytes anticipating which of the following abnormalities?

a) Hyperkalemia
b) Hypomagnesemia
c) Hyponatremia
d) Hypercalcemia

A

a) Hyperkalemia
Explanation:
Hyperkalemia is indicated by mental confusion, restlessness, nausea, weakness, and dysrhythmias (tall, peaked T waves). Hypercalcemia would likely be demonstrated by asystole. Hypomagnesemia would likely be demonstrated by hypotension, lethargy, and vasodilation. Hyponatremia would likely be indicated by weakness, fatigue, and confusion without change in T-wave formation.

341
Q

A patient has undergone a laryngectomy. The nurse notes evidence of wound breakdown. The nurse understands that the patient is at a high risk for developing which of the following?

a) Pneumonia
b) Dehydration
c) Pulmonary embolism
d) Carotid hemorrhage

A

d) Carotid hemorrhage

Explanation:
The carotid artery lies close to the stoma and may rupture from erosion if the wound does not heal properly. If wound breakdown occurs, the patient must be monitored carefully and identified as at high risk for carotid hemorrhage. Pulmonary embolism is associated with immobility. Dehydration may lead to poor wound healing and breakdown. Pneumonia is a risk for any postoperative patient.

342
Q

The nurse puts restraints on a patient without the
patient’s permission and without a physician’s
order. The nurse may be guilty of:
1. Battery
2. Assault
3. Neglect
4. Invasion of privacy

A

Battery

Unintentional touching without consent

344
Q

A patient with newly diagnosed emphysema is admitted to the medical–surgical unit for evaluation. Which of the following does the nurse recognize is a deformity of the chest wall that occurs as a result of overinflation of the lungs in this patient population?

a) Barrel chest
b) Pigeon chest
c) Funnel chest
d) Kyphoscoliosis

A

a) Barrel chest

Explanation:
A barrel chest occurs as a result of over inflation of the lungs. There is an increase in the anteroposterior diameter of the thorax. It occurs with aging and is a hallmark sign of emphysema and chronic obstructive pulmonary disease (COPD). In a patient with emphysema, the ribs are more widely spaced and the intercostal spaces tend to bulge on expiration. Funnel chest occurs when there is a depression in the lower portion of the sternum, which may result in murmurs. Pigeon chest occurs as a result of displacement of the sternum resulting in an increase in the anteroposterior diameter. Kyphoscoliosis is characterized by elevation of the scapula and a corresponding S-shaped spine. This deformity limits lung expansion within the thorax.

345
Q

The nursing instructor is teaching students about the types of lung cancer. Which type of lung cancer is characterized as fast growing and can arise peripherally?

a) Adenocarcinoma
b) Squamous cell carcinoma
c) Large cell carcinoma
d) Bronchoalveolar carcinoma

A

c) Large cell carcinoma

Explanation:
Large cell carcinoma is a fast-growing tumor that tends to arise peripherally. Bronchoalveolar cell cancer arises from the terminal bronchus and alveoli and is usually slow growing. Adenocarcinoma presents as peripheral masses or nodules and often metastasizes. Squamous cell carcinoma arises from the bronchial epithelium and is more centrally located.

346
Q

The nurse is caring for a patient diagnosed with pneumonia. The nurse will assess the patient for tactile fremitus by completing which of the following?

a) Asking the patient to say “one, two, three” while auscultating the lungs
b) Asking the patient to repeat “ninety-nine” as the nurse’s hands move down the patient’s thorax
c) Placing the thumbs along the costal margin of the chest wall and instructing the patient to inhale deeply
d) Instructing the patient to take a deep breath and hold it while the diaphragm is percussed

A

b) Asking the patient to repeat “ninety-nine” as the nurse’s hands move down the patient’s thorax

Explanation:
While the nurse is assessing for tactile fremitus, the patient is asked to repeat “ninety-nine” or “one, two, three,” or “eee, eee, eee” as the nurse’s hands move down the patient’s thorax. The vibrations are detected with the palmar surfaces of the fingers and hands, or the ulnar aspect of the extended hands, on the thorax. The hand or hands are moved in sequence down the thorax. Corresponding areas of the thorax are compared. Asking the patient to say “one, two, three” while auscultating the lungs is not the proper technique for assessing for tactile fremitus. The nurse assesses for anterior respiratory excursion by placing the thumbs along the costal margin of the chest wall and instructing the patient to inhale deeply. The nurse assesses for diaphragmatic excursion by instructing the patient to take a deep breath and hold it while the diaphragm is percussed.

347
Q

The nurse caring for a patient with repeated episodes of contact dermatitis is providing instruction to prevent future episodes. Which of the following should the nurse include?

a) Avoid cosmetics with fragrance.
b) Use a fabric softener.
c) Wear gloves during the day.
d) Wash skin in very hot water.

A

a) Avoid cosmetics with fragrance.

Explanation:
The nurse should teach the patient to avoid cosmetics, soaps, and laundry detergents that contain fragrance. Other prevention methods include avoidance of heat and fabric softeners. Glove use for cleaning and washing dishes should be limited to no longer than 15 to 20 minutes/day, and cotton-lined gloves should be used.

348
Q

The nurse caring for a patient with tuberculosis anticipates administering which vitamin with isoniazid (INH) to prevent INH-associated peripheral neuropathy?

a) Vitamin B6
b) Vitamin D
c) Vitamin C
d) Vitamin E

A

a) Vitamin B6

Explanation:
 Vitamin B6 (pyridoxine) is usually administered with INH to prevent INH-associated peripheral neuropathy. Vitamins C, D, and E are not appropriate.
349
Q

Which of the following infecting agents is the cause of scabies?

a) Bacteria
b) Reactivated virus
c) Itch mite
d) Parasitic fungi

A

c) Itch mite

Explanation:
Several skin disorders involve an infecting agent. Scabies is caused by an itch mite. Parasitic fungi cause dermatophytosis in skin, scalp, and nails. Shingles is caused by a reactivated virus.

350
Q

The nurse caring for a patient with diverticulitis is preparing to administer the patient’s medications. The nurse anticipates administration of which category of medications due to the patient’s diverticulitis?

a) Antispasmodic
b) Anti-inflammatory
c) Antianxiety
d) Antiemetic

A

a) Antispasmodic
Explanation:
The nurse anticipates administration of antispasmodic medication to decrease intestinal spasm associated with diverticulitis. The patient may also be ordered an opioid analgesic to relieve the associated pain. There is no indication that the patient needs antianxiety, antiemetic, or anti-inflammatory medications at this time.

351
Q

During the auscultation of a patient’s heart sounds, the nurse notes an S4. The nurse recognizes that an S4 is associated with which of the following?

a) Hypertensive heart disease
b) Turbulent blood flow
c) Heart failure
d) Diseased heart valves

A

a) Hypertensive heart disease

Explanation:
Auscultation of the heart requires familiarization with normal and abnormal heart sounds. An extra sound just before S1 is an S4 heart sound, or atrial gallop. An S4 sound often is associated with hypertensive heart disease. A sound that follows S1 and S2 is called an S3 heart sound or a ventricular gallop. An S3 heart sound is often an indication of heart failure in an adult. In addition to heart sounds, auscultation may reveal other abnormal sounds, such as murmurs and clicks, caused by turbulent blood flow through diseased heart valves.

352
Q

A nurse is caring for a patient in the cardiovascular intensive care unit (CVICU) following a coronary artery bypass graft (CABG). Which of the following clinical findings requires immediate intervention by the nurse?

a) Heart rate: 66 bpm
b) Pain score: 5/10.
c) Blood pressure: 110/68 mmHg
d) CVP reading: 1 mmHg

A

d) CVP reading: 1 mmHg
Explanation:
The central venous pressure (CVP) reading of 1 is low (2–6 mmHg) and indicates reduced right ventricular preload, commonly caused by hypovolemia. Hypovolemia is the most common cause of decreased cardiac output after cardiac surgery. Replacement fluids such as colloids, packed red blood cells, or crystalloid solutions may be prescribed. The other findings require follow-up by the nurse; however, addressing the CVP reading is the nurse’s priority.

353
Q

The nurse assesses the patient and observes reddish-purple to dark blue macules, plaques, and nodules. The nurse recognizes that these manifestations are associated with which of the following conditions?

a) Syphilis
b) Platelet disorders
c) Allergic reactions
d) Kaposi’s sarcoma

A

d) Kaposi’s sarcoma

Explanation:
Kaposi’s sarcoma is a frequent comorbidity of the patient with AIDS. With platelet disorders, the nurse observes ecchymosis (bruising) and purpura (bleeding into the skin). Urticaria (wheals or hives) is the manifestation of allergic reactions. A painless chancre or ulcerated lesion is a typical finding in the patient with syphilis

354
Q

The antibiotic of choice utilized in the treatment of acute bacterial rhinosinusitis (ABRS) includes which of the following?

a) Levofloxacin (Levaquin)
b) Keflex (Cephalexin)
c) Amoxicillin (Augmentin)
d) Ceftin (Cefuroxime)

A

c) Amoxicillin (Augmentin)

Explanation:
Antibiotics should be administered as soon as the diagnosis of ABRS is established. Amoxicillin-clavulanate (Augmentin) is the antibiotic of choice. For patients who are allergic to penicillin, doxycycline (Vibramycin) or respiratory quinolones, such as levofloxacin (Levaquin) or moxifloxacin (Avelox), can be used. Other antibiotics previously prescribed to treat ABRS, including cephalosporins such as cephalexin (Keflex) and cefuroxime (Ceftin), are no longer recommended as they are not effective in treating antibiotic-resistant organisms that are now more commonly implicated in ABRS.

355
Q

The circulating nurse is documenting all medications administered during a surgical procedure. The anesthesiologist administers an opioid analgesic. What medication would the nurse check as having being administered?

a) Fentanyl (Sublimaze)
b) Metocurine (Metubine)
c) Mivacurium (Mivacron)
d) Etomidate (Amidate)

A

a) Fentanyl (Sublimaze)
Explanation:
Fentanyl is an opioid analgesic. Mivacurium and metocurine are muscle relaxants. Etomidate is an anesthetic agent

356
Q

Which of the following is an appropriate
intervention to maintain the respiratory system
of the immobilized patient?
1. Turn the patient every 4 hours.
2. Maintain a maximum fluid intake of 1500 mL/
day.
3. Apply an abdominal binder continuously while
the patient is in bed.
4. Encourage the patient to deep breathe and cough
every 1 to 2 hours

A
  1. Encourage the patient to deep breathe and cough
    every 1 to 2 hours

This technique produces a forceful, productive
cough without excessive fatigue

357
Q

Which actions should a nurse perform to prevent deep vein thrombosis when caring for a postsurgical patient?

a) Maintain bed rest.
b) Reinforce the need to perform leg exercises every hour when awake.
c) Instruct the patient to prop pillow under the knees.
d) Administer prophylaxis high-dose heparin.

A

b) Reinforce the need to perform leg exercises every hour when awake.

Explanation:
The nurse should reinforce the need to perform leg exercises every hour when awake. Maintaining bed rest increases the pooling of blood in the lower extremities, increasing the risk for deep vein thrombosis. The patient may be given low-dose heparin for prophylaxis treatment but not a high-dose heparin. The nurse should instruct the patient not to prop a pillow under the knees because the patient can constrict the blood vessels.

358
Q

The nurse is caring for a patient with systolic blood pressure of 135 mm Hg. This finding would be classified as which of the following?

a) Stage 1 hypertension
b) Prehypertension
c) Normal
d) Stage 2 hypertension

A

b) Prehypertension
Explanation:
A systolic blood pressure of 135 mm Hg is classified as prehypertension. A systolic BP of less than 120 mm Hg is normal. A systolic BP of 140 to 159 mm Hg is stage I hypertension. A systolic BP of greater than or equal to 160 is classified as stage II hypertension.

359
Q

The nurse knows that there are three types of chronic pharyngitis. Which of the following is characterized by numerous swollen lymph follicles on the pharyngeal wall?

a) Aphonia
b) Chronic granular
c) Atrophic
d) Hypertrophic

A

b) Chronic granular

Explanation:
Chronic granular pharyngitis is characterized by numerous swollen lymph follicles on the pharyngeal wall. Aphonia refers to the inability to use one’s voice. Atrophic pharyngitis is characterized by a membrane that is thin, white, glistening, and at times wrinkled. Hypertrophic pharyngitis is characterized by general thickening and congestion of the pharyngeal mucous membrane.

360
Q

The nurse is interpreting blood gases for a patient with acute respiratory distress syndrome (ARDS). Which set of blood gas values indicates respiratory acidosis?

a) pH 7.47, Paco2 28, HCO3 30
b) pH 7.49, Paco2 34, HCO3 25
c) pH 7.25, Paco2 48, HCO3 24
d) pH 7.87, Paco2 38, HCO3 28

A

c) pH 7.25, Paco2 48, HCO3 24

Explanation:
pH 7.25, Paco2 48, HCO3 24 = respiratory acidosis pH 7.87, Paco2 38, HCO3 28 = metabolic alkalosis pH 7.47, Paco2 28, HCO3 30 = respiratory alkalosis pH 7.49, Paco2 34, HCO3 25 = respiratory alkalosis

361
Q

. In a situation in which there is insufficient staff to
implement competent care, a nurse should:
1. Organize a strike
2. Refuse the assignment
3. Inform the patients of the situation
4. Accept the assignment but make a protest in
writing to the administration

A
  1. Accept the assignment but make a protest in
    writing to the administration

Need to follow the institution’s policies and
procedures on how to handle these situations and
use the chain of command

362
Q

A nurse caring for a patient with cardiomyopathy determines a diagnosis of anxiety related to a fear of death. Which of the following patient behaviors would indicate to the nurse that the patient’s level of anxiety has decreased?

a) The patient eagerly awaits visits from family.
b) The patient is resting in bed watching TV.
c) The patient answers questions about physical status with no problem.
d) The patient is able to discuss the prognosis freely.

A

d) The patient is able to discuss the prognosis freely.
Explanation:
As anxiety decreases, patients will be able to discuss prognosis freely, verbalize fears and concerns, and participate in support groups.

363
Q

The nurse is caring for a patient who is to undergo a thoracentesis. In preparation for the procedure, the nurse will position the patient in which of the following positions?

a) Prone
b) Supine
c) Sitting on the edge of the bed
d) Lateral recumbent

A

c) Sitting on the edge of the bed

Explanation:
If possible, it is best to place the patient upright or sitting on the edge of the bed with the feet supported and arms and head on a padded over-the-bed table. Other positions in which the patient could be placed include straddling a chair with arms and head resting on the back of the chair, or lying on the unaffected side with the head of the bed elevated 30 to 45 degrees if unable to assume a sitting position.

364
Q

A postanesthesia care unit (PACU) nurse is preparing to discharge a patient home following ankle surgery. The patient keeps staring at the ceiling while being given discharge instructions. What action by the nurse is appropriate?

a) Review the instructions with the patient and accompanying adult.
b) Ask the patient, “Do you understand?”
c) Give the written instructions to the patient’s 16-year-old child.
d) Continuously repeat the instructions until the patient restates them.

A

a) Review the instructions with the patient and accompanying adult.
Explanation:
The effects of the anesthesia may impair the memory or concentration of the patient. It is important that the discharge instructions are covered with the patient and an accompanying adult. Giving the instructions to a 16-year-old is not appropriate. Repeating the instruction until the patient restates them does not ensure that the patient will remember them because of how anesthesia can impair the memory. Asking if the patient understands the instructions only elicits an yes or no answer but does not give insight on if the patient comprehending the instructions

366
Q

Postoperative day 2, a patient requires wound care for a surgical wound using second-intention healing. What type of dressing change should the nurse anticipate doing?

a) Cleaning the wound with sterile saline and applying cyanoacrylate tissue adhesive
b) Packing the wound bed with sterile saline–soaked dressing and covering with dry dressing
c) Cleaning the wound with soap and water, then leaving open to air
d) Covering the well approximated wound edges with a dry dressing

A

b) Packing the wound bed with sterile saline–soaked dressing and covering with dry dressing
Explanation:
Postoperative surgical wounds that are allowed to heal using second-intention healing are usually packed with a sterile saline dressing and covered with a dry dressing. The edges of a second-intention healing wound are not approximated. The wound may be cleaned using sterile saline but the nurse would not apply a cyanoacrylate tissue adhesive. The wound should not be left open to the air, as it could expose the wound to microorganisms and dry out the wound bed impairing healing.

367
Q

A patient in the emergency department has developed wheezing and shortness of breath. The nurse gives the ordered medicated nebulizer treatment now and in 4 hours. Which standard of practice is performed?

A. Planning

B. Evaluation

C. Assessment

D. Implementation

A

D. Implementation

Implementation is completing coordinating care and the prescribed plan of care.

369
Q

When performing a skin assessment, the nurse notes a localized skin infection of a single hair follicle. The nurse documents the presence of which of the following?

a) Cheilitis
b) Carbuncle
c) Comedone
d) Furuncle

A

d) Furuncle

Explanation:
Furuncles are localized skin infections of a single hair follicle. They can occur anywhere on the body, but are most prevalent in areas subjected to irritation, pressure friction, and excessive perspiration, such as the back of the neck, the axillae, or the buttocks. A carbuncle is a localized skin infection involving several hair follicles. Cheilitis refers to dry cracking at the corners of the mouth. Comedones are the primary lesions of acne, caused by sebum blockage in the hair follicle.

370
Q

The nurse is caring for a patient with a skin lesion that is oozing. The surrounding skin is acutely inflamed. What type of dressing should the nurse apply?

a) Protective
b) Occlusive
c) Interactive
d) Passive

A

c) Interactive

Explanation:
The nurse should apply an interactive dressing to absorb the exudate while providing a moist environment for wound healing. Protective and passive dressings do not absorb exudate; they only have a protective function. Occlusive dressings are used to cover topical medication.

372
Q

The nurse has instructed a patient on how to perform pursed-lip breathing. The nurse recognizes the purpose of this type of breathing is to accomplish which of the following?

a) Promote the patient’s ability to intake oxygen
b) Promote a more efficient and controlled ventilation and to decrease the work of breathing
c) Promote the strengthening of the patient’s diaphragm
d) Improve oxygen transport, induce a slow, deep breathing pattern, and assist the patient to control breathing

A

d) Improve oxygen transport, induce a slow, deep breathing pattern, and assist the patient to control breathing

Explanation:
Pursed-lip breathing, which improves oxygen transport, helps induce a slow, deep breathing pattern and assists the patient to control breathing, even during periods of stress. This type of breathing helps prevent airway collapse secondary to loss of lung elasticity in emphysema.

373
Q

A patient diagnosed with a myocardial infarction (MI) has begun an active rehabilitation program. The nurse recognizes an overall goal of rehabilitation for a patient who has had an MI includes which of the following?

a) Limiting the effects and progression of atherosclerosis
b) Prevention of another cardiac event
c) Returning the patient to work and a preillness lifestyle
d) Improvement of the quality of life

A

d) Improvement of the quality of life
Explanation:
Overall, cardiac rehabilitation is a complete program dedicated to extending and improving quality of life. An immediate objective of rehabilitation of the MI patient is to limit the effects and progression of atherosclerosis. An immediate objective of rehabilitation of the MI patient is to return the patient to work and a preillness lifestyle. An immediate objective of rehabilitation of the MI patient is to prevent another cardiac event.

375
Q

The nurse is caring for a postoperative patient with an indwelling urinary catheter. The hourly urinary output at 9 am is 80 mL. The nurse assesses the hourly urinary output at 10 am at 20 mL. What is the highest priority action by the nurse?

a) Document the findings.
b) Irrigate the catheter with sterile normal saline.
c) Reassess the output at 11 am.
d) Notify the physician.
Show Answer

A

d) Notify the physician.

Explanation:
If the patient has an indwelling urinary catheter, hourly outputs are monitored and rates of less than 30 mL/h are reported. Any urinary output less than 30 mL/h should be reported to the physician immediately. The urinary output will be reassessed at 11 am but waiting to notify the physician could cause harm to the patient. The findings should be documented but this is not the highest priority. A urinary catheter may need to be irrigated but a postoperative patient with a low urinary output is demonstrating a complication that needs to be reported immediately.

376
Q

Which of the following actions is most appropriate for the nurse to take when the patient demonstrates subcutaneous emphysema along the suture line or chest dressing 2 hours after chest surgery?

a) Report the finding to the physician immediately.
b) Apply a compression dressing to the area.
c) Record the observation.
d) Measure the patient’s pulse oximetry.

A

b) Apply a compression dressing to the area.

Explanation:
Subcutaneous emphysema is a typical postoperative finding in the patient after chest surgery. During surgery the air within the pleural cavity is expelled through the tissue opening created by the surgical procedure. Subcutaneous emphysema is absorbed by the body spontaneously after the underlying leak is treated or halted.

377
Q

A patient has been transported to the operating room for emergent surgery. Which statement by the nurse best supports the need for emergent surgery?

a) ”The patient was unresponsive, had a distended abdomen, and unstable vital signs following a motor vehicle accident.”
b) ”The patient had severe pain and a laceration to the face with minimal bleeding after being attacked by a dog 1 hour ago.”
c) ”The patient had epigastric abdominal pain, an elevated white blood count, and vomiting for 1 day.”
d) ”The patient was tachycardic, had progressive weight loss, and bouts of insomnia as a result of hyperthyroidism.”

A

a) ”The patient was unresponsive, had a distended abdomen, and unstable vital signs following a motor vehicle accident.”
Explanation:
Emergency surgery means that the patient requires immediate attention and the disorder may be life threatening. The patient with unstable vital signs and a distended abdomen following a motor vehicle accident requires immediate attention. The patient with left sided abdominal pain may not need surgery. Epigastric pain with vomiting for 1 day is usually not an indication for emergent surgery. Lacerations to the face require sutures, not emergent surgery. A thyroidectomy to treat hyperthyroidism is a required surgery, not an emergent one.

377
Q

What is the priority action when the circulating nurse is completing a second verification of the surgical procedure and surgical site?

a) Review the complications and allergies with the anesthesiologist.
b) Obtain the attention of all members of the surgical team.
c) Ask the surgeon if the marked surgical site is correct.
d) Discuss the surgical procedure and surgical site with the patient.

A

b) Obtain the attention of all members of the surgical team.
Explanation:
The second verification of the surgical procedure and surgical site should include all members of the surgical team. This verification should be done at one time with all members of the team involved. The marked surgical site is confirmed with all members of the surgical team, not just the surgeon or patient. Complications, allergies, and anticipated problems are also discussed among the entire surgical team.

378
Q

A pt comes to the clinic and asks the nurse why the skin of the forehead, palms, and soles has a yellow-orange tint. There is no yellowing of the sclera or mucous membranes. What should the nurse question the pt regarding?

  1. “Have you been ingesting large quantities of alcohol”
  2. “Have you been diagnosed with Addison’s disease?”
  3. “Have you been out in the sun a lot?”
  4. “Have you been eating large amounts of carotene-rich foods?”
A

“Have you been eating large amounts of carotene-rich foods”

379
Q

The area of the heart that is located at the third intercostal (IC) space to the left of the sternum is which of the following?

a) Epigastric area
b) Aortic area
c) Pulmonic area
d) Erb’s point

A

d) Erb’s point
Explanation:
Erb’s point is located at the third IC space to the left of the sternum. The aortic area is located at the second IC space to the right of the sternum. The pulmonic area is at the second IC space to the left of the sternum. The epigastric area is located below the xiphoid process.

380
Q

The nurse cares for a client with a wound in the late regeneration phase of tissue repair. The wound may be protected by applying a:

  1. Transparent film
  2. Hydrogel dressing
  3. Collogenase dressing
  4. Wet to dry dressing
A
  1. Transparent film; Wounds in the regeneration phase of healing need to be protected as new tissue grows. Answers 2, 3, and 4 are dressings used to remove nonviable tissue.
381
Q

It is important for the nurse to encourage the patient diagnosed with hypertension to rise slowly from a sitting or lying position for which of the following reasons?

a) Gradual changes in position provide time for the heart to increase rate of contraction to resupply oxygen to the brain.
b) Gradual changes in position help reduce the heart’s work to resupply oxygen to the brain.
c) Gradual changes in position help reduce the blood pressure to resupply oxygen to the brain.
d) Gradual changes in position provide time for the heart to reduce its rate of contraction to resupply oxygen to the brain.

A

a) Gradual changes in position provide time for the heart to increase rate of contraction to resupply oxygen to the brain.

Explanation:
It is important for the nurse to encourage the patient to rise slowly from a sitting or lying position because gradual changes in position provide time for the heart to increase its rate of contraction to resupply oxygen to the brain and not blood pressure or heart rate.

383
Q

Your client has a Braden scale score of 17. Which is the most appropriate nursing action?

  1. Assess the client again in 24h; the score is within normal limits.
  2. Implement a turning schedule; the client is at increased risk for skin breakdown.
  3. Apply a transparent wound barrier to major pressure sites; the client is at moderate risk for skin breakdown.
  4. Request an order for a special low-air-loss bed; the client is at very high risk for skin breakdown.
A
  1. Implement a turning schedule; the client is at increased risk for skin breakdown; A score ranging from 15 to 18 is considered at risk and a turning schedule is appropriate.

Option 1 requires a score above 18 (normal and ongoing assessment indicated). Option 3, moderate risk, for which a transparent barrier would be appropriate, is applied to persons with scores of 13 to 14. Option 4, very high risk, is assigned for those with a score of 9 or less.

384
Q

A health care issue often becomes an ethical
dilemma because:
1. Decisions must be made based on value systems
2. The choices involved do not appear to be clearly
right or wrong
3. Decisions must be made quickly, often under
stressful conditions
4. A patient’s legal rights coexist with a health
professional’s obligations

A
  1. The choices involved do not appear to be clearlyright or wrong

Ethical problems come from controversy and
conflict

385
Q

The nurse is assessing a patient for obstructive sleep apnea (OSA). Which of the following are signs and symptoms of OSA? Select all that apply.

a) Polycythemia
b) Insomnia
c) Loud snoring
d) Pulmonary hypotension
e) Evening headaches

A

a) Polycythemia, b) Insomnia, c) Loud snoring

Explanation:
Signs and symptoms include excessive daytime sleepiness, frequent nocturnal awakening, insomnia, loud snoring, morning headaches, intellectual deterioration, personality changes, irritability, impotence, systemic hypertension, dysrhythmias, pulmonary hypertension, corpulmonale, polycythemia, and enuresis.

386
Q

The nurse working at a physician’s office is providing teaching to the parent of a child diagnosed with Tinea captis (ringworm of the head). How often should the nurse instruct the parent to shampoo the child’s hair with Nizoral or a selenium sulfide shampoo?

a) Twice daily
b) Weekly
c) Once
d) Daily

A

d) Daily

Explanation:
The nurse should instruct the parent to shampoo the child’s hair with Nizoral or a selenium sulfide shampoo two or three times per day

388
Q

An anxious preoperative surgical patient is encouraged to concentrate on a pleasant experience or restful scene. What cognitive coping strategy would the nurse document as being used?

a) Imagery
b) Distraction
c) Progressive muscular relaxation
d) Optimistic self-recitation

A

a) Imagery
Explanation:
Imagery has proven effective for anxiety in surgical patients. Optimistic self-recitation is practiced when the patient is encouraged to recite optimistic thoughts such as, “I know all will go well.” Distraction is employed when the patient is encouraged to think of an enjoyable story or recite a favorite poem. Progressive muscular relaxation requires contracting and relaxing muscle groups and is a physical coping strategy as opposed to a cognitive strategy.

389
Q

A group that lobbies at the state and federal levels
for advancement of nurses’ role, economic interests,
and health care is the:
1. State Boards of Nursing
2. American Nurses Association
3. American Hospital Association
4. National Student Nurses Association

A
  1. American Nurses Association

The ANA’s purpose is to improve the
professional development and general welfare of
nurses.

390
Q

Stiffness of the neck or inability to bend the neck is referred to as which of the following?

a) Dysphagia
b) Aphonia
c) Nuchal rigidity
d) Xerostomia

A

c) Nuchal rigidity

Explanation:
Nuchal rigidity is the stiffness of the neck or inability to bend the neck. Aphonia is impaired ability to use one’s voice due to distress or injury to the larynx. Xerostomia is dryness of the mouth from a variety of causes. Dysphagia is difficulty swallowing.

391
Q

You are participating in a clinical care coordination conference for a patient with terminal cancer. You talk with your colleagues about using the nursing code of ethics for professional registered nurses to guide care decisions. A nonnursing colleague asks about this code. Which of the following statements best describes this code?

A. Improves self–health care

B. Protects the patient’s confidentiality

C. Ensures identical care to all patients

D. Defines the principles of right and wrong to provide patient care

A

D. Defines the principles of right and wrong to provide patient care

When giving care, it is essential to provide a specified service according to standards of practice and to follow a code of ethics. The code of ethics is the philosophical ideals of right and wrong that define the principles you will use to provide care for your patients. The code serves as a guide for carrying out nursing responsibilities to provide quality nursing care and the ethical obligations of the profession.

392
Q

A patient who has had a recent myocardial infarction develops pericarditis and complains of level 6 (on a scale of 0–10) chest pain with deep breathing. Which of these ordered pro re nata (PRN) medications will be the most appropriate for the nurse to administer?

a) Acetaminophen (Tylenol) 650 mg per os (po) every 4 hours
b) Ibuprofen (Motrin) 800 mg po every 8 hours
c) Morphine sulfate 6 mg IVP every 2–4 hours
d) Fentanyl 2 mg intravenous pyelogram (IVP) every 2–4 hours

A

b) Ibuprofen (Motrin) 800 mg po every 8 hours
Explanation:
Pain associated with pericarditis is caused by inflammation, thus nonsteroidal anti-inflammatory drugs (NSAIDs) like ibuprofen are most effective. Opioid analgesics are usually not used for the pain associated with pericarditis.

393
Q

The volume of air inhaled and exhaled with each breath is termed which of the following?

a) Tidal volume
b) Vital capacity
c) Residual volume
d) Vital capacity

A

a) Tidal volume

Explanation:
Tidal volume is the volume of air inhaled and exhaled with each breath. Residual volume is the volume of air remaining in the lungs after a maximum expiration. Vital capacity is the maximum volume of air exhaled from the point of maximum inspiration. Expiratory reserve volume is the maximum volume of air that can be exhaled after a normal inhalation.

394
Q

An 87-year-old man is admitted to the hospital for cellulitis of the left arm. He ambulates with a walker and takes a diuretic medication to control symptoms of fluid retention. Which intervention is most important to protect him from injury?
1.Leave the bathroom light on.
2. Withhold the client’s diuretic medication.
3 Provide a bedside commode.
4. Keep the side rails up.

A

3.Provide a bedside commode.

Rationale: The placement of the bedside commode next to his bed will assist in decreasing the number of steps he is required to ambulate. This will assist in protecting him from injury due to falls. Option 1: Leaving the light on would assist the client in locating the bathroom, but would not reduce the risk of fall when rushing to the bathroom. Option 2: The nurse cannot withhold a client’s medication without consulting with the primary care provider. Option 4: If the client has orders to be up with assistance and the side rails are up, he is at risk for falls as well as falling from a greater distance.

395
Q

The nurse is preparing to apply ECG electrodes to a male patient who requires continuous cardiac monitoring. Which of the following should the nurse complete to optimize skin adherence and conduction of the heart’s electrical current?

a) Clip the patient’s chest hair prior to applying the electrodes.
b) Once the electrodes are applied, change them every 72 hours.
c) Apply baby powder to the patient’s chest prior to placing the electrodes.
d) Clean the patient’s chest with alcohol prior to application of the electrodes.

A

a) Clip the patient’s chest hair prior to applying the electrodes.
Explanation:
The nurse should complete the following actions when applying cardiac electrodes: Clip (do not shave) hair from around the electrode site, if needed; if the patient is diaphoretic (sweaty), apply a small amount of benzoin to the skin, avoiding the area under the center of the electrode; debride the skin surface of dead cells with soap and water and dry well (or as recommended by the manufacturer). Change the electrodes every 24 to 48 hours (or as recommended by the manufacturer); examine the skin for irritation and apply the electrodes to different locations.

396
Q

When receiving a report at the beginning of your shift, you learn that your assigned client has a surgical incision that is healing by primary intention. You know that your client’s incision is:

  1. Well approximated, with minimal or no drainage.
  2. Going to take a little longer than usual to heal.
  3. Going to have more scarring than most incisions.
  4. Draining some serosanguineous drainage.
A
  1. Well approximated, with minimal or no drainage; Primary intention means that the wound edges are well approximated, with minimal or no tissue loss as well as formation of minimal granulation tissue and scarring.
398
Q

Which of the following is a correct endotracheal tube cuff pressure?

a) 25 mm Hg
b) 17 mm Hg
c) 13 mm Hg
d) 21 mm Hg

A

b) 17 mm Hg

Explanation:
Cuff pressures should be checked with a calibrated aneroid manometer device every 6 to 8 hours to maintain cuff pressures between 15 and 20 mm Hg. The other values are not within the normal range for adequate cuff pressure.

399
Q

he nurse obtains a health history from a patient with a prosthetic heart valve and new symptoms of infective endocarditis. Which question by the nurse is most appropriate to ask?

a) Do you have a family history of endocarditis?
b) Do you live with any domesticated animals in your home?
c) Have you recently vacationed outside of the United States?
d) Have you been to the dentist recently?

A

d) Have you been to the dentist recently?
Explanation:
Invasive procedures, particularly those involving mucosal surfaces (e.g., those involving manipulation of gingival tissue or periapical regions of teeth), can cause a bacteremia, which rarely lasts more than 15 minutes. However, if a patient has any anatomic cardiac defects or implanted cardiac devices (e.g., prosthetic heart valve, pacemaker, implantable cardioverter defibrillator [ICD]), bacteremia can cause bacterial endocarditis.

400
Q

A patient has developed a boil on the face and the nurse observes the pt squeezing the boil. What does the nurse understand is a potential severe complication of this manipulation?

  1. Scarring
  2. Brain Abscess
  3. Erythema
  4. Cellultis
A

Brain Abscess

401
Q

The nurse has medicated a postoperative patient for complaints of nausea. Which medication would the nurse document as having been given?

a) Ondansetron (Zofran)
b) Prednisone (Deltasone)
c) Propofol (Diprivan)
d) Warfarin (Coumadin)

A

a) Ondansetron (Zofran)
Explanation:
Odansetron (Zofran) is an antiemetic and one of the most commonly prescribed medications for nausea and vomiting. Warfarin (Coumadin) is an anticoagulant. Prednisone (Deltasone) is a corticosteroid. Propofol (Diprivan) is an anesthetic agent.

402
Q

The nurse is caring for a critically ill patient in the ICU). The nurse documents the patient’s respiratory rate as bradypnea. The nurse recognizes bradypnea is associated with which of the following conditions?

a) Metabolic acidosis
b) Pneumonia
c) Increased intracranial pressure
d) Pulmonary edema

A

c) Increased intracranial pressure

Explanation:
Bradypnea is associated with increased intracranial pressure, brain injury, and drug overdose. Respirations are slower than normal rate (

403
Q

The nurse is developing a plan of care for a patient with toxic epidermal necrolysis (TEN) or Stevens-Johnson syndrome. Which of the following should the nurse include?

a) Use of friction when repositioning patient
b) Continuous current of warm air
c) Frequent inspection of oral cavity
d) Limiting fluids

A

c) Frequent inspection of oral cavity

Explanation:
The nurse should frequently inspect the oral cavity of a patient with TEN or Stevens-Johnson syndrome. Additionally, care should be taken to reduce friction and shear when turning or repositioning the patient. Fluids should not be limited because these patients are susceptible to dehydration. A continuous current of warm air on denuded skin can worsen dehydration.

404
Q

Which of the following items are used to perform wound care irrigation? Select all that apply.

  1. Clean gloves
  2. Sterile gloves
  3. Refrigerated irrigating solution
  4. 60-mL syringe
A

1, 2, and 4; To irrigate a wound, the nurse uses clean gloves to remove the old dressing and to hold the basin collecting the irrigating fluid plus sterile gloves to apply the new dressing. A 60-mL syringe is the correct size to hold the volume of irrigating solution plus deliver safe irrigating pressure. The irrigation fluid should be at room or body temperature– certainly not refrigerated. 

405
Q

A home health nurse visits a client who twisted an ankle in the morning. The client has an ice bag on the ankle. Which one of the client’s chronic conditions contraindicates the use of ice?

  1. Gastritis
  2. Diabetes
  3. Glaucoma
  4. Osteoporosis
A
  1. Diabetes; Diabetes contradicts the use for ice. Clients with neurological or circulatory impairment are at risk for injury with ice use.
406
Q

The nurse is teaching a patient with an ostomy how to change the pouching system. Which of the following should the nurse include in the teaching of a patient with no peristomal skin irritation?

a) Dry skin thoroughly after washing
b) Dust with nystatin powder
c) Apply Kenalog spray
d) Apply barrier powder

A

a) Dry skin thoroughly after washing
Explanation:
The nurse should teach the patient without peristomal skin irritation to dry the skin thoroughly after washing. Barrier powder, Kenalog spray, and nystatin powder are used when there is peristomal skin irritation and/or fungal infection.

407
Q

The classic lesions of impetigo are manifested as which of the following?

a) Comedones in the facial area
b) Abscess of skin and subcutaneous tissue
c) Honey-crusted lesions on an erythematous base
d) Patches of grouped vesicles on red and swollen skin

A

c) Honey-crusted lesions on an erythematous base

Explanation:
The classic lesions of impetigo are honey-crusted lesions on an erythematous base. Comedones in the facial area are representative of acne. A carbuncle is an abscess of skin and subcutaneous tissue. Herpes zoster is exhibited by patches of grouped vesicles on red and swollen skin.